0262f1f2279eb-The CLAT Post November, 2021

You might also like

Download as pdf or txt
Download as pdf or txt
You are on page 1of 68

www.toprankers.

com

MONTHLY EDITION

NOV, 2021 ISS UE - 22 ` 100/-

M ember states of the


Shanghai Cooperation
Organisation (SCO) on Friday
1. Which of the following is
NOT true about the United
Nations Convention on
4. Which of the following is
TRUE about the protocol
which was adopted?
adopted a protocol to strengthen Transnational Organised (a) To continue exchanging
cooperation in preventing and Crime? the national legislation
combating growing menace of to combat the menace
human trafficking, especially of trafficking in persons,
women and children. especially women and
Addressing the gathering of children.
Prosecutors General from SCO
(b) To provide protection
member states via video bridge,
and assistance in victims
Solicitor General Tushar Mehta
of trafficking within
highlighted the initiatives taken
their competence.
by India to curtail the growth of
transnational organised crime, (c) To develop cooperation
including trafficking in persons, (a) The Convention between the educational
2. Which of the following (training) organizations
ratification of United Nations against Transnational
Convention for the Suppression Articles of the Indian (institutions) of the
Organized Crime
of the Traffic in Persons and Constitution prohibits SCO member states
or UNTOC is also
of the Exploitation of the and criminalises human in the field of training
known as the Palermo
Prostitution of Others as also trafficking and forced and advanced training
Convention.
United Nations Convention on labour? of prosecutors, whose
(b) it was adopted in Italy in
Transnational Organised Crime (a) Article 21 competence include
2000.
(UNCTOC). (b) Article 22 combating trafficking
(c) The United Nations in persons especially,
According to a law ministry (c) Article 23
Office on Drugs and women and children.
statement, a protocol following
Crime (UNODC) is (d) Article 24
the results of the nineteenth (d) All of the above
meeting of Prosecutors General the custodian of the
UNTOC. 3. In which of the following


was signed and adopted by the 5. India became a full member
year was the Shanghai
SCO member states The salient (d) India ratified the of the SCO in which of the
Cooperation Organisation
features of the protocol include UNTOC in 2015 following years?
established?
strengthening cooperation in becoming the fourth (a) 2011 (b) 2014
preventing and combating. South Asian country to (a) 1989 (b) 1995
(c) 2017 (d) 2019
do so. (c) 2001 (d) 2008

1. (d) India ratified the and security alliance. and children. persons especially, women and
UNTOC in 2011 becoming The Shanghai Cooperation To continue exchanging the children.
the fourth South Asian Organization (SCO) is national legislation to combat To conduct bilateral and
country to do so. an intergovernmental the menace of trafficking in multilateral activities,
The nodal agency for all organization founded in persons, especially women and including thorough
dealings with UNTOC in Shanghai on 15 June 2001. children. videoconferencing, on
India is the Central Bureau of 4. (d) All of the above To provide protection and countering and combating
Investigation (CBI). A Protocol following the assistance in victims of the threat of trafficking in
2. (c) Article 23 results of the Nineteenth trafficking within their persons, especially women and
Article 23 of the Indian meeting of Prosecutors competence. children.
Constitution explicitly General of the SCO Member To develop cooperation 5. (c) 2017
prohibits and criminalises States was signed and adopted between the educational In the historical meeting of
human trafficking and forced by the SCO Member States. (training) organizations the Heads of State Council
labour. The salient features of the (institutions) of the SCO of the SCO held in June 2017
3. (c) 2001 Protocol are as under: member states in the field in Astana, the status of a full
The Shanghai Cooperation To strengthen cooperation of training and advanced member of the Organization
Organization, also known in preventing and combating training of prosecutors, was granted to the Republic of
as the Shanghai Pact, is an growing menace of trafficking whose competence include India and the Islamic Republic
Eurasian political, economic, in persons especially, women combating trafficking in of Pakistan in this meeting.
Bhopal +91-7676564400 Indore +91-9589613810, +91-731-4987379 Prayagraj +91-8114000926, +91-8114000927 Kanpur +91-8576066660, +91-8707605589 E: support@toprankers.com
Gurugram +91-8448444207 Delhi +91-9810610466 Ranchi +91-9334969993 Lucknow +91-6390576666 Jabalpur +91-7004386936 W: www.toprankers.com
3 The CLAT Post • November 2021

Passage (Q.6-Q.10):

I ndia pushed for “safeguarding


the interests of the developing
world” as Prime Minister
Narendra Modi addressed the
G20 summit at sessions on
climate change and sustainable
development on Sunday, said
G20 Sherpa and Commerce
Minister [1].
No time-bound agreements were
reached as leaders of the world’s
top economies ended the summit
in [2], recommitting to providing
$100 billion a year to counter
climate change, and pushing for
greater vaccine equality to fight
the COVID pandemic. G20
countries also committed to more international public economy recover and (c) 1994 (d) 1992
ending international financing private financing for “green” or diversify its supply
for all new coal plants by the end environmental projects. chains. 9. Which of the following is
of 2021, but made no mention Correct about G20?
(b) India also praised
of domestic commitments on 6. Which of the following will (a) It is an informal group
the G-20’s decision
ending coal power generation. replace [1] in the above of 19 countries and
to introduce a 15%
passage? European Union
The final communique, which minimum corporate
was agreed after negotiations (a) Nirmala tax in order to make (b) World Bank and OECD
overnight spoke only of the “key Sitharaman the global financial are its representatives
relevance of achieving global net (b) Piyush Goyal architecture “more just (c) Its members account for
zero” on carbon emissions “by or (c) Nitin Gadkari and fair.” 50% of the world’s GDP.
around mid-century”. (c) India praised the
(d) Prakash Javadekar (d) All of the above
Amongst the other highlights European Union’s Indo-
of the Rome Leaders statement 7. Which of the following is Pacific strategy, as well 10. Which of the following will
was a decision to pursue the TRUE about the India’s as France’s role as its replace [2] in the above
recognition of more vaccines by stand at the summit? leader. passage?
the World Health Organization (a) India emphasised (a) Buenos Aires
(d) All of the above
under a “One Health approach” the need of robust
for the world, and providing (b) Osaka
global supply chains 8. India joined the G20 in
finances and technology for which of the following (c) Rome
and encouraged G-20
vaccine production at “mRNA years? (d) Hamburg
countries to work with
Hubs” in South Africa, Brazil
India to help the global (a) 2002 (b) 1999
and Argentina, and to mobilise

6. (b) Piyush Goyal chains and encouraged G-20 since it was established as Its members account for 85%
Addressing the media, G20 countries to work with India Finance Ministers Forum in of the world’s GDP, and two-
Sherpa and Commerce to help the global economy 1999. thirds of its population.
Minister Piyush Goyal recover and diversify its supply 9. (a) It is an informal group of The G20 Summit is formally
hailed the communique chains. India also praised the 19 countries and European known as the “Summit on
for acknowledging India’s G-20’s decision to introduce a Union Financial Markets and the
concerns on climate justice, 15% minimum corporate tax It is an informal group World Economy”.
and for getting developed in order to make the global of 19 countries and the 10. (c) Rome
nations agreed to providing financial architecture “more European Union (EU), The 2021 G20 Rome summit
finance and providing just and fair.” India praised the with representatives of the was the sixteenth meeting of
technologies to help less European Union’s Indo-Pacific International Monetary Fund Group of Twenty, which was
developed nations. strategy, as well as France’s role and the World Bank. held in Rome, the capital city
7. (d) All of the above as its leader. It does not have a permanent of Italy, on 30–31 October
India emphasised the need 8. (b) 1999 secretariat or Headquarters. 2021.
of robust global supply India is a member of the G20
Bhopal +91-7676564400 Indore +91-9589613810, +91-731-4987379 Prayagraj +91-8114000926, +91-8114000927 Kanpur +91-8576066660, +91-8707605589 E: support@toprankers.com
Gurugram +91-8448444207 Delhi +91-9810610466 Ranchi +91-9334969993 Lucknow +91-6390576666 Jabalpur +91-7004386936 W: www.toprankers.com
4 The CLAT Post • November 2021

Passage (Q.11-Q.15): of the East Asia Summit, is NOT TRUE about India- (c) Act Develop Policy

P rime Minister Narendra committed to strengthening the ASEAN relation? (d) Act North Policy
Modi will attend the 18th East Asia Summit and making it (a) In 2022, the ASEAN-
ASEAN-India Summit on more effective for dealing with India relations will 14. Which of the following will
Thursday via video conferencing. contemporary challenges. complete 30 years. replace [2] in the above
As per the official release, the According to the release, PM passage?
(b) India became a Sectoral
East Asia Summit is the premier Modi will review the status of Partner of ASEAN in (a) 7 (b) 10
leaders-led forum in the Indo- the ASEAN-India Strategic 1992. (c) 11 (d) 16
Pacific. Since its inception in [1], Partnership at the 18th ASEAN-
it has played a significant role India Summit. He is participating (c) India became a 15. Which year has been
in the strategic and geopolitical in the summit at the invitation of Dialogue Partner in highlighted as India-
evolution of East Asia. the Sultan of Brunei. 1996. ASEAN Friendship by the
Apart from the [2] ASEAN (d) India became a Summit leaders?
11. Which of the following will level Partner in 2007.
member states, East Asia Summit (a) 2021 (b) 2022
replace [1] in the above
includes India, China, Japan,
passage? 13. ASEAN is central to which (c) 2023 (d) 2025
the Republic of Korea, Australia,
(a) 2001 (b) 2003 of the following Indian
New Zealand, the United States,
and Russia, the release said. (c) 2005 (d) 2007 Policy?
India, being a founding member (a) Act West Policy
12. Which of the following is
(b) Act East Policy

11. (c) 2005 12.(d) India became a Summit level region. India and ASEAN Laos, Malaysia, Myanmar,
The East Asia Summit (EAS) Partner in 2007. partnership conducts various the Philippines, Singapore,
process was initiated in In 2022, the ASEAN-India dialogue systems such as Thailand, and Vietnam.
2005 with the convening of relations will complete 30 a Summit, Ministerial and 15. (b) 2022
the 1st East Asia Summit years. India became a Sectoral Senior Officials meetings. The Year 2022 has been
in Kuala Lumpur, Malaysia, Partner of ASEAN in 1992, 14. (b) 10 highlighted as India-ASEAN
At its inception, the East a Dialogue Partner in 1996, The Association of Southeast Friendship by the leaders.
Asia Summit comprised 16 and a Summit level Partner in Asian Nations (ASEAN) India-ASEAN Summit is held
participating countries, namely 2002. is a regional grouping that every year and provides an
ASEAN Member States, 13. (b) Act East Policy promotes economic, political, opportunity for India and
Australia, China, India, Japan, ASEAN is central to India’s and security cooperation Asean to engage at the highest
New Zealand, and the Republic Act East Policy and the wider among its ten members: level.
of Korea. vision of the Indo-Pacific Brunei, Cambodia, Indonesia,
Bhopal +91-7676564400 Indore +91-9589613810, +91-731-4987379 Prayagraj +91-8114000926, +91-8114000927 Kanpur +91-8576066660, +91-8707605589 E: support@toprankers.com
Gurugram +91-8448444207 Delhi +91-9810610466 Ranchi +91-9334969993 Lucknow +91-6390576666 Jabalpur +91-7004386936 W: www.toprankers.com
5 The CLAT Post • November 2021


Passage (Q.16-Q.20): organizations i.e., State

T he National Mission for Program Management


Clean Ganga (NMCG) Groups (SPMGs).
got registered in the Guinness (d) It has a Rs. 20,000-crore,
Book of World Records on the centrally-funded, non-
first day of the Ganga Utsav -- lapsable corpus and
the River Festival 2021 for the consists of nearly 288
highest number of photos of projects.
handwritten notes uploaded on
Facebook in an hour. 19. National Ganga Council is
Jal Shakti Minister [1] posted chaired by who among the
his message on the Ganga following?
on Facebook, after which the (a) Prime Minister
Guinness activity was opened (b) Home Minister
for the public at large. (c) Retired Supreme Court
“Lakhs of entries were recorded Judge
during the one-hour duration (d) President
of the activity. The participation
of people from all walks of 20. Which of the following
life was particularly inspiring. is TRUE about National
Several people posted their self- Shekhawat 18. Which of the following Ganga Council?
created literary pieces on the (d) Pashupati Kumar Paras is NOT TRUE about the
(a) The National Ganga
event’s Facebook page,” a senior Namami Gange Project?
17. Which of the following Council is formed
official said. (a) Namami Gange aims at under the Environment
portals was launched by reducing the pollution
The record was established on (Protection) Act
the Union minister for Jal of the river Ganga along
the first day of the Ganga Utsav (EPA),1986.
Shakti on the inaugural day with the conservation
-- the River Festival 2021 for (b) National Mission
of Ganga Utsav? and rejuvenation of the
the highest number of photos for Clean Ganga
of handwritten notes uploaded (a) Continuous Learning river banks.
and Activity Portal (NMCG) acts as an
on the social media platform in (b) It was approved as a implementation arm
an hour. (b) Continuous Flow of ‘Flagship Programme’ of the National Ganga
Information Portal by the Union Council.
16. Which of the following will Government in June
(c) Complaints and (c) It has a two-tier
replace [1] in the above 2018.
Information Activity management structure
passage?
Portal (c) The program is being and comprises of
(a) Giriraj Singh implemented by the
(d) Continuous Portal Governing Council and
(b) Ramchandra Prasad for the Learning and National Mission for Executive Committee.
Singh Activity Clean Ganga (NMCG),
(d) All of the above
(c) Gajendra Singh and its state counterpart

16. (c) Gajendra Singh Shekhawat The Continuous Learning and 18. (b) It was approved as a The Prime Minister of India is
Ministry of Jal Shakti is a Activity Portal (CLAP) was ‘Flagship Programme’ by the the ex officio Chairperson of
ministry under Government launched by the hon’ble Union Union Government in June the Council.
of India which was formed in minister for Jal Shakti Shri 2018. 20. (d) All of the above
May 2019 under the second Gajendra Singh Shekhawat Namami Gange Programme The National Ganga
Modi ministry. This was and other dignitaries on is an Integrated Conservation Council is formed under the
formed by merging of two the inaugural day of ‘Ganga Mission, approved as a Environment (Protection) Act
ministries; Ministry of Water Utsav – the River Festival ‘Flagship Programme’ by (EPA),1986. National Mission
Resources, River Development 2021’. CLAP is an initiative the Union Government in for Clean Ganga(NMCG) acts
& Ganga Rejuvenation and by Namami Gange, created June 2014 to accomplish the as an implementation arm of
Ministry of Drinking Water and executed by TREE Craze twin objectives of effective the National Ganga Council.
and Sanitation. Minister Foundation under leadership abatement of pollution and It has a two-tier management
responsible: Gajendra Singh of Ms Bhawna Badola, CEO conservation and rejuvenation structure and comprises
Shekhawat. of TREE Craze Foundation. of National River Ganga. of Governing Council and
17. (a) Continuous Learning and CLAP is also funded and 19. (a) Prime Minister Executive Committee.
Activity Portal supported by World Bank.
Bhopal +91-7676564400 Indore +91-9589613810, +91-731-4987379 Prayagraj +91-8114000926, +91-8114000927 Kanpur +91-8576066660, +91-8707605589 E: support@toprankers.com
Gurugram +91-8448444207 Delhi +91-9810610466 Ranchi +91-9334969993 Lucknow +91-6390576666 Jabalpur +91-7004386936 W: www.toprankers.com
6 The CLAT Post • November 2021

Passage (Q.21-Q.25):

F ifteen years after the


Supreme Court had issued
directions for police reforms,
the National Human Rights
Commission (NHRC) has
asked the Union Ministry of
Home Affairs and the State
Governments to set up Police
Complaints Authorities as per
the judgment in [1].
The NHRC’s core advisory
group on criminal justice
system reforms has said there
is an “immediate need to set up
police complaints authorities
at the State/UT and district
level” as per the Supreme
Court’s directives, according by the police and the burden of (a) 1958 (b) 1977 the Paris Principles of
to the minutes of its meeting proof to explain the injury lies Human Rights.
(c) 1986 (d) 2000
published on Tuesday. The core on the authority concerned.
group, which met on August 23. Which of the following 24. Which of the following will
18, recommended to the 21. Which of the following will is NOT true about the replace [2] in the above
MHA and the States that the replace [1] in the above National Human Rights passage?
status of compliance should be passage? Commission? (a) 100th (b) 110th
displayed on the websites of the (a) Prakash Singh vs. Union (a) The National Human (c) 112th (d) 113th
Ministry and the State Home of India, 2006
Departments. Rights Commission was
(b) Gian Kaur vs. State of established in 1993. 25. Who can be a chairman of
The NHRC group also said the Punjab, 1994 National Human Rights
MHA and the Law Ministry (b) It is an independent
Commission?
should consider implementing (c) AK Gopalan vs. State of statutory body as per
Madras, 1950 the provisions of the (a) Retired Supreme Court
the recommendations of the [2]
Protection of Human Judge / CJI
report of the Law Commission (d) Vandana vs. State of UP,
to add Section 114B to the 2016 Rights Act of 1993. (b) Retired High Court
Indian Evidence Act. This (c) Protection of Human Judge
would ensure that in case a 22. Malimath Committee on Rights Act was amended (c) Retired Speaker of Lok
person sustains injuries in Reforms in the Criminal in 2008. Sabha
police custody, it is presumed Justice System was set up
(d) NHRC was established (d) Either a or b
that the injuries were inflicted in which of the following
in conformity with
years?

21. (a) Prakash Singh vs. Union of India was submitted to Deputy MHA and the Law Ministry Two Members - Candidates
India, 2006 Prime Minister L.K. Advani, should consider implementing with the knowledge or practical
The Supreme Court in the who was also in charge of the the recommendations of experience in the matters of
landmark Prakash Singh case Home portfolio, in 2003. the 113th report of the Law Human Rights
(2006) gave seven directives 23. (c) Protection of Human Commission to add Section Deemed Members (Ex-officio
where considerable work in Rights Act was amended in 114 B to the Indian Evidence Members) - Deemed members
police reforms is still needed. 2008. Act. are chairpersons of the below
22. (d) 2000 The National Human Rights 25. (a) Retired Supreme Court national commissions:
Malimath Committee was Commission (NHRC) Judge / CJI National Commission for
headed by Justice V.S. established in 1993, is an Composition of NHRC Minorities
Malimath, former Chief Justice independent statutory body Chairman of NHRC - National Commission for
of the Karnataka and Kerala as per the provisions of the Retired Chief Justice of India Scheduled Castes
High Courts. This Committee Protection of Human Rights Member 1 - Retired Judge National Commission for
began its work in 2000 when it Act of 1993 which was of Supreme Court of India Scheduled Tribes
was constituted by the Home amended in 2006. Member 2 - One who is/has National Commission for
Ministry. The Report related to 24. (d) 113th been a Chief Justice of a High Women
the criminal justice system in The NHRC group also said the Court
Bhopal +91-7676564400 Indore +91-9589613810, +91-731-4987379 Prayagraj +91-8114000926, +91-8114000927 Kanpur +91-8576066660, +91-8707605589 E: support@toprankers.com
Gurugram +91-8448444207 Delhi +91-9810610466 Ranchi +91-9334969993 Lucknow +91-6390576666 Jabalpur +91-7004386936 W: www.toprankers.com
7 The CLAT Post • November 2021

(a) Part XV of the


Indian constitution
deals with elections,
and establishes a
commission for these
matters.
(b) Section 324- Section
329 form the part of
Part XV.
(c) Until 1959, the
commission was a single
member body, but later
two additional Election
Commissioners were
added.
(d) The Model Code of
Conduct is operational
from the date on
which the election
schedule is announced
Passage (Q.26-Q.30): (MBC) in education and public (c) The law violated the until the date of result

T he Madras High Court employment in Tamil Nadu was Supreme Court’s ruling announcement.
on Monday set aside the earmarked for the Vanniyakula on reservation.
Kshatriya community. 30. The Central Government
provision for [1] per cent internal (d) None of the above introduced which of the
reservation for Vanniyakula
26. Which of the following will 28. The origins of the Model following Constitutional
Kshatriyas in education and
replace [1] in the above Code of Conduct for Amendment to provide
public employment in Tamil
passage? Elections is found in the the reservation for the
Nadu.
(a) 7% (b) 10% assembly elections of which Economically Weaker
The bench comprising Justice Section?
M Duraisamy and Justice (c) 10.5% (d) 13.5% of the following States?
Murali Shankar said the law, (a) Maharashtra (a) 100th Constitution
which was announced by the 27. The Law was challenged on (Amendment) Act, 2019
the basis of which of the (b) Kerala
previous AIADMK government (b) 103rd Constitution
following? (c) Haryana (Amendment) Act, 2019
and implemented by the
current DMK government, was (a) The law was passed with (d) Tamil Nadu (c) 106th Constitution
‘unconstitutional’. political motivation. (Amendment) Act, 2019
29. Consider the following
As per the law, [1] per cent (b) The law was passed about the Elections in India (d) 110th Constitution
of the 20 per cent reservation in the violation to the and choose the incorrect? (Amendment) Act, 2019
for Most Backward Classes constitution of India.

26. (c) 10.5% been provided with a “political internal reservation for the IRS officers. Until 1989, the
The Madras High Court on motive”. community. commission was a single
Monday set aside the provision The announcement for 28. (b) Kerala member body, but later
for 10.5 per cent internal providing 10.5 per cent The origins of the MCC lie two additional Election
reservation for Vanniyakula international reservations for in the Assembly elections of Commissioners were added.
Kshatriyas in education and the Vanniyakula Kshatriya Kerala in 1960, when the State 30. (b) 103rd Constitution
public employment in Tamil community was made only administration prepared a (Amendment) Act, 2019
Nadu. moments before the model ‘Code of Conduct’ for political The 10% EWS quota was
27.(a) The law was passed with code of conduct for the actors. introduced under the 103rd
political motivation. Assembly election earlier this 29. (c) Until 1959, the commission Constitution (Amendment)
A set of writ petitions were year came into force. The was a single member body, but Act, 2019 by amending
filed challenging the law. In AIADMK government made later two additional Election Articles 15 and 16.
several southern districts of the announcement as per the Commissioners were added. It inserted Article 15 (6) and
the state, agitations were also demand of its allied party, The Election Commissioners Article 16 (6).
held and people claimed that Pattali Makkal Katchi (PMK), are usually retired IAS or
the internal reservation had which sought 20 per cent
Bhopal +91-7676564400 Indore +91-9589613810, +91-731-4987379 Prayagraj +91-8114000926, +91-8114000927 Kanpur +91-8576066660, +91-8707605589 E: support@toprankers.com
Gurugram +91-8448444207 Delhi +91-9810610466 Ranchi +91-9334969993 Lucknow +91-6390576666 Jabalpur +91-7004386936 W: www.toprankers.com
8 The CLAT Post • November 2021
Passage (Q.31-Q.35):

T he central government is
preparing to give a fresh
push to the establishment of an
All India Judicial Service (AIJS)
on the lines of the central civil
services. Law Minister [1] has
called a meeting of state law
ministers in the last week of this
month to discuss the AIJS and
issues related to infrastructure
in the lower judiciary. The idea
of centralised recruitment of
judges has been debated in legal
circles for decades, and remains
contentious.
The AIJS is a reform push to
centralise the recruitment of
judges at the level of additional
district judges and district
judges for all states.
In the same way that the Union
Public Service Commission
conducts a central recruitment
process and assigns successful introduced in which of the (a) Article 224 and the States.
candidates to cadres, judges following years? (b) Article 226 (b) Under Article 312,
of the lower judiciary are (a) 1958 (b) 1976 (c) Article 230 Rajya Sabha is required
proposed to be recruited (c) 1998 (d) 2006 (d) Article 233 to pass a resolution
centrally and assigned to states. supported by not less
33.Who introduced the first 35. Which of the following is
than two-thirds of its
31. Which of the following will judicial reforms in India TRUE?
members present and
replace [1] in the above during the British period? (a) In 1976, the 42nd voting. Thereafter,
passage? (a) Lord Minto Constitutional Parliament has to enact
(a) Kiren Rijiju (b) Lord Dalhousie Amendment amended a law creating the AIJS.
(b) Ravi Shankar Prasad (c) Lord Macauley Article 312 (1), allowing
(c) Prakash Javadekar (d) Lord Cornwallis (c) This means no
Parliament to enact
(d) Smiriti Irani constitutional
legislation establishing
34.Which article of the amendment will
one or more All-India
32. The first proposal with Constitution of India deal be required for
Services, including an
regards to the All India with the appointment of establishment of AIJS.
AIJS, that would be
Judicial Services was district judges? (d) All of the above
shared by the Union

31. (a) Kiren Rijiju the country to make it speedy 1793 respectively. allowing Parliament to enact
Law Minister Kiren Rijiju and less expensive. The AIJS 34. (d) Article 233 legislation establishing one
has called a meeting of state was first proposed by the 14th Article 233 Constitution or more All-India Services,
law ministers in the last week report of the Law Commission (1) Appointments of persons including an AIJS, that would
of this month to discuss the in 1958. to be, and the posting and be shared by the Union and
AIJS and issues related to 33. (d) Lord Cornwallis promotion of, district judges the States. Under Article 312,
infrastructure in the lower Lord Cornwallis was Governor in any State shall be made by Rajya Sabha is required to
judiciary. General from the year 1786 to the Governor of the State in pass a resolution supported by
32. (a) 1958 1793 and his most noted work consultation with the High not less than two-thirds of its
The 14th Law Commission was in the field of criminal Court exercising jurisdiction members present and voting.
of India Report on Reform of judicature. He introduced in relation to such State. Thereafter, Parliament has to
Judicial Administration (1958) changes in the judicial system 35. (d) All of the above enact a law creating the AIJS.
This Law Commission Report in three years – 1787, 1790 and In 1976, the 42nd This means no constitutional
engaged on how to improve 1793. These were known as Constitutional Amendment amendment will be required
the judicial administration in Judicial Plan of 1787, 1790 and amended Article 312 (1), for establishment of AIJS.
Bhopal +91-7676564400 Indore +91-9589613810, +91-731-4987379 Prayagraj +91-8114000926, +91-8114000927 Kanpur +91-8576066660, +91-8707605589 E: support@toprankers.com
Gurugram +91-8448444207 Delhi +91-9810610466 Ranchi +91-9334969993 Lucknow +91-6390576666 Jabalpur +91-7004386936 W: www.toprankers.com
9 The CLAT Post • November 2021

Passage (Q.36-Q.40):
China delivered its largest and
most advanced warship to
Pakistan on Monday. The Type
054A/P frigate, named [1], has
been designed and built by [2].
On receiving the ship, Pakistan’s
ambassador to China, Moin Ul-
Haq, said in a navy news release
that the frigate will strengthen
the service’s ability to respond
to maritime challenges, ensure
seaward defence, and maintain
peace, stability and the regional
balance of power.
The ship is technologically
advanced and is equipped with a
new combat management system
developed by the CSSC.
It also has advanced sensors and
weapons; according to a report, with the Pakistan Air Indian Ocean and the (a) China State
the frigate is equipped with the Force to build JF-17 Arabian Sea. Shipbuilding
SR2410C long-range and Type
Thunder fighter aircraft. (d) All of the above Corporation Limited
517/SUR17B air-surveillance
radars. (b) China has acquired (b) National Shipbuilding
Pakistan’s Gwadar 38. India has raised a new Corporation of China
36. Which of the following will port in the Arabian army aviation brigade in
(c) United Navy of
replace [1] in the above Sea which connects the eastern sector of Line
Republic of China
passage? with China’s Xinjiang of Actual Control (LAC)
in which of the following (d) State Defence Arms
(a) PNS Tughril province by land in the
States? Corporation
USD 60 billion China
(b) PNS Zulfiquar
Pakistan Economic (a) Sikkim 40. The Concept of Line Actual
(c) PNS Shamsheer Corridor (CPEC). (b) Arunachal Pradesh Control came in a bilateral
(d) PNS Saif (c) The modernisation agreement in which of the
(c) Himachal Pradesh
of the Pakistan Navy following years?
37. Which of the following is (d) Uttarakhand
India’s concern with the coupled with the (a) 1961 (b) 1965
growing closeness between acquisition of the naval 39. Which of the following will (c) 1989 (d) 1993
China and Pakistan? bases was expected to replace [2] in the above
shore up the Chinese passage?
(a) China also partners Navy’s presence in the

36. (a) PNS Tughril the Pakistan Air Force to build Pakistan Navy coupled with
Designed and built by China JF-17 Thunder fighter aircraft. the acquisition of the naval The Type 054A/P frigate,
State Shipbuilding Corporation Besides building its first bases was expected to shore up named PNS Tughril, has been
Limited (CSSC), the frigate military base in Djibouti in the the Chinese Navy’s presence designed and built by China
was delivered to the Pakistan Horn of Africa in the Indian in the Indian Ocean and the State Shipbuilding Corporation
Navy in a commissioning Ocean, China has acquired Arabian Sea. Limited (CSSC).
ceremony in Shanghai, Pakistan’s Gwadar port in the 38. (b) Arunachal Pradesh 40. (d) 1993
according to Global Times. Arabian Sea which connects The Indian Army has added LAC stands for Line of Actual
The Type 054A/P frigate with China’s Xinjiang province a major aviation capability to Control. It is a boundary
was named the PNS Tughril, by land in the USD 60 billion the force with a new Aviation between China and India. The
according to a statement the China Pakistan Economic Brigade raised to look after the concept of “Line of Actual
Pakistan Navy sent to the Corridor (CPEC). troops posted along the Line of Control” (LAC) came in a
Global Times on Monday. China is also developing Sri Actual Control with Arunachal bilateral agreement in 1993,
37. (d) All of the above Lanka’s Hambantota port after Pradesh. although there was no concrete
Besides the advanced naval it acquired it on 99 years’ lease. 39. (a) China State Shipbuilding settlement on ground positions
ships, China also partners with The modernisation of the Corporation Limited between these two countries.
Bhopal +91-7676564400 Indore +91-9589613810, +91-731-4987379 Prayagraj +91-8114000926, +91-8114000927 Kanpur +91-8576066660, +91-8707605589 E: support@toprankers.com
Gurugram +91-8448444207 Delhi +91-9810610466 Ranchi +91-9334969993 Lucknow +91-6390576666 Jabalpur +91-7004386936 W: www.toprankers.com
10 The CLAT Post • November 2021

Passage (Q.41-Q.45):
Iran has increased its stockpile
of [1] enriched uranium to
25 kilograms (55 pounds),
state media reported on
Friday, potentially adding to
complications dogging efforts to
revive Tehran’s 2015 nuclear deal
with world powers.
Negotiations are due to resume
on Nov. 29 in Vienna, But
Western powers have said
Iran’s accelerating enrichment
of uranium closer to weapons
grade, breaching limits set by
the pact after Washington under
then-President Donald Trump 42. Which of the following is reactor fuel and nuclear as weapons-grade
withdrew from the pact is
NOT true with regards to weapons. uranium.
dimming chances of salvaging it.
Joint Comprehensive Plan (d) In May 2020, the USA (d) Highly enriched
“So far we have produced 25 of Action? abandoned the deal uranium has a purity of
kilograms of [1] uranium, which,
(a) In 2015, Iran with the criticising it as flawed 20% or more and is used
except for countries with nuclear
P5+1 group of world and reinstated and in research reactors.
weapons, no other country is
able to produce,” Iranian media powers - the USA, UK, tightened its sanctions.
France, China, Russia, 44. Which of the following
quoted Behrouz Kamalvandi, 43. Which of the following is NOT a party to the
spokesman for the Atomic and Germany agreed on
a long-term deal on its NOT TRUE about meetings which are
Energy Organisation of Iran, as Uranium Enrichment? scheduled to be held in
saying. nuclear programme.
(a) U-235 is a fissile Vienna?
Iran has in the past denied (b) Under the deal, Iran
agreed to curb its material that can sustain (a) USA
seeking nuclear weapons, saying
nuclear activity in a chain reaction in a (b) European Union
it is refining uranium only for
return for the lifting of nuclear reactor.
civilian energy uses, and has said (c) China
its breaches are reversible if the sanctions and access to (b) Enrichment process
(d) Iran
United States lifts sanctions and global trade. increases the proportion
rejoins the agreement. (c) The agreement allowed of U-235 through the 45. International Atomic
Iran to accumulate process of isotope Energy Agency was
41. Which of the following will small amounts of separation established in which of the
replace [1] in the above uranium for research (c) For nuclear weapons, following years?
passage? but it banned the enrichment is required (a) 1945 (b) 1957
(a) 25% (b) 40% enrichment of uranium, upto 50% or more (c) 1967 (d) 1975
(c) 50% (d) 60% which is used to make which is known

41. (d) 60% world powers. accord. to resuscitate the deal would
The total amount now 42. (d) In May 2020, the USA 43. (c) For nuclear weapons, resume on 29th November
includes 113.8 kg enriched abandoned the deal criticising enrichment is required upto 2021 in Vienna.
to 20 percent, up from 84.3 it as flawed and reinstated and 50% or more which is known as 45. (b) 1957
kg in September, and 17.7 kg tightened its sanctions. weapons-grade uranium. IAEA’s origins can be traced
enriched up to 60 percent, up In May 2018, the USA For nuclear weapons, back to an address of the
from 10 kg, the report said. abandoned the deal criticising enrichment is required upto former US President Dwight
The document will be it as flawed and reinstated and 90% or more which is known Eisenhower to the General
discussed during the IAEA’s tightened its sanctions. as weapons-grade uranium. Assembly of the United
Board of Governors, scheduled The USA held that it would 44. (d) Iran Nations in 1953.
for next week with diplomats attempt to force all countries After months of delays, The address was known as
preparing to restart talks in to stop buying Iranian oil the European Union, Iran ‘Atoms for Peace’ and this
Vienna on November 29 over and put pressure on Iran and the US have recently was the organisation’s first
reviving the moribund 2015 to negotiate a new nuclear announced that indirect talks name when it was formally
nuclear deal between Iran and established in 1957.
Bhopal +91-7676564400 Indore +91-9589613810, +91-731-4987379 Prayagraj +91-8114000926, +91-8114000927 Kanpur +91-8576066660, +91-8707605589 E: support@toprankers.com
Gurugram +91-8448444207 Delhi +91-9810610466 Ranchi +91-9334969993 Lucknow +91-6390576666 Jabalpur +91-7004386936 W: www.toprankers.com
11 The CLAT Post • November 2021

Passage (Q.46-Q.50): The Indian embassy said France 47. France signed a “strategic Alliance in which of the
India and France agreed to reiterated its commitment to partnership” with India for following year?
strengthen defence and security fully support Prime Minister the first time after the end (a) 2014 (b) 2015
partnership by enhancing Narendra Modi’s vision of of the Cold War in which
(c) 2017 (d) 2018
intelligence and information “Atmanirbhar Bharat (self- of the following years?
sharing, bolstering capabilities, reliant India)” and defence (a) 1990 (b) 1992 50. The Indian Air Force
expanding military drills and industrialisation, joint research (IAF) has inducted French
and technology development (c) 1996 (d) 1998
pursuing new initiatives in Rafale multi-role combat
maritime, space and cyber in India across a wide range of 48. Which of the following is aircraft in its fleet. Which
domains, the Indian embassy in advanced capabilities. NOT one of the Military of the following companies
Paris said on Saturday. Exercises between France manufacture Rafale?
46. Which of the following will
The two countries resolved to replace [1] in the above and India? (a) Gulfstream
expand the defence ties at a passage? (a) Exercise Garuda Aerospace
meeting of India-France strategic
(a) Ajit Doval (b) Exercise Shakti (b) Airbus
dialogue that was co-chaired by
India’s National Security Advisor (b) D Padsalgikar (c) Exercise Yudh Abhiyas (c) Dassault Aviation
[1] and Emmanuel Bonne, the (c) Pankaj Saran (d) Exercise Varuna (d) Safran
Diplomatic Advisor to French (d) Rajinder Khanna
President Emmanuel Macron, in 49. India and France initiated
Paris on Friday. the International Solar

46. (a) Ajit Doval countries with which India cooperation endeavor between session of the United Nations
The National Security Advisor signed a “strategic partnership” India and the US. The exercise Climate Change Conference of
is the senior official on the after the end of the Cold War, was started in 2004 under the the Parties (COP-21) in Paris.
National Security Council of in January 1998. US Army Pacific Partnership 50. (c) Dassault Aviation
India, and the chief adviser to France was one of the very Program. The exercise is Introduced in 2001, the Rafale
the Prime Minister of India few countries to support hosted alternately between is a French twin-engine and
on national security policy India’s decision to test nuclear both countries. multirole fighter aircraft
and international affairs. Ajit weapons in 1998. 49. (b) 2015 designed and built by Dassault
Doval is the current NSA, and 48. (c) Exercise Yudh The International Solar Aviation being produced for
has the same rank as a Union Abhiyas Alliance (ISA) was launched by both the French Air Force and
Cabinet Minister. Exercise Yudh Abhiyas is PM Modi and former French for carrier-based operations in
47. (d) 1998 the largest running joint president Francois Hollande the French Navy.
France was one of the first military training and defence in November 2015 at the 21st
Bhopal +91-7676564400 Indore +91-9589613810, +91-731-4987379 Prayagraj +91-8114000926, +91-8114000927 Kanpur +91-8576066660, +91-8707605589 E: support@toprankers.com
Gurugram +91-8448444207 Delhi +91-9810610466 Ranchi +91-9334969993 Lucknow +91-6390576666 Jabalpur +91-7004386936 W: www.toprankers.com
12 The CLAT Post • November 2021

Passage (Q.51-Q.55): statements, pushed for collective of the country from (d) Law and Justice
Top security officials of eight efforts to cope with threats such 1996 to 2001 and was Minister
countries, including India, Iran as terrorism and drug trafficking notorious for their strict
emanating from Afghanistan implementation of the 54. The National Security
and Russia, on Wednesday called
for closer coordination and and to provide humanitarian aid Sharia or Islamic law Council was established
collective efforts to tackle threats to the Afghan people before the there. under which of the
such as terrorism emanating onset of winter. following Prime Ministers
(b) The Taliban officially
from Afghanistan and to address 51. The Regional Security of India?
refers to itself as the
a looming humanitarian crisis. dialogue on Afghanistan ‘Islamic Emirate of (a) PV Narsimha Rao
In his opening remarks at the was initiated on behalf of Afghanistan’. (b) Atal Bihari Vajpayee
Delhi Regional Security Dialogue which of the following? (c) The current head of the (c) Manmohan Singh
on Afghanistan being hosted by (a) Ministry of Defence Taliban is Mullah Omar. (d) Narendra Modi
India, National Security Adviser (b) National Security (d) Taliban was formed in
(NSA) Ajit Doval said the time Council Secretariat 1994. 55. The United Nations
has come for closer consultations
(c) Research Wing and Security Council under the
and greater coordination among 53. Who among the following
Analysis presidency of India passed
regional countries to find is NOT a member of which of the following
solutions to recent developments (d) Intelligence Bureau National Security Council resolution against Taliban?
in the war-torn country. Secretariat?
52. Which of the following (a) Resolution 2593
The NSAs and security council (a) Home Minister
is NOT TRUE about the (b) Resolution 2693
secretaries of Iran, Russia,
Taliban? (b) Defence Minister
Kazakhstan, Kyrgyzstan, (c) Resolution 2683
Tajikistan, Turkmenistan and (a) The Taliban controlled (c) Finance Minister (d) Resolution 2493
Uzbekistan, in their brief opening almost three-quarters

51. (b) National Security Council China. Finance are its members and Minister.
Secretariat 52. (c) The current head of the the National Security Adviser 55. (a) Resolution 2593
India’s top security Taliban is Mullah Omar. acts as its Secretary. The India-led United Nations
establishment, the National The current head of the 54. (b) Atal Bihari Vajpayee Security Council (UNSC)
Security Council Secretariat, Taliban is Hibatullah The NSC was established in adopted a Resolution 2593
has taken the lead in Akhundzada Mullah Omar is 1998 by the government of on Taliban. The resolution,
organising the in-person regarded as the founder of the AB Vajpayee. Brajesh Mishra sponsored by France, UK and
meeting. Taliban. He died in 2013. served as the country’s first the US, was adopted with 13
Invitations were sent to 53. (d) Law and Justice National Security Advisor members, including India,
Afghanistan’s neighbours such Minister (NSA). Before the NSC was voting in favour, none against
as Pakistan, Iran, Tajikistan, The Ministers of Home Affairs, formed, these functions were it. Two permanent and veto-
and Uzbekistan, and other key Defence, External Affairs and carried out by the Principal wielding members Russia and
players including Russia, and Secretary to the Prime China abstained.
Bhopal +91-7676564400 Indore +91-9589613810, +91-731-4987379 Prayagraj +91-8114000926, +91-8114000927 Kanpur +91-8576066660, +91-8707605589 E: support@toprankers.com
Gurugram +91-8448444207 Delhi +91-9810610466 Ranchi +91-9334969993 Lucknow +91-6390576666 Jabalpur +91-7004386936 W: www.toprankers.com
13 The CLAT Post • November 2021

of drug policy.
(c) Standards and
expectations from civil
society experts on drug
policy implementation
remain same from
country to country.
(d) Drug policies
disproportionately affect
people marginalised
on the basis of their
gender, ethnicity, sexual
orientation and socio-
economic status.

58. The Narcotic Drugs and


Psychotropic Substances
Act was passed in India
in which of the following
year?
(a) 1975 (b) 1978
(c) 1985 (d) 1996

59. Which of the following will


replace [2] in the above
passage?
Passage (Q.56-Q.60): countries which is assessed as a (a) 15 (b) 16 (a) Concern Worldwide
A first of its kind global drug “drug policy failure and a sign for (c) 17 (d) 18 (b) Amnesty International
policy index 2021 that ranks 30 urgent measures to deal with the
(c) Harm Reduction
countries drawn from across situation. 57. Which of the following is
Consortium
regions on how humane and 48 out of 100 is a drug policy NOT true with regards to
the findings? (d) International
health-driven their drug laws and fail in anyone’s book. This index
Consortium on Drug
policies are, gives India a score highlights the huge room for (a) Inequality is deeply
Policy
of 46 out of 100 with Norway at improvement across the board,” seated in global drug
the top with 74 and Brazil at the said Ann Fordhamn, executive policies, with the top- 60. United Nations Office on
bottom scoring 26. In terms of director of the International ranking 5 countries Drugs and Crime was
ranking India is at [1]. Drug Policy Consortium which scoring 3 times as much established in which of the
The index highlights that the led the development of the index as the lowest-ranking 5 following year?
global dominance of drug with the partners in the [2]. countries.
policies based on repression and (a) 1994 (b) 1997
56. Which of the following will (b) The Index provides an
punishment has led to low scores (c) 1998 (d) 2002
replace [1] in the above essential accountability
overall, with a median score and evaluation
passage?
of just 48 out of 100 for all 30 mechanism in the field

56. (d) 18 country. that prohibits a person the the Global Drug Policy Index
India ranked 18th out of 30 Standards and expectations production/manufacturing/ which was released by the
countries in the 1st edition of from civil society experts on cultivation, possession, sale, Harm Reduction Consortium
the Global Drug Policy Index drug policy implementation purchasing, transport, storage, in November 2021.
which was released by the vary from country to country. and/or consumption of any 60. (b) 1997
Harm Reduction Consortium 58. (c) 1985 narcotic drug or psychotropic It was established in 1997 and
in November 2021. The Narcotic Drugs and substance. was named as a United Nations
57. (c) Standards and expectations Psychotropic Substances Act, 59. (c) Harm Reduction Office on Drugs and Crime
from civil society experts on 1985, commonly referred to Consortium (UNODC) in 2002.
drug policy implementation as the NDPS Act, is an Act India ranked 18th out of 30
remain same from country to of the Parliament of India countries in the 1st edition of
Bhopal +91-7676564400 Indore +91-9589613810, +91-731-4987379 Prayagraj +91-8114000926, +91-8114000927 Kanpur +91-8576066660, +91-8707605589 E: support@toprankers.com
Gurugram +91-8448444207 Delhi +91-9810610466 Ranchi +91-9334969993 Lucknow +91-6390576666 Jabalpur +91-7004386936 W: www.toprankers.com
14 The CLAT Post • November 2021

Passage (Q.61-Q.65): Amit Shah on October 23 replace [1] in the above does the ICAO recognises?
On November 2, the recently while he was on a visit to the passage? (a) three (b) Four
launched flight from Srinagar Union Territory of Jammu and (a) First (b) Third (c) Five (d) Six
to Sharjah had to fly an Kashmir. The flight is being
(c) Fourth (d) Sixth
additional 40 minutes over operated by budget airline 64. Which of the following will
its scheduled route of 3 hours [2]. Since then, the flight had 62. Which of the following replace [2] in the above
and 45 minutes for the first been taking a 3-hour-45- is NOT true about the passage?
time. The change of route was minute route via the airspace International Civil Aviation
of Pakistan, before going (a) Indigo (b) GoFirst
due to Pakistan denying the Organisation?
permission to use its airspace southwest to reach UAE’s (c) Air India (d) Vistara
Sharjah. However, all of that (a) ICAO is a United
for this flight, which means that 65. Freedom of Air rules
stopped on November 2, when Nations (UN)
the flight had to take a longer emanates from which of
the country refused to allow specialized agency
route to avoid the neighbouring
the flight to enter Pakistan’s (b) India is among its 193 the following Conventions?
country’s airspace. A report in
IE has cited experts as saying territory. This led to the flight members. (a) Brussels
that this refusal could mean the having to fly south over the (c) It is headquartered in Convention
violation of the [1] freedom of states of Rajasthan and Gujarat Montreal, Canada. (b) Chicago Convention
air by Pakistan. before flying out over the
(d) It was established in (d) Geneva
Arabian Sea and reaching the
The direct flight from Srinagar 1945. Convention
UAE via Omani airspace.
to Sharjah had been launched (d) Montreal Convention
by Union Home Minister 61. Which of the following will 63. How many Freedoms of Air

61. (a) First in 1947, which laid the Organisation (ICAO). 65. (b) Chicago Convention
Experts say Pakistan’s refusal foundation for the standards 64. (b) GoFirst Freedom of air rule emanates
to allow GoFirst to use its and procedures for peaceful On October 23, Home from the Chicago Convention
airspace during the Srinagar global air navigation. Minister Amit Shah, while on in 1944. The signatories to
Sharjah flight is a violation of 63. (c) Five a visit to Jammu & Kashmir, the convention decided to
the first freedom of air. The convention provides Nine launched a direct flight set rules that would act as
62. (d) It was established in freedoms of air, but only the between Srinagar and Sharjah fundamental building blocks
1945. first five freedoms have been operated by budget airline to international commercial
It is a United Nations (UN) officially recognized by the GoFirst (formerly known as aviation.
specialized agency, established International Civil Aviation GoAir).
Bhopal +91-7676564400 Indore +91-9589613810, +91-731-4987379 Prayagraj +91-8114000926, +91-8114000927 Kanpur +91-8576066660, +91-8707605589 E: support@toprankers.com
Gurugram +91-8448444207 Delhi +91-9810610466 Ranchi +91-9334969993 Lucknow +91-6390576666 Jabalpur +91-7004386936 W: www.toprankers.com
15 The CLAT Post • November 2021

Passage (Q.66-Q.70): particular year.


On 8 November, President (d) Bharat Ratna was
Ram Nath Kovind conferred instituted in 1954
the prestigious civilian awards
in New Delhi to the 2020 and 69. Which of the following is
2021 recipients. The Padma TRUE about the Padma
Vibhushan, Padma Bhushan, Awards?
and Padma Shri are the highest (a) The Padma Awards are
civilian awards – after the announced annually
Bharat Ratna – recognising on Republic Day (26th
the contributions to various January).
fields like social work, science,
(b) Government servants
medicine, literature, sports, and
including those working
art.
with PSUs, except
The list of winners has doctors and scientists,
made headlines all week, are not eligible for these
signalling that it’s no longer Awards.
an award reserved for the
elite. The inspiring stories (c) A higher category of
of the winners — from the to the improvement of society 68. Which of the following is Padma award can be
barefoot environmentalist at grassroots levels.” NOT true about Bharat conferred on a person
from Karnataka Tulasi Gowda Ratna? only where a period
and transgender folk artist 66. In which of the following of at least five years
(a) It is awarded in has elapsed since
Manjamma Jogati to “elephant years was the Padma
recognition of conferment of the
man” Dr Kushal Konwar Awards established?
exceptional service/ earlier Padma award.
Sarma and orange seller (a) 1952 (b) 1954 performance of the
and school builder Harekala (c) 1958 (d) 1962 (d) All of the above
highest order in
Hajabba — prove that the
any field of human 70. Who among the following
awards truly honour significant 67. The Padma Awards endeavour. was NOT a recipient of
achievements to India’s heritage committee is constituted by
and development. who among the following (b) The recommendations the first Padma Vibhushan
every year? for Bharat Ratna are Award?
Even industrialist Anand
Mahindra, who would have made by the Prime (a) Balasaheb Gangadhar
(a) Prime Minister Minister to the
been considered a typical Kher
(b) President President of India
recipient of the award five (b) V. K. Krishna Menon
years ago, tweeted that he felt (c) Speaker of Lok (c) The number of
Sabha (c) Jigme Dorjic Zakir
“undeserving” of the award in Bharat Ratna Awards
Husain
comparison to the individuals (d) Home Minister is restricted to a
“making seminal contributions maximum of five in a (d) S. Radhakrishnan

66. (b) 1954 Secretary, Secretary to the Bharat Ratna & Padma awards five years has elapsed since
Padma Awards, which were President and four to six in 1954. The latter had three conferment of the earlier
instituted in the year 1954, eminent persons as members. classes namely Pahela Varg, Padma award. All persons
is announced every year on The recommendations of the Dusra Varg and Tisra Varg. without distinction of race,
the occasion of Republic Day committee are submitted to These were subsequently occupation, position or sex
except for brief interruption(s) the Prime Minister and the renamed as Padma Vibhushan, are eligible for these awards.
during the years 1978 and 1979 President of India for approval. Padma Bhushan and Padma However, Government
and 1993 to 1997. 68. (c) The number of Bharat Shri vide Presidential servants including those
67. (a) Prime Minister Ratna Awards is restricted to a Notification issued on January working with PSUs, except
All nominations received maximum of five in a particular 8, 1955. doctors and scientists, are not
for Padma Awards are year. The Padma Awards are eligible for these Awards.
placed before the Padma The number of Bharat announced annually on 70. (d) S. Radhakrishnan
Awards Committee, which Ratna Awards is restricted Republic Day (26th January). C. Rajagopalachari, S.
is constituted by the Prime to a maximum of three in a It is one of the highest civilian Radhakrishnan, C. V. Raman
Minister every year. The particular year. honours of India. A higher were the First recipient of the
Padma Awards Committee 69. (d) All of the above category of Padma award can Bharat Ratna Awards in 1954
is headed by the Cabinet The Government of India be conferred on a person only
Secretary and includes Home instituted two civilian awards- where a period of at least
Bhopal +91-7676564400 Indore +91-9589613810, +91-731-4987379 Prayagraj +91-8114000926, +91-8114000927 Kanpur +91-8576066660, +91-8707605589 E: support@toprankers.com
Gurugram +91-8448444207 Delhi +91-9810610466 Ranchi +91-9334969993 Lucknow +91-6390576666 Jabalpur +91-7004386936 W: www.toprankers.com
16 The CLAT Post • November 2021

Passage (Q.71-Q.75): transparency; and capacity for 72. Which of the following (a) Denmark
India has slipped five points in civil society oversight, including NOT one of the factors for (b) Australia
the global bribery risk rankings the media role. calculating the Score?
(c) Japan
settling at the [1] spot among Contrastingly, the world’s most (a) Enforcement and anti-
(d) North Korea
194 nations analysed for corruption-free country with bribery deterrence.
compliance risks businesses face the least business bribery risk (b) Business interactions 74. TRACE International was
internationally. has a score of 1 on the domain with the government. founded in which of the
The TRACE Bribery Risk Matrix of business interaction with the following years?
government; 1 on anti-bribery (c) Bribery in the
(TRACE Matrix) published on (a) 1996 (b) 1999
deterrence, 6 on civil service aristocracy class.
Wednesday has been measuring
business bribery risk since [2] transparency and 1 on civil and (d) Capacity for civil (c) 2001 (d) 2008
and assigns an overall country media oversight. society oversight which
includes the media’s 75. Which of the following will
risk score based on the combined
71. Which of the following will role. replace [2] in the above
and weighted score of four
replace [1] in the above passage?
domains — business interactions
passage? 73. Which of the following (a) 2011 (b) 2014
with government; anti-bribery
deterrence and enforcement; (a) 77th (b) 81st Countries posed the lowest (c) 2017 (d) 2020
government and civil service (c) 82nd (d) 85th Commercial Bribery risk?

71. (c) 82nd Enforcement and anti-bribery Venezuela and Eritrea pose the easier and less expensive to
India has slipped five points deterrence. highest commercial bribery reduce the risk of bribery.
in the global bribery risk Business interactions with the risk, while Denmark, Norway, 75. (b) 2014
rankings settling at the 82nd government. Finland, Sweden and New It was originally published
spot among 194 nations Government and civil service Zealand present the lowest. in 2014 to meet a need in
analysed for compliance risks transparency. 74. (c) 2001 the business community for
businesses face internationally. Capacity for civil society TRACE is a non-profit more reliable and nuanced
72. (c) Bribery in the aristocracy oversight which includes the business association dedicated information about the risks
class. media’s role. to anti-bribery, compliance of commercial bribery
Score is calculated on the basis 73. (a) Denmark and good governance. worldwide.
of four factors: North Korea, Turkmenistan, Founded in 2001 to make it
Bhopal +91-7676564400 Indore +91-9589613810, +91-731-4987379 Prayagraj +91-8114000926, +91-8114000927 Kanpur +91-8576066660, +91-8707605589 E: support@toprankers.com
Gurugram +91-8448444207 Delhi +91-9810610466 Ranchi +91-9334969993 Lucknow +91-6390576666 Jabalpur +91-7004386936 W: www.toprankers.com
17 The CLAT Post • November 2021

Passage (Q.76-Q.80): 76. Which of the following will Benjamin Netanyahu on Arabia
In a major step forward, Gulf replace [1] in the above September 15, 2020
passage? 79. Israel has been engaged in
Arab navies and Israeli warships (d) The accords are name
a “shadow war” with which
have conducted their first joint (a) Gulf of Aqaba after the patriarch
of the following Countries?
military exercise in coordination (b) Red Sea Abraham, regards as a
with the United States Naval prophet in both Judaism (a) Iraq
Forces. The five-day drill in (c) Arabian Sea
and Islam. (b) Saudi Arabia
the [1] included naval vessels (d) Caspian Sea
(c) Iran
from the United Arab Emirates 78. Which of the following
(UAE), Bahrain, Israel and the 77. Which of the following statements does not depicts (d) Qatar
US. The maritime drill began statements is not correct the significance of the
about the Abraham 80. Which of the following is an
last Wednesday and engaged Abraham Accords? incorrect statement out of
in several exercises, including Accords?
(a) The Abraham Accords the following?
boarding, search and seizure (a) The Abraham Accords mandated that the
tactics helping to ensure freedom are a joint statement (a) Egypt had signed a
United Arab Emirates peace-deal with Israel in
of navigation aboard the USS made between Israel, and Bahrain each open
Portland transport dock ship, the the United States 1979.
embassies in Israel.
BBC reported. and the United Arab (b) Jordan was the second
(b) The religious Arab-country to have
Aiming to enhance Emirates on August 13,
significance is that it signed a peace-deal with
interoperability between 2020
would allow Muslims Israel.
participating forces’ maritime (b) It also refers to the to visit the Al-Aqsa
interdiction teams, the warships agreement reached (c) India had recognized
Mosque in Jerusalem,
conducted the joint security drill between Israel, Israel as an independent
one of Islam’s holiest
in the virtual shipping route that Bahrain and the UAE country in 1950.
sites.
feeds into the Suez Canal, the to normalise relations
(c) Sudan and Morocco (d) Israel had got its
US Navy said in a statement. between the three.
normalised relations independence in 1949.
However, the 5th fleet did not
(c) The original Abraham with Israel in the same
specify where exactly the drill
was carried out. It is worth the Accords were signed year as the signing of
mention that the [1] connects the by the UAE’s Foreign the accords.
Mediterranean Sea by the Suez Minister Abdullatif
(d) China has shown a
Canal with the Indian Ocean by bin Rashid Al Zayani,
desire to play a larger
the Gulf of Aden, which is one Bahrain’s Foreign
role in the area, and
of the key oil shipping routes, Minister Abdullah bin
it has close ties with
Associated Press reported. Zayed Al Nahyan, and
the UAE, Israel, and,
Israeli Prime Minister
increasingly, Saudi

76. (b) Red Sea joint statement made between Abraham, regards as a prophet and is close to both UAE and
Forces from the United Arab Israel, the United States and in both Judaism and Islam. Israel and, increasingly, Saudi
Emirates, Bahrain, Israel the United Arab Emirates on 78. (d) China has shown a desire Arabia.
and the United States Naval August 13, 2020. It also refers to play a larger role in the area, 79. (c) Iran
Forces Central Command to the agreement reached and it has close ties with the Since February 2021, Iran
(NAVCENT) began a between Israel, Bahrain and UAE, Israel, and, increasingly, and Israel have been accused
multilateral maritime security the UAE to normalise relations Saudi Arabia of engaging in what analysts
operations exercise in the Red between the three. The original Option a, b and c have directly have called a “shadow war”, in
Sea on Wednesday, NAVCENT Abraham Accords were resulted or followed the which vessels linked to each
said on Thursday. signed by the UAE’s Foreign Abraham Accords, however, nation have come under attack
77. (c) The original Abraham Minister Abdullah bin Zayed option d is not a signification in waters around the Gulf in
Accords were signed by the Al Nahyan, Bahrain’s Foreign but merely the intensifying tit-for-tat exchanges.
UAE’s Foreign Minister Minister Abdullatif bin Rashid picture of the geo-politics 80. (d) Israel had got its
Abdullatif bin Rashid Al Al Zayani, and Israeli Prime in the middle east. The US independence in 1949.
Zayani, Bahrain’s Foreign Minister Benjamin Netanyahu is a pre-eminent power in After Israel declared its
Minister Abdullah bin Zayed on September 15, 2020. The the Middle East, but Russia independence on May 14,
Al Nahyan, and Israeli Prime first UAE Ambassador to has made a space for itself, 1948, the fighting intensified
Minister Benjamin Netanyahu Israel, Mohammed Al Khaja spending a lot less money. with other Arab forces
on September 15, 2020 arrived in the country on In recent years, China has joining the Palestinian Arabs
The Abraham Accords are a March 1, 2021 The accords indicated its willingness to in attacking territory in the
are name after the patriarch play a larger role in this region, former Palestinian mandate.
Bhopal +91-7676564400 Indore +91-9589613810, +91-731-4987379 Prayagraj +91-8114000926, +91-8114000927 Kanpur +91-8576066660, +91-8707605589 E: support@toprankers.com
Gurugram +91-8448444207 Delhi +91-9810610466 Ranchi +91-9334969993 Lucknow +91-6390576666 Jabalpur +91-7004386936 W: www.toprankers.com
18 The CLAT Post • November 2021

Passage (Q.81-Q.85):
Prime Minister Narendra Modi
on Tuesday said that while
institutions lose relevance after a
few decades, the Comptroller and
Auditor General (CAG) of India
is a heritage and every generation
should cherish it.
Modi, who inaugurated a statue
of Sardar Vallabhbhai Patel
at the CAG office where he is
attending the first Audit Diwas
in Delhi, said there are very few
institutions that become stronger,
more mature and relevant with
the passage of time.
The PM said that while assessing
the work of the government, the
CAG has the advantage of an
outsider’s viewpoint. “We make
systematic improvements with
the help of whatever you tell us,
we see it as cooperation,” he was
quoted as saying by news agency
ANI.

81. Audit Diwas will be


celebrated on which of the (c) The CAG is the (c) Article 148 (a) Appointed by the Chief
following days? statutory auditor of Justice of India by a
(d) Article 151
(a) 12th November Government-owned warrant under his hand
(b) 15th November corporations. 84. CAG enjoys the same Status and seal.
(d) It conducts as which of the following? (b) CAG has a tenure of five
(c) 16th November
supplementary audit of (a) Chief Justice of years or upto the age of
(d) 18th November
government companies, India 62 years, whichever is
82. Which of the following is where Government is (b) Sitting Supreme Court earlier.
NOT TRUE about a CAG? having an equity share Judge (c) He holds his office
of 51 per cent. (c) Chief Justice of High till the pleasure of the
(a) CAG is the Statutory
Body in India. Court President.
83. The office of CAG takes its
(b) CAG is empowered power from which of the (d) Sitting Justice of High (d) CAG can be removed
to Audit all receipts following Articles in the Court by the President on the
and expenditure of the Constitution of India? same grounds and in the
central as well as state 85. Which of the following same manner as a judge
(a) Article 124 statements is TRUE?
governments. of the Supreme Court.
(b) Article 196

81. (c) 16th November of India. precedence. It enjoys same Tenure: A period of six years
Prime Minister, Narendra 83. (c) Article 148 status as the sitting judge of or upto the age of 65 years,
Modi, is set to address the CAG is the Constitutional Supreme Court. whichever is earlier.
event of first Audit Diwas on Authority in India. It was 85. (d) CAG can be removed by Removal: CAG can be removed
November 16, 2021. established in accordance with the President on the same by the President on the same
82. (a) CAG is the Statutory Body Article 148 of the Constitution grounds and in the same grounds and in the same
in India. of India. manner as a judge of the manner as a judge of the
CAG is the Constitutional 84. (b) Sitting Supreme Court Supreme Court. Supreme Court. He does not
Authority in India It was Judge Appointed by the President of hold his office till the pleasure
established in accordance with The CAG has been ranked India by a warrant under his of the President.
Article 148 of the Constitution at 9th place in the order of hand and seal.
Bhopal +91-7676564400 Indore +91-9589613810, +91-731-4987379 Prayagraj +91-8114000926, +91-8114000927 Kanpur +91-8576066660, +91-8707605589 E: support@toprankers.com
Gurugram +91-8448444207 Delhi +91-9810610466 Ranchi +91-9334969993 Lucknow +91-6390576666 Jabalpur +91-7004386936 W: www.toprankers.com
19 The CLAT Post • November 2021

86. Which of the following will (d) Patel Committee


replace [1] in the above
passage? 89. Which of the following
Articles of the Indian
(a) Article 41
Constitution is in Conflict
(b) Article 44 with the Article 14, right to
(c) Article 46 equality?
(d) Article 51 (a) Article 19
(b) Article 21
87. Which of the following is
INCORRECT about the (c) Article 25
Uniform Civil Code? (d) Article 23
(a) UCC is one that would
provide for one law 90. In which of the following
for the entire country, Cases has the Supreme
applicable to all Court of India stressed the
religious communities need for UCC?
in their personal matters (a) Shah Bano Case
such as marriage, (b) Keshwananda Bharti
divorce, inheritance, Case
adoption etc. (c) Minerwa Mills
(b) It is one of the Case
Passage (Q.86-Q.90): without insisting or awaiting Fundamental Duties.
approval of the competent (d) Maneka Gandhi Case
The Allahabad High Court has (c) The code will simplify
asked the Centre to consider district authority with regard to
conversion of faith. the complex laws
Supreme Court guidelines for
around marriage
the implementation of Uniform “The UCC is necessity and ceremonies, inheritance,
Civil Code across the country. mandatorily required today. succession, adoptions
In response to as many as 17 It cannot be made ‘purely
making them one for
petitions related to interfaith voluntary’ as was observed by
all.
marriages, the court had Friday B.R. Ambedkar 75 years back,
asked the central government in view of the apprehension and (d) All of the above
to consider setting up a panel fear expressed by the members
for implementing the mandate 88. Which of the following
of the minority community,” the
of [1], which says that the “state Committees was formed in
court said.
shall endeavour to secure for the 1941 to codify Hindu Law?
Justice Suneet Kumar said it is
citizens a UCC throughout the the need of the hour that the (a) Ambedkar
territory of India”. Parliament comes up with a Committee
The court directed marriage “single family code” to protect (b) Motilal Nehru
registrar or officer of petitioners’ interfaith couples from being Committee
districts to immediately register “hounded as criminals”. (c) BN Rau
the marriage of the petitioners Committee

86. (b) Article 44 territory of India. Hindu laws. The committee, religion gets into conflict
Article 44 says that the “State Article 44 is one of the in accordance with scriptures, with the concepts of equality
shall endeavour to secure for Directive Principles of State recommended a codified enshrined under Article 14 of
the citizens a uniform civil Policy (DPSP). Hindu law, which would give Indian Constitution.
code throughout the territory 88. (c) BN Rau Committee equal rights to women. The 90. (a) Shah Bano Case
of India”. Increase in legislations dealing 1937 Act was reviewed and Starting with its judgment
87. (b) It is one of the with personal issues in the far the committee recommended in Shah Bano Case in 1985,
Fundamental Duties. end of the British rule forced a civil code of marriage and the top court has dwelt on
the government to form the succession for Hindus. the UCC in several of its
Article 44 of the Constitution B N Rau Committee to codify 89. (c) Article 25 judgments. In many of these,
lays down that the state shall Hindu law in 1941. The task Article 25 of Indian the court favoured a common
endeavour to secure a UCC of the Hindu Law Committee constitution, that seeks to law for all citizens and
for the citizens throughout the was to examine the question preserve the freedom to reminded the Parliament of
of the necessity of common practise and propagate any the spirit of Article 44.
Bhopal +91-7676564400 Indore +91-9589613810, +91-731-4987379 Prayagraj +91-8114000926, +91-8114000927 Kanpur +91-8576066660, +91-8707605589 E: support@toprankers.com
Gurugram +91-8448444207 Delhi +91-9810610466 Ranchi +91-9334969993 Lucknow +91-6390576666 Jabalpur +91-7004386936 W: www.toprankers.com
20 The CLAT Post • November 2021

Passage (Q.91-Q.95): stock holding limits as


The Samyukt Kisan Morcha well as curbs on inter-
(SKM) held a meeting on State and intra-State
Sunday at the Singhu border trade.
of the national capital, Delhi, (c) it created a framework
where it discussed the future for contract farming.
course of action on farmers’ (d) All of the above
protests. The meeting comes in
the backdrop of Prime Minister 94. Which of the following is
Narendra Modi’s surprise CORRECT?
announcement last week
(a) A combined effect of
declaring that his government
these laws was to help
will withdraw the controversial
in creating a ‘One
agriculture laws that prompted
Nation, One Market’ for
year-long protests from tens of
agricultural produce.
thousands of farmers and posed
a significant political challenge (b) A model APMC Act was
to the administration. organisation of farmers’ bodies, (d) Farmers Support passed in 2003 and then
The Union Cabinet is likely to the SKM, to this effect, where Price (Protection and circulated among the
discuss the withdrawal of the they discussed where the future Guarantee) Act, 2020 states.
three contentious farm laws at a of the protests lies. (c) The Farmers are
meeting on Wednesday, people 92. Minimum Support Price is demanding MSP as
familiar with the development 91. Which of the following is given on how many Crops recommended by
told HT. They also hinted NOT one of the Farms laws in India? the Swaminathan
that the Cabinet is expected which are being repealed? (a) 19 (b) 21 Commission.
to discuss the introduction of (a) Farmers Produce (c) 22 (d) 23 (d) All of the above
a new bill during the Winter Trade and Commerce
Session to repeal the three laws. (Promotion and 93. Which of the following was 95. The Swaminathan
However, even though the Facilitation) Act, 2020 one of the aims of the three Commission Report on
controversial farm laws (b) Farmers (Empowerment farm laws? MSP is also known as?
were on their way out, the and Protection) (a) The laws aimed to do (a) C2+50 Formula
protesting farmers said that Agreement on Price away with government (b) C1+50 Formula
the demonstrations would Assurance and Farm interference in
continue until Parliament (c) C3+35% Formula
Services Act, 2020 agricultural trade by
repealed the three contentious creating trading areas (d) C3+35% Formula
(c) Essential Commodities
legislation. Sunday’s meeting free of middlemen
(Amendment) Act, 2020
was conducted by the umbrella
(b) It sought to remove

91. (d) Farmers Support Price commodities such as cereals, government interference laws was to help in creating
(Protection and Guarantee) pulses, oilseeds, edible oils, in agricultural trade by a ‘One Nation, One Market’
Act, 2020 onion and potato from the list creating trading areas free of for agricultural produce.
Farmers Produce Trade and of essential commodities. middlemen and government A model APMC Act was
Commerce (Promotion and 92. (d) 23 taxes outside the structure of passed in 2003 and then
Facilitation) Act, 2020: It As of now, CACP recommends Agricultural Produce Market circulated among the states.
is aimed at allowing trade MSPs of 23 commodities, Committees (APMCs). The Farmers are demanding
in agricultural produce which comprise 7 cereals It will allow farmers an option MSP as recommended by the
outside the existing APMC (paddy, wheat, maize, to sell their produce directly to Swaminathan Commission.
(Agricultural Produce Market sorghum, pearl millet, barley these new zones, without going 95. (a) C2+50 Formula
Committee) mandis. and ragi), 5 pulses (gram, tur, through the middlemen and The Swaminathan Commission
Farmers (Empowerment and moong, urad, lentil), 7 oilseeds paying levies such as mandi Report states that the
Protection) Agreement on (groundnut, rapeseed-mustard, fees. government should raise the
Price Assurance and Farm soyabean, seasmum, sunflower, It sought to remove stock MSP to at least 50% more than
Services Act, 2020: It seeks safflower, nigerseed), and holding limits as well as curbs the weighted average cost of
to provide a framework for 4 commercial crops (copra, on inter-State and intra-State production. It is also known as
contract farming. sugarcane, cotton and raw trade, and create a framework the C2+ 50% formula.
Essential Commodities jute). for contract farming.
(Amendment) Act, 2020: 93. (d) All of the above 94. (d) All of the above
It is aimed at removing The Bills aim to do away with A combined effect of these
Bhopal +91-7676564400 Indore +91-9589613810, +91-731-4987379 Prayagraj +91-8114000926, +91-8114000927 Kanpur +91-8576066660, +91-8707605589 E: support@toprankers.com
Gurugram +91-8448444207 Delhi +91-9810610466 Ranchi +91-9334969993 Lucknow +91-6390576666 Jabalpur +91-7004386936 W: www.toprankers.com
21 The CLAT Post • November 2021

Passage (Q.96-Q.100): but also to the myriad benefits 97. International Day of Girl 99. Which of the following
Educational disruption due to of attending school, at an Child is celebrated on is the theme for the
prolonged closure of schools unparalleled scale,” said Stefania which of the following International Day of the
across the globe will not only Giannini, [1], Assistant Director- days? Girl Child 2021?
have alarming effects on learning General for Education. (a) 11th October (a) My voice, our equal
loss but also poses threat to “Educational disruption of this (b) 12th October future
gender equality, a new study by extent has alarming effects on (b) GirlForce: Unscripted
[1] has pointed out. learning loss and school dropout. (c) 11th November
and Unstoppable
The global study titled “When Beyond this, it poses threats (d) 12th November
to gender equality, including (c) With Her: A Skilled Girl
schools shut: Gendered impacts 98. Which of the following
effects on health, well-being Force
of COVID-19 school closures”
and protection that are gender declaration is the most (d) Digital generation. Our
brings to the fore that girls and
specific,” Giannini sai(d). progressive blueprint for Generation.
boys, young women and men
advancing women’s rights?
were affected differently by
96. Which of the following will (a) Durban 100. [1] in the above passage
school closures, depending on
replace [1] in the above Declaration was established in which
the context.
passage? year?
“At the peak of the COVID-19 (b) Beijing Declaration
pandemic, 1.6 billion students (a) UNICEF (a) 1945 (b) 1956
(c) Chicago
in 190 countries were affected (b) UNESCO Declaration (c) 1976 (d) 1998
by school closures. Not only did (c) UNHRC (d) Moscow Declaration
they lose access to education, (d) UNDP

96. (b) UNESCO The Report was released 99. (d) Digital generation. Our 100. (a) 1945
Educational disruption due to on the occasion of the 2021 Generation. From 1 to 16 November 1945,
prolonged closure of schools International Day of the Girl It is celebrated for empowering a Conference to establish
across the globe will not only Child (11th October). and amplifying the voices of this Organization met in
have alarming effects on 98. (b) Beijing Declaration young girls around the world. London and the Constitution
learning loss but also poses The Beijing Declaration and The theme for International of UNESCO is signed by
threat to gender equality, a Platform for Action is widely Day of Girl Child 2021 is 41 States. The Constitution
new study by UNESCO has known as the most progressive ‘Digital generation’. Our will come into force on
pointed out. blueprint for advancing generation’. 4 November 1946 after
97. (a) 11th October women’s rights. ratification by 20 signatories.
Bhopal +91-7676564400 Indore +91-9589613810, +91-731-4987379 Prayagraj +91-8114000926, +91-8114000927 Kanpur +91-8576066660, +91-8707605589 E: support@toprankers.com
Gurugram +91-8448444207 Delhi +91-9810610466 Ranchi +91-9334969993 Lucknow +91-6390576666 Jabalpur +91-7004386936 W: www.toprankers.com
22 The CLAT Post • November 2021

EXPLAINED: WHAT IS THE FARM LAWS


REPEAL BILL, 2021?
Farm laws repealed: What is the Farm Laws Repeal Bill, 2021? Has the government given any
reason for repeal of the farm laws?
The Lok Sabha Monday passed
the Farm Laws Repeal Bill,
2021 without any discussion.
The Bill, which is aimed at
repealing three farm laws, was
introduced in the house by
Union Minister of Agriculture
and Farmers’ Welfare Narendra
Singh Tomar.

What is the Farm Laws Repeal


Bill, 2021?

The Farm Laws Repeal Bill,


2021 is aimed at repealing
the three farm laws –
Farmers (Empowerment and amending the Essential on November 19. Two days The six-page Bill contains only
and Protection) Agreement Commodities Act, 1955. The after the Prime Minister’s three sections. The first section
on Price Assurance and Bill was necessitated after announcement, the Union defines the title of the Act –
Farm Services Act, 2020, Prime Minister Narendra Cabinet cleared the draft of the Farm Laws Repeal Act,
the Farmers’ Produce Trade Modi announced the the Bill. Now the Bill has been 2021, the second section has
and Commerce (Promotion government’s intention introduced in Lok Sabha. provisions to repeal three farm
and Facilitation) Act, 2020, to repeal the three laws in laws, and the third section
view of ongoing farmers’ How many sections are there relates to omitting sub-section
the Essential Commodities in the Bill?
(Amendment) Act, 2020 – protests against these laws (1A) from section 3 of the
Bhopal +91-7676564400 Indore +91-9589613810, +91-731-4987379 Prayagraj +91-8114000926, +91-8114000927 Kanpur +91-8576066660, +91-8707605589 E: support@toprankers.com
Gurugram +91-8448444207 Delhi +91-9810610466 Ranchi +91-9334969993 Lucknow +91-6390576666 Jabalpur +91-7004386936 W: www.toprankers.com
23 The CLAT Post • November 2021

Essential Commodities Act, may be issued if there is a “Without taking away the was inserted vide the Essential
1955. hundred per cent increase in existing mechanisms available Commodities (Amendment)
the retail price of horticultural to farmers, new avenues were Act, 2020 (22 of 2020),” it
What is sub-section (1A) produce; or fifty per cent provided for trade of their states.
under section 3 of the increase in the retail price of produce. Besides, farmers were
Essential Commodities Act non-perishable agricultural free to select the avenues of How many days the farm laws
which is being removed? foodstuffs, over the price their choice where they can get were in effect?
prevailing immediately more prices for their produce The journey of three farm
The government had inserted
preceding twelve months, or without any compulsion,” the laws began on June 5, 2020
sub-section (1A) in the
average retail price of last five statement said. when the President of India
section 3 of the Essential
years, whichever is lower. promulgated three ordinances­
Commodity Act, 1955 that “However, the operation of
empowers the government to Has the government given any the aforesaid Farm laws has – Essential Commodities
control production, supply, reason for repeal of the farm been stayed by the Hon’ble (Amendment) Ordinance,
distribution, etc., of essential laws? Supreme Court of India. 2020; The Farming Produce
commodities. The sub-section During the COVID period, Trade and Commerce
(1A) provides a mechanism Agriculture Minister Narendra the farmers have worked hard (Promotion and Facilitation)

to increase production and


to regulate the supply of Singh Tomar, who piloted the fulfil the needs of the nation. Ordinance, 2020; and The
foodstuffs, including cereals, Farm Laws Repeal Bill, 2021, As we celebrate the 75th Year Farmers (Empowerment
pulses, potato, onions, edible has stated several reasons for of Independence— “Azadi and Protection) Agreement
oilseeds and oils under taking this legislative step. Ka Amrit Mahotsav”, the on Price Assurance and
“extraordinary circumstances” In a statement of objects and need of the hour is to take Farm Services Ordinance,
which may include war, reasons, which forms the part everyone together on the 2020. These ordinances were
famine, extraordinary price of the Bill, Tomar said, “Even path of inclusive growth and later replaced with proper
rise and natural calamity of though only a group of farmers development,” it said. legislation in September 2020.
grave nature. It also prescribes are protesting against these However, the implementation
the price triggers for imposing laws, the Government has tried “In view of the above, the of three farm laws was stayed
stock limits. Under the sub- hard to sensitise the farmers aforesaid Farm Laws are by the Supreme Court on
section (1A), any action on on the importance of the Farm proposed to be repealed. It is January 12, 2021. So, these
imposing stock limit shall be Laws and explain the merits also proposed to omit sub- laws were in effect for only 221
based on price rise and an through several meetings and section (1A) of section 3 of days.
order for regulating stock limit other forums.” the Essential Commodities
of any agricultural produce Act, 1955 (10 of 1955) which
Bhopal +91-7676564400 Indore +91-9589613810, +91-731-4987379 Prayagraj +91-8114000926, +91-8114000927 Kanpur +91-8576066660, +91-8707605589 E: support@toprankers.com
Gurugram +91-8448444207 Delhi +91-9810610466 Ranchi +91-9334969993 Lucknow +91-6390576666 Jabalpur +91-7004386936 W: www.toprankers.com
24 The CLAT Post • November 2021

Passage (Q.101-Q.105): of public trust in the police. (c) Indian Police the north.
The SMART Policing idea The experts from various Foundation (d) All of the above
was envisioned, articulated Institutions of repute, including (d) India Civil Services
and introduced by Prime IIT-Kanpur and Tata Institute Organisation 104.Which of the following
Minister Narendra Modi of Social Sciences (TISS) in State is the worst performer
at the Conference of DGPs Mumbai were involved in this 103.Which of the following is on the Index?
of State and Central Police survey. CORRECT about SMART (a) Bihar
Organizations, held at Policing Index?
101.Which of the following (b) Uttar Pradesh
Guwahati, in 2014 and it (a) The SMART scores
envisaged systemic changes to States has topped in the (c) Chhattisgarh
are set on a scale of 1
transform the Indian police Index? to10 and are indicative (d) Punjab
to be strict and sensitive, (a) Andhra Pradesh of the levels of citizen
modern and mobile, alert 105.The subject of Police
(b) Goa satisfaction, a score of comes under which of
and accountable, reliable and 10 being the highest
responsive, techno-savvy and (c) Uttar Pradesh the following lists of
level of satisfaction. the schedule 7 of the
trained. (d) Telangana
(b) The survey had 10 sets Constitution of India?
The purpose of the survey 102.The survey was conducted of questionnaires
was to gather information on (a) Union List
by which of the following (c) Southern states and
citizens’ perceptions of the (b) State List
organisation? some in the Northeast
impact of the SMART policing (c) Concurrent List
initiative and to gauge public (a) Indian Police Services fared better on most
perceptions of the quality of (b) Indian Police Reform policing indices (d) None of the above
policing in India and the level Commission compared to states in

101. (a) Andhra Pradesh think tank and policy advocacy citizen satisfaction, a score of The bottom five states
The top five states with the platform, conducted the “IPF 10 being the highest level of are Bihar, Uttar Pradesh,
highest score on overall SMART Policing Survey satisfaction. The survey had 10 Chhattisgarh, Jharkhand and
policing are Andhra Pradesh, 2021” in 29 states and Union sets of questionnaires Punjab.
Telangana, Assam, Kerala and Territories of the country and Southern states and some in 105. (b) State List
Sikkim. published its report. the Northeast fared better Police come under the state
102. (c) Indian Police 103. (d) All of the above on most policing indices list of schedule 7 of the Indian
Foundation The SMART scores are set compared to states in the constitution.
The Indian Police Foundation on a scale of 1 to10 and are north.
(IPF), a multidisciplinary indicative of the levels of 104. (a) Bihar
Bhopal +91-7676564400 Indore +91-9589613810, +91-731-4987379 Prayagraj +91-8114000926, +91-8114000927 Kanpur +91-8576066660, +91-8707605589 E: support@toprankers.com
Gurugram +91-8448444207 Delhi +91-9810610466 Ranchi +91-9334969993 Lucknow +91-6390576666 Jabalpur +91-7004386936 W: www.toprankers.com
25 The CLAT Post • November 2021

Passage (Q.106-Q.110): bedspreads, and the street Tourism Organization (c) The World Tourism
[1], a 28-square km weavers’ doesn’t seem to end. Outside headquartered? Organization is
hamlet on the outskirts of the shops are swanky cars and (a) Lisbon, Portugal the United Nations
Hyderabad, has been catapulted SUVs driven from Hyderabad Specialised Agency
(b) Madrid, Spain
onto the global stage after and other places. “This is responsible for
bagging the ‘Best World nothing. On Saturday and (c) Bern, Switzerland the promotion of
Tourism Village’ tag awarded by Sundays, we cannot even talk. (d) Paris, France responsible, sustainable
United Nations World Tourism People come from other States and universally
Organisation (UNWTO). The too for festival and wedding 108.[1] is also known as which accessible tourism.
village is a shimmering pleats shopping,” says Satyanarayana, of the following?
(d) None of the above
of colour with billboards of silk secretary of the biggest (a) Silk City of India
sarees, cotton fabrics and phone shopping complex in the (b) Cotton City of India 110.Which of the following
numbers. village. is a member nation of
(c) City of Temples
As vehicles turn left from the UNWTO?
106. Which of the following (d) Coconut City of India
Hyderabad highway and enter replaces [1] in the passage? (a) United States of
the MG Road leading to the [1] 109.Which of the following is America
(a) Nampally
village, signs of development incorrect about UNWTO? (b) United Kingdom
are immediately apparent. Farm (b) Asifnagar
(a) It was established in the (c) Australia
lands have been filled up, hills (c) Golconda
year 1975. (d) China
have been cleared for housing (d) Pochampally
plots, layouts dot the landscape. (b) It declared September
The main street is lined with 107.In which of the following 27 as ‘World Tourism
shops advertising [1] Ikat, cities is UN World Day’.

106. (d) Pochampally the United Nations specialized 109. (d) None of the above chosen as on that day in 1970,
Pochampally, a 28-square agency entrusted with the UNWTO the United the Statutes of the UNWTO
km weavers’ hamlet on the promotion of responsible, Nations Specialised Agency were adopted.
outskirts of Hyderabad, has sustainable and universally responsible for the promotion 110. (d) China
been catapulted onto the accessible tourism. Its of responsible, sustainable The United States, Australia,
global stage after bagging the headquarters are in Madrid, and universally accessible and the United Kingdom
‘Best World Tourism Village’ Spain. tourism. It was established in are major powers in World
tag awarded by United Nations 108. (a) Silk City of India the year 1975. Since 1980, the Tourism, but not a member
World Tourism Organisation Pochampally is often referred United Nations World Tourism of the World Tourism
(UNWTO). to as the silk city of India for Organization has celebrated Organization (UNWTO).
107. (b) Madrid, Spain the exquisite sarees that are World Tourism Day as However, China is a member
The World Tourism woven through a unique style international observances on of UNWTO.
Organization (UNWTO) is called Ikat. September 27. This date was
Bhopal +91-7676564400 Indore +91-9589613810, +91-731-4987379 Prayagraj +91-8114000926, +91-8114000927 Kanpur +91-8576066660, +91-8707605589 E: support@toprankers.com
Gurugram +91-8448444207 Delhi +91-9810610466 Ranchi +91-9334969993 Lucknow +91-6390576666 Jabalpur +91-7004386936 W: www.toprankers.com
26 The CLAT Post • November 2021

Passage (Q.111-Q.115):
The Centre has set in motion the
process of creating the National
Interlinking of Rivers Authority
(NIRA), an independent
autonomous body for planning,
investigation, financing and
the implementation of the river
interlinking projects in the
country.
NIRA, to be headed by a
Government of India Secretary-
rank officer, will replace the
existing [1] and will function
as an umbrella body for all
river linking projects. The
new body will coordinate
with neighbouring countries
and concerned states and
departments and will also have
powers on issues related to
environment, wildlife and forest
clearances under river linking
projects and their legal aspects.

111.Which of the following


is replaced by [1] in the Purpose Vehicle (SPV) (d) All of the above is the first river-linking
passage? for individual link project in India?
(a) National Water projects. 113. Sir Arthur Cotton,
(a) Mandovi-Zuari
Development Agency a British general and
(b) It can raise funds and (b) Ken-Betwa
(b) National Water irrigation engineer
act as a repository of
Informatics Centre suggested linking which (c) Periyar-Tungabhadra
borrowed funds or
of the following rivers for (d) Sabarmati-Luni
(c) National River money received on
navigational purposes?
Conservation deposit or loan given on
(a) Ganga and Cauvery 115. NIRA will be held by
Directorate interest.
which of the following?
(d) National Mission for (c) It will have powers (b) Brahmaputra and
Godavari (a) Cabinet-rank member
Clean Ganga on issues related to
environment, wildlife (c) Sutlej, Beas, Ravi, (b) Secretary-rank member
112.Which of the following is and forest clearances Chenab, and Jhelum (c) Retired Supreme Court
true regarding NIRA? under river linking (d) Krishna and Narmada Judge
(a) It will have the authority projects and their legal (d) Minister of State
to set up a Special aspects. 114. Which of the following

111. (a) National Water repository of borrowed funds The ambitious idea of to transfer surplus water from
Development Agency or money received on deposit interlinking of Indian rivers the Ken river in MP to Betwa
NIRA, to be headed by or loan given on interest. originated in the British in UP to irrigate the drought-
a Government of India Besides, it will also have a engineering thinking with Sir prone Bundelkhand region.
Secretary-rank officer, will mandate to coordinate with Arthur Cotton, an irrigation 115. (b) Secretary-rank member
replace the existing National neighbouring countries “as engineer, who suggested NIRA, to be headed by
Water Development Agency directed” by the Ministry of interlinking the Ganga and the a Government of India
and will function as an Jal Shakti or the Ministry of Cauvery rivers for navigational Secretary-rank officer, will
umbrella body for all river External Affairs. It will also purposes. replace the existing National
linking projects. have the power to set up a 114. (b) Ken-Betwa Water Development Agency
112. (d) All of the above Special Purpose Vehicle (SPV) The Ken-Betwa Link (NWDA) and will function as
NIRA will have the power for individual link projects. Project (KBLP) is the River an umbrella body for all river
to raise funds and act as a 113. (a) Ganga and Cauvery interlinking project that aims linking projects.
Bhopal +91-7676564400 Indore +91-9589613810, +91-731-4987379 Prayagraj +91-8114000926, +91-8114000927 Kanpur +91-8576066660, +91-8707605589 E: support@toprankers.com
Gurugram +91-8448444207 Delhi +91-9810610466 Ranchi +91-9334969993 Lucknow +91-6390576666 Jabalpur +91-7004386936 W: www.toprankers.com
27 The CLAT Post • November 2021

Passage (Q.116-Q.120): in a statement. Prime Minister (a) It seeks to build peace (b) Irina Bokova
Srinagar, the summer capital Modi congratulated people of through international (c) Kristalina Georgieva
of Jammu and Kashmir, has Jammu and Kashmir on the cooperation in
(d) Tedros Adhanom
joined the UNESCO’s network achievement. Education, the Sciences
of creative cities, the world body and Culture. 120.Which of the following
116. Which of the following
announced on Monday, with (b) UNESCO’s it NOT TRUE about the
is replaced by [1] in the
Prime Minister Narendra Modi Headquarters are UNESCO Creative Cities
passage?
hailing the inclusion as a “fitting located in Washington Network?
recognition” for the vibrant (a) 51 (b) 67 DC. (a) The network covers
cultural ethos of the old city. It (c) 49 (d) 42 eleven creative fields.
(c) Three UNESCO
has been designated as a creative
117.Which of the following is member states are not (b) It was created in 2010.
city of craft and folk arts, it said.
the UNESCO Creative City UN members: Cook (c) The UNESCO
Worldwide, [1] new cities have Islands, Niue, and
joined the UNESCO’s Creative of Gastronomy in India? Creative Cities aims
Palestine. to achieve Sustainable
Cities Network (UCCN) (a) Mumbai
following their designation by (d) The Organization has Development Goal 15
(b) Hyderabad
UNESCO Director-General 193 Members and 11 through innovative
(c) Varanasi Associate Members. thinking and action.
[2], in recognition of their
commitment to placing culture (d) Chennai (d) All of the above.
119.Which of the following
and creativity at the heart is replaced by [2] in the
118.Which of the following
of their development and to
is NOT TRUE about passage?
sharing knowledge and good
UNESCO? (a) Audrey Azoulay
practices, the UNESCO said

116. (c) 49 a statement. 118. (b) UNESCO’s Headquarters and Cultural Organization
Worldwide, 49 new cities have 117. (b) Hyderabad are located in Washington DC. (UNESCO) since 2017,
joined the UNESCO’s Creative Mumbai has been designated UNESCO’s Headquarters becoming the second female
Cities Network (UCCN) as Creative City of Films are located in Paris and the leader of the organization.
following their designation by and Hyderabad as a Creative Organization has more than 50 120. (d) All of the above.
UNESCO Director-General, City of Gastronomy. Earlier, field offices around the world. The network covers seven
in recognition of their Indian cities like Chennai and 119. (a) Audrey Azoulay creative fields. It was created
commitment to placing culture Varanasi have been included Audrey Azoulay is a French in 2004. The UNESCO
and creativity at the heart in UNESCO Cities of Music civil servant and politician Creative Cities aims to achieve
of their development and to while. Jaipur has been included who has been serving as the Sustainable Development
sharing knowledge and good in the City of Crafts and Folk Director-General of the United Goal 11 through innovative
practices, the UNESCO said in Arts. Nations Educational, Scientific thinking and action.
Bhopal +91-7676564400 Indore +91-9589613810, +91-731-4987379 Prayagraj +91-8114000926, +91-8114000927 Kanpur +91-8576066660, +91-8707605589 E: support@toprankers.com
Gurugram +91-8448444207 Delhi +91-9810610466 Ranchi +91-9334969993 Lucknow +91-6390576666 Jabalpur +91-7004386936 W: www.toprankers.com
28 The CLAT Post • November 2021

Passage (Q.121-Q.125): 123.The Banking Ombudsman


Prime Minister Narendra Modi Scheme was first launched
launched two consumer-centric in India in which of the
initiatives of the Reserve Bank of following Years?
India (RBI) on Friday (November (a) 2001 (b) 2006
12) via video conferencing. The (c) 2003 (d) 2009
two initiatives are the [1] and
the Reserve Bank Integrated 124.Which of the following
Ombudsman Scheme. With this, is replaced by [2] in the
the country has opened up the passage?
government bond market for
(a) Institutional Buyers
[2] investors. Along with PM
Modi, Finance Minister Nirmala (b) Non-Institutional
Sitharaman and RBI Governor Buyers
Shaktikanta Das were also part of (c) Retail
the event. (d) Foreign Investors
PM Modi said the two schemes
launched will expand the scope 125.Which of the following is
of investment in the country NOT TRUE?
and make access to capital (a) The scheme is
markets easier and more secure jurisdiction neutral.
for investors. “In the last 7 (b) Its processing centre has
years, NPAs (non-performing been set-up in Delhi
assets) were recognized with for initial handling
transparency. The focus has been
122.Which of the following is Finance” in 1926. This of complaints in any
on resolution; multiple reforms
NOT TRUE in relation to commission was also language.
were carried out in the financial
Reserve Bank of India? known as Hilton Young (c) The regulated entity will
system and public sector banks,”
(a) The concept of Reserve Commission. not have any right to
he added.
Bank of India was (c) In 1959, the Reserve appeal in cases where
121. Which of the following based on the strategies Bank of India was an award is issued by
is replaced by [1] in the formulated by Dr. nationalized and the ombudsman against
passage? Ambedkar in his book became a member bank it for not furnishing
(a) RBI Retail Direct named “The Problem of of the Asian Clearing satisfactory and timely
Scheme the Rupee – Its origin Union. information.
(b) RBI Non-Institutional and its solution”. (d) The Central Office of (d) There will also be a
Buyers Scheme (b) This central banking the Reserve Bank was multilingual toll-free
(c) RBI Institutional Buyers institution was initially established number that will
Scheme established based on in Calcutta but was provide all relevant
the suggestions of the permanently moved to information on
(d) RBI Foreign Investors “Royal Commission Mumbai in 1937. grievance redress.
Scheme on Indian Currency &

121. (a) RBI Retail Direct Scheme of India was nationalized and an efficient forum for bank Scheme. With this, the country
became a member bank of the customers for the resolution of has opened up the government
Prime Minister Narendra Asian Clearing Union. complaints related to certain bond market for retail
Modi launched two consumer- In 1949, the Reserve Bank of services rendered by the banks. investors.
centric initiatives of the India was nationalized and 124. (c) Retail 125. (b) Its processing centre has
Reserve Bank of India (RBI) became a member bank of the Prime Minister Narendra been set-up in Delhi for initial
on Friday (November 12) via Asian Clearing Union. Modi launched two consumer- handling of complaints in any
video conferencing. The two 123. (b) 2006 centric initiatives of the language.
initiatives are the RBI Retail The Banking Ombudsman Reserve Bank of India (RBI) Its processing centre has been
Direct Scheme and the Reserve Scheme (BOS) was first on Friday (November 12) via set-up in Chandigarh for
Bank Integrated Ombudsman introduced in 2006. It was video conferencing. The two initial handling of complaints
Scheme. updated in July 2017. As initiatives are the RBI Retail in any language.
122. (c) In 1959, the Reserve Bank per the RBI site, the BOS is Direct Scheme and the Reserve
Bank Integrated Ombudsman
Bhopal +91-7676564400 Indore +91-9589613810, +91-731-4987379 Prayagraj +91-8114000926, +91-8114000927 Kanpur +91-8576066660, +91-8707605589 E: support@toprankers.com
Gurugram +91-8448444207 Delhi +91-9810610466 Ranchi +91-9334969993 Lucknow +91-6390576666 Jabalpur +91-7004386936 W: www.toprankers.com
29 The CLAT Post • November 2021

Passage (Q.126-Q.130): Road Transport and


The needs of defence and Highways.
environment have to be (b) The cost of the project
balanced and a “nuanced” is around Rs. 12,000
approach is required, the
Supreme Court said on Tuesday Crore.
while hearing an appeal (c) The work under
against the widening of roads the programme is
in Uttarakhand hills for the being implemented
“Char Dham project”. While a on Engineering,
non-profit had opposed road-
widening -- citing the felling Procurement and
of trees in the wildlife-rich Construction (EPC)
area that’s leading to increasing mode.
instances of landslides -- the (d) All of the above
Centre had sought the court’s
approval to widen the roads to 128.Which of the following
border to 10 meters. is replaced by [2] in the
The Char Dham highway passage?
project which connects the
four shrines of Gangotri, [1], (a) Greater Himalaya
Kedarnath and Badrinath in (b) Garhwal Himalaya
[2], has 899-km road which the (c) Nanda Devi
Centre wants to broaden near
Dehradun. The top court had (d) Mana Peak
earlier said that roads can’t be
wider than a total of 5 meters. 129.Border Road Organisation
The Centre had cited the was established in which of
“tremendous” Chinese troop the following year?
buildup on the other side and
(a) 1955 (b) 1960
said under the circumstances,
“wide roads are of strategic (c) 1966 (d) 1975
importance”.
130.Which of the following
126. Which of the following is the demarcation that
is replaced by [1] in the separates Indian-controlled
passage? territory from Chinese-
(a) Rishikesh controlled territory?
(b) Haridwar (a) Line of Control
(c) Yamunotri (b) Line of Actual Control
(d) Mount Kailash (c) Durand Line
(d) McMohan Line
127.Which of the following is
TRUE with regards to the
Chardham Prariyojna?
(a) It is a programme taken
up by the Ministry of

126. (c) Yamunotri programme taken up by the project which connects the maintains the roads in the
The Char Dham highway Ministry of Road Transport four shrines of Gangotri, bordering areas of the country.
project which connects the and Highways. The cost of Yamunotri, Kedarnath It was established in 1960 for
four shrines of Gangotri, the project is around Rs. and Badrinath in Garhwal the development of roads in
Yamunotri, Kedarnath 12,000 Crore. The work under Himalayas, has 899-km road terms of strategic importance.
and Badrinath in Garhwal the programme is being which the Centre wants to 130. (b) Line of Actual Control
Himalayas, has 899-km road implemented on Engineering, broaden near Dehradun. The Line of Actual Control
which the Centre wants to Procurement and Construction 129. (b) 1960 (LAC) is the demarcation that
broaden near Dehradun. (EPC) mode. Border Roads Organisation is separates Indian-controlled
127. (d) All of the above 128. (b) Garhwal Himalaya an undertaking of Government territory from Chinese-
Chardham Pariyojna is a The Char Dham highway of India which constructs and controlled territory.
Bhopal +91-7676564400 Indore +91-9589613810, +91-731-4987379 Prayagraj +91-8114000926, +91-8114000927 Kanpur +91-8576066660, +91-8707605589 E: support@toprankers.com
Gurugram +91-8448444207 Delhi +91-9810610466 Ranchi +91-9334969993 Lucknow +91-6390576666 Jabalpur +91-7004386936 W: www.toprankers.com
30 The CLAT Post • November 2021

committee has 10
members.

133.Which of the following


is replaced by [2] in the
passage?
(a) 2010 (b) 2011
(c) 2015 (d) 2016

134.Which of the following


is NOT TRUE about the
National Monuments
Authority?
(a) National Monument
Authority is set up
under the Ministry of
Culture.
(b) NMA considers
grant of permissions
to applicants for
construction
Passage (Q.131-Q.135): the chairperson of the NMA (b) Kiren Rijiju related activity in
Congress chief whip in the should have “proven experience (c) Arjun Munda the prohibited and
Rajya Sabha Jairam Ramesh and expertise in the field of
(d) Anurag Thakur regulated areas.
on Thursday moved a privilege archaeology, country and town
planning, architecture, heritage, (c) NMA was established in
motion against Culture Minister 132.Which of the following is
conservation architecture or 2011.
[1] over the appointment of not true about Privilege
former MP Tarun Vijay as the law….” Committee? (d) All of the above
chairperson of the National The Congress leader wrote (a) It is a Standing 135.Which of the following
Monuments Authority, a post that for the first time, the Committee. Articles of the Constitution
for which, Mr. Ramesh said, Mr. government had appointed a
(b) It examines the cases provides for privileges for
Vijay was not qualified. chairperson whose educational
and professional background of breach of the the President?
In a letter to Rajya Sabha
does not meet the requirements privileges of the House (a) Article 105
Chairman M. Venkaiah Naidu
of law passed by Parliament in and its members (b) Article 110
ahead of the winter session of
March [2]. and recommends
Parliament from November (c) Article 361
appropriate action.
29, Mr. Ramesh cited a section (d) Article 268
131. Which of the following (c) The Lok Sabha
of the Ancient Monuments
is replaced by [1] in the committee has 25
and Archaeological Sites and
passage? members.
Remains (Amendment and
Validation) Act, [2] that says (a) G. Kishan Reddy (d) The Rajya Sabha

131. (a) G. Kishan Reddy appropriate action. The Lok says the chairperson of the been setup as per provisions
Gangapuram Kishan Reddy Sabha committee consists of NMA should have “proven of The Ancient Monuments
is an Indian politician who is 15 members, while the Rajya experience and expertise and Archaeological Sites and
currently serving as Minister Sabha committee has 10 in the field of archaeology, Remains AMASR (Amendment
of Tourism, Culture and members. country and town planning, and Validation) Act, 2010
Development of North Eastern 133. (a) 2010 architecture, heritage, which was enacted in March,
Region of India. In a letter to Rajya Sabha conservation architecture or 2010.
132. (c) The Lok Sabha committee Chairman M. Venkaiah Naidu law….” 135. (c) Article 361
has 25 members. ahead of the winter session of 134. (c) NMA was established in The parliamentary privileges
Privilege Committee is a Parliament from November 2011. do not extend to the President
Standing Committee which 29, Mr. Ramesh cited a section About NMA. National who is also an integral part of
examines the cases of breach of of the Ancient Monuments Monuments Authority the Parliament. Article 361 of
the privileges of the House and and Archaeological Sites and (NMA) under the Ministry the Constitution provides for
its members and recommends Remains (Amendment and of Culture, Govt. of India has privileges for the President.
Validation) Act, 2010 that
Bhopal +91-7676564400 Indore +91-9589613810, +91-731-4987379 Prayagraj +91-8114000926, +91-8114000927 Kanpur +91-8576066660, +91-8707605589 E: support@toprankers.com
Gurugram +91-8448444207 Delhi +91-9810610466 Ranchi +91-9334969993 Lucknow +91-6390576666 Jabalpur +91-7004386936 W: www.toprankers.com
31 The CLAT Post • November 2021

(b) US$ 100 billion


(c) US$1 trillion
(d) US$5 trillion

138.International Solar Alliance


was launched by India and
France on the sidelines of
which of the following?
(a) COP 3
(b) COP 21
(c) COP 18
(d) COP 15

139.Which of the following is


true about ISA?
(a) The ISA is an
intergovernmental
treaty-based
organisation.
(b) ISA was co-founded by
Passage (Q.136-Q.140): of the ISA,Union Minister contribution to more rapid India and France during
In a big boost to accelerate for Environment, Forest & deployment of solar globally. It
the 2015 climate change
global adoption of solar Climate Change, [2] said, that will be particularly important
conference in Paris.
energy, John Kerry, U.S. Special this move will strengthen the for developing countries.”
ISA and propel future action (c) ISA is the nodal agency
Presidential Envoy for Climate
on providing a clean source of 136. Which of the following for implementing One
announced at the UNFCCC
energy to the world” is replaced by [1] in the Sun One World One
COP26 today that the United
passage? Grid (OSOWOG).
States of America (USA) Signing the framework
has joined the International agreement, U.S. Special (a) 101st (b) 100th (d) All of the above
Solar Alliance (ISA) as a Presidential Envoy for Climate (c) 104th (d) 110th
member country. U.S becomes John Kerry, said, “It has long 140.Which of the following
the [1] country to sign the been coming, and we are happy 137.ISA aims to mobilize how is replaced by [2] in the
framework agreement of the to join the International Solar much funding by 2030 to passage?
ISA to catalyze global energy Alliance, which Prime Minister assist developing countries (a) Prakash Javedekar
transition through a solar-led Narendra Modi took the lead in expanding their solar (b) Bhupendra Yadav
approach. in making. We worked out the power grids to meet their
(c) Piyush Goyal
Welcoming the United States details and this is a process energy needs?
we are pleased to be a part (d) Dharmendra Pradhan
of America as the [1] member (a) US$ 500 billion
of. This will be an important

136. (a) 101st If the world has to move to a tropic of Cancer and tropic ISA is the nodal agency for
U.S becomes the 101st clean and green future, these of Capricorn as prospective implementing One Sun One
country to sign the framework interconnected transnational members. World One Grid (OSOWOG),
agreement of the ISA to grids are going to be critical 139. (d) All of the above which seeks to transfer solar
catalyze global energy solutions,” PM Modi said The ISA is an power generated in one region
transition through a solar-led during the launch. intergovernmental treaty- to feed the electricity demands
approach. 138. (b) COP 21 based organisation with a of others.
137. (c) US$1 trillion The ISA, is an Indian initiative global mandate to catalyse 140.(b) Bhupendra Yadav
ISA also aims to mobilize that was launched by the solar growth by helping to Bhupendra Yadav is the
US$1 trillion of funding by Prime Minister of India and reduce the cost of financing Union Cabinet Minister of
2030 to assist developing the President of France on and technology. ISA, co- Labour and Employment,
countries in expanding their 30th November 2015 in Paris, founded by India and France Environment, Forest and
solar power grids to meet France on the side-lines of during the 2015 climate Climate Change in the
their energy needs. “The One the Conference of the Parties change conference in Paris Government of India.
Sun One World One Grid and (COP-21), with 121 solar has assumed centre-stage for
Green Grids Initiative is an resource rich countries lying India’s attempts at a global
idea whose time has come. fully or partially between the climate leadership role.
Bhopal +91-7676564400 Indore +91-9589613810, +91-731-4987379 Prayagraj +91-8114000926, +91-8114000927 Kanpur +91-8576066660, +91-8707605589 E: support@toprankers.com
Gurugram +91-8448444207 Delhi +91-9810610466 Ranchi +91-9334969993 Lucknow +91-6390576666 Jabalpur +91-7004386936 W: www.toprankers.com
32 The CLAT Post • November 2021

Passage (Q.141-Q.145):
India has promised to cut its
emissions to net zero by [1] -
missing a key goal of the COP26
summit for countries to commit
to reach that target by 2050.
Prime Minister Narendra Modi
made the pledge, the first time
India has set a net zero target, at
the Glasgow summit. Net zero,
or becoming carbon neutral,
means not adding to the amount
of greenhouse gases in the
atmosphere.
China has announced plans for
carbon neutrality by 2060, while
the US and EU aim to hit net
zero by 2050. The Indian leader is
one of more than 120 leaders to
have gathered in Glasgow for the
two-week conference.
Dozens gave speeches on
Monday laying out goals to tackle
the climate crisis, including UK
Prime Minister Boris Johnson,
US President Joe Biden and UN
Secretary General [2].

141. Which of the following


replaces [1] in the passage?
(a) 2055 (b) 2060
pledged to meet 50 (a) COP1 (b) COP3 Conference took place
(c) 2070 (d) 2075 percent of its energy in 1979.
(c) COP8 (d) COP16
requirements from (b) United Nations
142.Which of the following
renewable energy by 144.Which of the following Framework Convention
statements is NOT true?
2050. replaces [2] in the passage? on Climate Change
(a) India is the fourth
(d) Currently India (a) António Guterres came into effect in 1994.
biggest emitter of
produces more than (b) Catarina Vaz Pinto (c) COP1 Summit took
carbon dioxide in the
50% of the country’s (c) Ban Ki-Moon place at Kyoto, Japan in
world.
electricity from coal. 1997.
(b) India’s per capita (d) Kofi Annan
emission is 1.9 metric 143.Kyoto Protocol was adopted (d) Delhi Declaration was
tons. in which of the following 145.Which of the following is signed at COP8 in 2002.
Climate Summits? Incorrect?
(c) At COP26, India
(a) The First World Climate

141. (c) 2070 from coal. 145. (c) COP1 Summit took place
India has promised to cut its India is the fourth biggest 143. (b) COP3 at Kyoto, Japan in 1997.
emissions to net zero by 2070 emitter of carbon dioxide in Kyoto Protocol was adopted in The First World Climate
- missing a key goal of the the world and its per capita COP3 which took place from 1 Conference took place in 1979.
COP26 summit for countries emission is 1.9 metric tons. to 10 December 1997 in Kyoto, United Nations Framework
to commit to reach that target At COP26, India pledged to Japan. Convention on Climate
by 2050. meet 50 percent of its energy 144. (a) António Guterres Change came into effect in
142. (c) At COP26, India pledged requirements from renewable António Guterres is the ninth 1994. COP1 Summit took
to meet 50 percent of its energy energy by 2030. Currently Secretary-General of the place at Berlin, Germany in
requirements from renewable India produces more than 50% United Nations and took office 1995. Delhi Declaration was
energy by 2050. of the country’s electricity on 1st January 2017. signed at COP8 in 2002.
Bhopal +91-7676564400 Indore +91-9589613810, +91-731-4987379 Prayagraj +91-8114000926, +91-8114000927 Kanpur +91-8576066660, +91-8707605589 E: support@toprankers.com
Gurugram +91-8448444207 Delhi +91-9810610466 Ranchi +91-9334969993 Lucknow +91-6390576666 Jabalpur +91-7004386936 W: www.toprankers.com
33 The CLAT Post • November 2021

in 1945 by the United


Nations charter and
started working in April
1946.
(b) It is the principal
judicial organ of the
United Nations, situated
at the Peace Palace in
Brussels, Belgium.
(c) Unlike the six principal
organs of the United
Nations, it is the only
one not located in
Geneva, Switzerland.
(d) All of the above.

149.Which of the following


Passage (Q.146-Q.150): that he could mount a proper of 1963 is an Article of the United
The Pakistan government on appeal in Pakistani courts. The international treaty that Nations Charter lists the
Wednesday pushed through a bill law allows a foreign national defines a framework negotiation, enquiry,
to help Indian national [1] appeal to file a petition in a Pakistani for consular relations mediation etc. methods for
against the death sentence given high court for review and between independent the pacific settlement of
to him by a military court during reconsideration of a conviction states. disputes between States?
a joint session of Parliament that or sentence given by a military (b) A consul performs two (a) Article 38
was marred by protests by the court. functions: protecting (b) Article 33
opposition. in the host country
146. Which of the following (c) Article 22
The International Court of Justice replaces [1] in the passage? the interests of their
(Review and Re-consideration) countrymen, and (d) Article 6
(a) Kulbhushan Jadhav
Act of 2021 is on the same lines furthering the relations
as an ordinance issued last year (b) Sarabjit Singh 150. Kulbhushan Jadhav was
between the two states.
by the Pakistan government to sentenced to death by a
(c) Abhinandan Varthaman (c) Immunity given to
facilitate [1]’s right to appeal Pakistani military court
(d) Ravindra Kaushik consular is similar to in which of the following
against the death sentence given diplomat.
to him for alleged involvement 147.Which of the following years?
in espionage and subversive (d) All of the above (a) 2002
is true about Vienna
activities. Convention on consular 148.Which of the following is (b) 2010
Experts said the law would not relations? true about International (c) 2015
make a material difference to the (a) The Vienna Convention Court of Justice? (d) 2017
case of [1] and was more about on Consular Relations (a) ICJ was established
meeting legal requirements so

146.(a) Kulbhushan Jadhav independent states. The International Court maintenance of international
Kulbhushan Jadhav is an A consul performs two of Justice, is one of the six peace and security, shall,
Indian national who is on functions: protecting in the principal organs of the United first of all, seek a solution
the death row in Pakistan. host country the interests Nations. It is the principal by negotiation, enquiry,
He is accused of carrying of their countrymen, and judicial organ of the United mediation, conciliation,
out espionage and sabotage furthering the relations Nations, situated at the arbitration, judicial settlement,
activities against Pakistan between the two states. Peace Palace in The Hague, resort to regional agencies
at the behest of India’s Consular and Diplomat work Netherlands. Unlike the six or arrangements, or other
intelligence agency. India has out of the same premises, principal organs of the United peaceful means of their own
denied the allegations. and under this treaty they are Nations, it is the only one not choice.
147.(d) All of the above afforded most of the same located in New York, USA. 150.(d) 2017
The Vienna Convention on privileges. 149.(b) Article 33 Kulbhushan Jadhav was
Consular Relations of 1963 Immunity given to consular is Article 33 of the UN Charter sentenced to death by a
is an international treaty similar to diplomat. The treaty states that the parties to any Pakistani military court on
that defines a framework for has been ratified by 179 states. dispute, the continuance of charges of espionage and
consular relations between 148.(a) ICJ was established which is likely to endanger the terrorism in April 2017.
Bhopal +91-7676564400 Indore +91-9589613810, +91-731-4987379 Prayagraj +91-8114000926, +91-8114000927 Kanpur +91-8576066660, +91-8707605589 E: support@toprankers.com
Gurugram +91-8448444207 Delhi +91-9810610466 Ranchi +91-9334969993 Lucknow +91-6390576666 Jabalpur +91-7004386936 W: www.toprankers.com
34 The CLAT Post • November 2021

Bhopal +91-7676564400 Indore +91-9589613810, +91-731-4987379 Prayagraj +91-8114000926, +91-8114000927 Kanpur +91-8576066660, +91-8707605589 E: support@toprankers.com
Gurugram +91-8448444207 Delhi +91-9810610466 Ranchi +91-9334969993 Lucknow +91-6390576666 Jabalpur +91-7004386936 W: www.toprankers.com
35 The CLAT Post • November 2021

Bhopal +91-7676564400 Indore +91-9589613810, +91-731-4987379 Prayagraj +91-8114000926, +91-8114000927 Kanpur +91-8576066660, +91-8707605589 E: support@toprankers.com
Gurugram +91-8448444207 Delhi +91-9810610466 Ranchi +91-9334969993 Lucknow +91-6390576666 Jabalpur +91-7004386936 W: www.toprankers.com
36 The CLAT Post • November 2021

Bhopal +91-7676564400 Indore +91-9589613810, +91-731-4987379 Prayagraj +91-8114000926, +91-8114000927 Kanpur +91-8576066660, +91-8707605589 E: support@toprankers.com
Gurugram +91-8448444207 Delhi +91-9810610466 Ranchi +91-9334969993 Lucknow +91-6390576666 Jabalpur +91-7004386936 W: www.toprankers.com
37 The CLAT Post • November 2021

Bhopal +91-7676564400 Indore +91-9589613810, +91-731-4987379 Prayagraj +91-8114000926, +91-8114000927 Kanpur +91-8576066660, +91-8707605589 E: support@toprankers.com
Gurugram +91-8448444207 Delhi +91-9810610466 Ranchi +91-9334969993 Lucknow +91-6390576666 Jabalpur +91-7004386936 W: www.toprankers.com
38 The CLAT Post • November 2021

Bhopal +91-7676564400 Indore +91-9589613810, +91-731-4987379 Prayagraj +91-8114000926, +91-8114000927 Kanpur +91-8576066660, +91-8707605589 E: support@toprankers.com
Gurugram +91-8448444207 Delhi +91-9810610466 Ranchi +91-9334969993 Lucknow +91-6390576666 Jabalpur +91-7004386936 W: www.toprankers.com
39 The CLAT Post • November 2021

Bhopal +91-7676564400 Indore +91-9589613810, +91-731-4987379 Prayagraj +91-8114000926, +91-8114000927 Kanpur +91-8576066660, +91-8707605589 E: support@toprankers.com
Gurugram +91-8448444207 Delhi +91-9810610466 Ranchi +91-9334969993 Lucknow +91-6390576666 Jabalpur +91-7004386936 W: www.toprankers.com
40 The CLAT Post • November 2021

ONE LINERS
1)
The Asian Development Company Law Appellate has won the 2021 Booker the chairmanship of Italian
Bank (ADB) has approved a Tribunal (NCLAT), for a Prize for “The Promise”, Prime Minister
USD 250 million (about Rs period of four years or until his third shortlisted novel
1,875 crore) loan to support he attains the age of 70 years, which chronicles a family
India’s National Industrial whichever is the earliest. He in his homeland from the
was the former Chief Justice late apartheid era through
of Kerala High Court. to Jacob Zuma’s presidency.
NCLAT was formed by the Spanning several decades,
government under Section the book shows the family’s
410 of the Companies Act, growing disintegration as Mario Draghi. The summit
2013 the country emerges into concluded with the adoption
democracy. of the Rome Declaration by
4) The seventh Indian Navy G20 Leaders.
frigate of the P1135.6 class 7)
Addressing the COP26
Corridor Development was launched at the Yantar Glasgow Climate Summit 9) The Union Minister of
Program (NICDP). This is Shipyard at Kaliningrad in held in Scotland, Prime Cooperation, Amit Shah
the first subprogram of the Russia. The ship has been Minister Narendra Modi has launched the “Dairy
programmatic USD 500 formally named as Tushil, has said that India will reach Sahakar” scheme at
million loans to develop net-zero carbon emission by Anand, Gujarat during
11 industrial corridors 2070. PM Modi stressed on a function organised by
spanning 17 states a five-point plan Amul to celebrate the 75th
Foundation Year of Amul.
2) NITI (National Institution The total outlay of the Dairy
for Transforming India) Sahakar scheme is Rs 5000
Aayog and the World Bank crore. The scheme will be
(WB) are working together implemented by National
to facilitate a program for a Sanskrit word that means Cooperative Development
faster and easier financing protector shield. Tushil will Corporation ( NCDC)
be commissioned into the under the Ministry of
Indian Navy in mid-2023 Cooperation
followed by its sister ship by
10)
The Union Minister of
the end of 2023. or ‘Panchamrit’ to fight
State (Independent Charge)
the devastating effects of
5) A total of 84 Indian Air Earth Sciences Dr Jitendra
global warming and climate
Force (IAF) personnel took Singh officially launched
change. More than 120
part in the international India’s First Manned
world leaders were present
multilateral combat exercise Ocean Mission called the
of electric vehicles (EVs). at the COP26 Glasgow
Blue Flag 2021, along with “Samudrayan project” at
NITI Aayog and World Climate Summit. India
IAF’s Mirage 2000 aircraft Chennai. The Unique
Bank are setting up a is the last of the world’s
squadron at Israel’s Ovda
$300-million ‘first loss risk major carbon polluters to
Airbase. The theme of Blue
sharing instrument’, with announce a net-zero target.
Flag 2021: Integration of
the State Bank of India (SBI) China has said that it would
fourth and fifthgeneration
as its program manager. The reach the net-zero carbon
aircraft in complex
current rate of interest for emission goal in 2060 while
operational scenarios
electric two-wheelers & the United States and the
electric 3 Wheelers, which 6) South African playwright European Union are aiming
is in the range of 20-25 and novelist, Damon Galgut for reaching the goal by
Ocean Mission aims to have
per cent is expected to be 2050.
deep underwater manned
reduced to 10-12 per cent. 8) The 2021 G20 (Group of vehicles for carrying out
3) The Centre has appointed Twenty) Summit was held in subsea activities. With this
former Supreme Court Rome, Italy on October 30 technology, India joins the
Judge Justice Ashok and 31, 2021. It was the 16th elite club of nations such
Bhushan as the new meeting of the G20 group. as the USA, Russia, Japan,
Chairperson of the National The meeting was held under France and China to have
Bhopal +91-7676564400 Indore +91-9589613810, +91-731-4987379 Prayagraj +91-8114000926, +91-8114000927 Kanpur +91-8576066660, +91-8707605589 E: support@toprankers.com
Gurugram +91-8448444207 Delhi +91-9810610466 Ranchi +91-9334969993 Lucknow +91-6390576666 Jabalpur +91-7004386936 W: www.toprankers.com
41 The CLAT Post • November 2021
such underwater vehicles. Takeover aims to start broadband building, and institution-
services in India from building are exemplary.
11) Hurun India and EdelGive 14)
The International Day to
December 2022, with
have jointly released the End Impunity for Crimes • He is the key architect of
Edelgive Hurun India against Journalists is an UN- education in India. He
Philanthropy List 2021. recognized day observed was awarded Bharat Ratna
The list was topped by annually on 2 November. posthumously, India’s
Azim Premji, the founder highest civilian honour in
chairman of Wipro, with a 1992.
donation of Rs 9,713 crore
19)
The Union Cabinet
during fiscal 2020-21 which
2 lakh active terminals chaired by Prime Minister
is around Rs 27 crore a day.
subject to permission from Shri Narendra Modi has
HCL’s Shiv Nadar once
the government approved the declaration of
again retained the second
November 15 as Janjatiya
spot on the list with an 18)
In India, the National
The day draws attention to Gaurav Divas. 15 November
annual donation of Rs 1,263 Education Day is celebrated
the low global conviction has been chosen as this date
crore, marking a 59 per on 11 November every year
rate for violent crimes marks the birth anniversary
cent rise. Mukesh Ambani, to commemorate the birth
against journalists and of Sri Birsa Munda who is
the chairman of Reliance anniversary of Maulana
media workers, estimated at revered as Bhagwan (God)
Industries, and his family Abul Kalam Azad, the
only one in every ten cases. by tribal communities
occupied the third spot on first education minister of
across the country.
the philanthropy list with 15)
The International Day for independent India. The
an annual donation of Rs Preventing the Exploitation day was announced by 20)
The Meghalaya Cabinet
577 crore. of the Environment in War the Ministry of Human has approved the proposal
and Armed Conflict is an Resource Development of the creation of a new
12) Indian Grandmaster
international day observed on 11 September 2008. district called Eastern West
(GM) P Iniyan has won
annually on November 6. Maulana Abul Kalam Khasi Hills district. The new
the 5th Rujna Zora chess
Azad served as education district has been formed
tournament held at Serbia. 16) The Chennai-Mysore- minister from 15 August by upgrading the Mairang
International Master (IM) Chennai Shatabdi Express 1947 to 2 February 1958. civil subdivision. Mairang
Makarian Rudik of Russia has become the first train of will now be a sub-division
Southern Railway to receive under the West Khasi Hills
Integrated Management district. The new district
Systems (IMS) certification. will be inaugurated on
November 10, 2021 by
Meghalaya Chief Minister
Conrad Sangma. This will
increase the total number of
finished 2nd and another
districts in the state to 12
Indian Player V S Raaghul
finished 3rd and IM S. Nitin Maulana Abul Kalam 21)
The Government of the
placed 4th. P Iniyan is the It got the certificate for Azad was born in Mecca, United Kingdom (UK)
16th Indian Grand Master world-class maintenance Saudi Arabia in 1888. His has unveiled a £5 coin to
from Erode, Tamil Nadu. of the train, environment- mother was an Arab and commemorate the life and
His current International friendly resources and the daughter of Sheikh legacy of Mahatma Gandhi.
Chess Federation (FIDE) convenient and safe travel of Mohammad Zaher Watri It is the first time
ratings is 2556. the passengers. It is also the and Azad’s father, Maulana
first Shatabdi train and the Khairuddin, was a Bengali
13)
Ahmad Shah Ahmadzai, only second mail/express Muslim of Afghan origins
the former Prime Minister train of Indian Railways who came to Arab during
(PM) of Afghanistan and to receive this prestigious the Sepoy Mutiny and
a renowned jihadi leader certification proceeded to Mecca and
passed away at the age of settled there.
77 in Kabul, Afghanistan. 17) World’s richest person Elon
Ahmad Shah Ahmadzai Musk-owned SpaceX has • He came back to Calcutta with
incorporated its wholly- his family in 1890 when Mahatma Gandhi has
has served as the acting
owned subsidiary in India Abul Kalam was two years been commemorated on
PM of Afghanistan under
to start local broadband old. Maulana Abul Kalam an official UK coin. The
President Burhanuddin
operations. SpaceX’s satellite Azad’s contributions in the coin is available in a range
Rabbani during 1995-1996
broadband arm Starlink field of education, nation- of standards, including
prior to the 1996 Taliban
gold and silver, The
Bhopal +91-7676564400 Indore +91-9589613810, +91-731-4987379 Prayagraj +91-8114000926, +91-8114000927 Kanpur +91-8576066660, +91-8707605589 E: support@toprankers.com
Gurugram +91-8448444207 Delhi +91-9810610466 Ranchi +91-9334969993 Lucknow +91-6390576666 Jabalpur +91-7004386936 W: www.toprankers.com
42 The CLAT Post • November 2021
special collectors’ coin was Chattopadhyay also initiatives are the RBI Retail design by Mazagon Dock
designed by Heena Glover. authored novels like Direct Scheme and the Shipbuilders Limited (MDL)
Reserve Bank – Integrated Mumbai (Maharashtra)
22)
China has launched
Ombudsman Scheme. in collaboration with M/s
the world’s first Earth-
There will be a single point Naval Group, France.
science satellite, Guangmu
of reference for customers to Currently, three submarines
or SDGSAT-1 into
file their complaints, submit under Project-75 are
space from the Taiyuan
the documents, track status, functional with the Indian
Satellite Launch Center
and provide feedback. Navy viz. INS Karanj, INS
in the northern Shanxi
Kalvari, and INS Khanderi.
Province. The satellite was 29)
Domestic credit rating
launched by the Chinese “Patang” (2016), “Here Falls agency Brickwork Ratings 32) In continuation of a
Academy of Sciences the Shadow” (2017), and has estimated the gross tradition that started in
(CAS) and developed by “The Disappearance of Sally domestic product (GDP)
the International Research Sequeira” (2018) of India at 10-10.5 per cent
Center of Big Data for in the current financial
26) Former union minister and
Sustainable Development year, i.e. in 2021-22 (FY22).
Congress leader Salman
Goals (CBAS). Earlier this was estimated at
Khurshid, who recently
9 per cent.
23)
The United States of launched his book on the
America (USA) has joined Ayodhya verdict named 30) Vice Admiral R Hari Kumar
the International Solar “Sunrise over Ayodhya – has been named as the next 1950, Nepal Army Chief
Alliance (ISA) as a member Nationhood in our Times”. Chief of the Naval Staff by Gen Prabhu Ram Sharma
country. U.S is now the Khurshid said, “People used the Government of India. was conferred with the
to think that it will take He is presently posted as honorary rank of ‘General
100 years for the verdict to Flag Officer Commanding- of the Indian Army’ by
come. President Ram Nath
27)
The Reserve Bank of Kovind. Nepal conferred the
India has launched its honorary rank of ‘General
first global hackathon of Nepal Army’ to Indian
named “HARBINGER Army Chief Gen MM
101st country to sign the
2021 – Innovation for Naravane during his visit to
framework agreement of
Transformation”. The theme Kathmandu in November
the ISA. The framework
of HARBINGER 2021 is in-Chief Western Naval last year.
agreement was formally
‘Smarter Digital Payments’. Command. He will take on
signed by John Kerry, the 33)
The 32nd edition of the
The Hackathon invites the new role with effect from
U.S. Special Presidential India-Thailand Coordinated
November 30, 2021. He will
Envoy for Climate, at the Patrol (Indo-Thai CORPAT)
replace the incumbent Chief
COP26 climate summit in between the Indian Navy
of the Naval Staff, Admiral
Glasgow. and the Royal Thai Navy is
Karambir Singh, who will
being conducted from 12–
24) Fumio Kishida, leader of the complete his tenure on
14 November 2021. Indian
Liberal Democratic Party November 30, 2021.
Naval Ship (INS) Karmuk,
(LDP) has been re-elected
participants to identify and 31)
4th Scorpene submarine an indigenously built Missile
as the Prime Minister (PM)
develop solutions that have of the Project-75, Yard Corvette and His Majesty’s
of Japan following the
the potential to make digital 11878 was delivered to the Thailand Ship (HTMS)
victory of the LDP in the
payments accessible to the Indian Navy which will be Tayanchon, a Khamrosin
2021 Parliament election.
under-served, enhance commissioned as Class Antisubmarine Patrol
Fumio Kishida succeeded
the ease of payments and Craft, along with Maritime
Yoshihide Suga who
user experience while Patrol Aircraft from both
resigned as PM of Japan in
strengthening the security navies are participating in
September 2021.
of digital payments the CORPAT
25)
A new book titled ‘The and promote customer 34)
Chief of Defence Staff,
Cinema of Satyajit Ray’ protection. General Bipin Rawat will
written by the author
28)
Prime Minister Narendra inaugurate the 14th edition
Bhaskar Chattopadhyay
Modi has launched two INS (Indian Naval Ship) of ‘c0c0n’, an annual Hacking
and published by Westland
innovative customer-centric Vela. Project-75 includes and Cyber Security Briefing,
details the life of legendary
initiatives of the Reserve the construction of six which will be held virtually
Indian Filmmaker –
Bank of India (RBI). These submarines of Scorpene from Nov 10-13. The
‘Satyajit Ray’. Bhaskar
Bhopal +91-7676564400 Indore +91-9589613810, +91-731-4987379 Prayagraj +91-8114000926, +91-8114000927 Kanpur +91-8576066660, +91-8707605589 E: support@toprankers.com
Gurugram +91-8448444207 Delhi +91-9810610466 Ranchi +91-9334969993 Lucknow +91-6390576666 Jabalpur +91-7004386936 W: www.toprankers.com
43 The CLAT Post • November 2021
conference, which is being bestperforming countries of Mitrabha Guha scored Services Authorities Act
conducted by Kerala Police with higher climate his 2nd GM norm at the 1987. The Day is celebrated
in association with two performance for the third to make people aware of
non-profit organisations, year in a row.
Society for the Policing of
37) The Logistics Ease Across
Different States 2021 Index
was recently published by
the Ministry of Commerce
& Industry. This is the third Sheikh Russel International
edition of Index. In the GM Tournament 2021,
Bangladesh. the various provisions
41)
The first matches for under the Legal Services
the Birmingham 2022 Authorities Act and the
Cyberspace (POLCYB) right of the litigants.
Commonwealth Games
and Information Security
have been announced. 44) World Science Day for
Research Association
Women’s cricket is Peace and Development
(ISRA), would primarily
index, Gujarat, Haryana making its debut at the is celebrated each year on
discuss online scams
and Punjab were emerged as Commonwealth Games November 10 every year.
and defences during the
best performing states with with the T20 format. The This day is celebrated to
lockdown period. The
respect to mobility of goods last time cricket was played highlight the important role
theme of this year’s ‘c0c0n’
and efficiency of logistics in the multi-sporting that science play in society
is Improvise, Adapt and
chain. This index provide showpiece was in the 1998 and the need to engage the
Overcome.
ranking to states on the basis edition in Kuala Lumpur. wider public in debates on
35) India’s President Shri Ram of logistics infrastructure. The women’s cricket T20 emerging scientific issues.
Nath Kovind addressed competition will be held The year 2021 marks the
38) India ranked 18th out
the 51st Conference of at the Edgbaston Stadium 20th edition of World
of 30 countries in the
Governors and Lieutenant from the 29 July, with the Science Day for Peace and
1st edition of the Global
Governors at Rashtrapati bronze and gold medal Development. With climate
Drug Policy Index which
Bhavan in New Delhi. It is matches taking place on change becoming a serious
was released by the Harm
the fourth conference to be 7 August. India will face threat to the lives of billions
Reduction Consortium
Australia in the opening of people and the planet,
in November 2021. The
match when women’s cricket this year’s celebration will
index ranked Norway,
makes its debut at the 2022 highlight the importance
New Zealand, Portugal,
Commonwealth Games on of “Building Climate-Ready
the United Kingdom (UK),
July 29 while the final will Communities”.
and Australia as the top 5
be played on August 7. India
countries on humane and 45) Union Power Minister R K
in the early session, with
health-driven drug policies. Singh virtually inaugurated
presided over by President Pakistan playing Barbados,
Ram Nath Kovind. The 39)
India’s woman pistol star who was recently confirmed the diversion of the
Governors and Lieutenant Manu Bhaker and Iranian as the team from the West
Governors from all states Olympic champion Javad Indies that will take part in
and union territories across Foroughi has won the the Commonwealth Games.
the country attended the 10m air pistol mixed team
42) World Urbanism Day, also
conference along with gold at the inaugural ISSF
known as “World Town
Prime Minister Narendra President’s Cup at the
Planning Day”, is celebrated
Modi, Vice President Wroclaw in Poland.
on 8 November globally, to
Venkaiah Naidu, and Union 40)
International Master (IM) recognise and promote the
Home Minister Amit Shah. Mitrabha Guha from role of planning in creating Marusudar River of Pakal
36) India has been placed at 10th Kolkata, West Bengal livable communities Dul Hydro Electric Project
spot in the global Climate has become the 72nd in Kishtwar, J&K. Pakal
43)
In India, 09 November is Dul HE Project (1,000
Change Performance Index Grandmaster of India after
celebrated as “National MW) is being constructed
(CCPI) 2022 released by securing his 3rd and final
Legal Services Day” by Chenab Valley Power
Germanwatch on the side- Grandmaster (GM) norm
every year by all Legal Projects Pvt Limited
lines of the COP26. In at GM Third Saturday Mix
Services Authorities, (CVPPPL) and Jammu
2020 also India was at 10th 220, Novi Sad, Serbia. He
to commemorate the and Kashmir State Power
position. India has retained won this 3rd GM norm
enactment of the Legal Development Corporation.
its position as the top 10 against GM Nikola Sedlak
Bhopal +91-7676564400 Indore +91-9589613810, +91-731-4987379 Prayagraj +91-8114000926, +91-8114000927 Kanpur +91-8576066660, +91-8707605589 E: support@toprankers.com
Gurugram +91-8448444207 Delhi +91-9810610466 Ranchi +91-9334969993 Lucknow +91-6390576666 Jabalpur +91-7004386936 W: www.toprankers.com
44 The CLAT Post • November 2021
Marusudar River is a major 49) India finished the inaugural birthday is celebrated in the director of CBI and ED
tributary of the Chenab ISSF President’s Cup with the country. The decision have been appointed for
River. five medals, including two comes just ahead of state two-year tenure in office
Gold, two Silver and one polls in Punjab and Uttar by the Central Vigilance
46)
Srinagar, the summer
Bronze. The invitation- Pradesh where the farm Commission (CVC) Act,
capital of Jammu and
only tournament was protests were predicted 2003.
Kashmir, is among the 49
held at the Wroclaw in to dent the BJP’s electoral
cities selected worldwide to 53) Audit Diwas is celebrated
Poland, featuring the top- fortunes. Last year, in
join the UNESCO creative to mark the historic
12 shooters in each of the September, President Ram
cities network (UCCN). origin of the institution of
shotgun, pistol and rifle Nath Kovind gave his assent
Prime Minister Narendra Comptroller and Auditor
categories. India’s Manu for the three bills passed
Modi hailed the inclusion General of India (CAG)
Bhaker bagged two gold by the Parliament. What’s
as a “fitting recognition” for and the contribution it has
medals. Medal Winners ahead? Since the three farm
the vibrant cultural ethos made to the governance,
include: bills
of the old city. It has been transparency and
designated as a creative Gold accountability over the past
city of craft and folk arts, several years. Currently,
• 25m Rapid Fire pistol
UNESCO. former Lt. Governor of UT
Mixed Team competition:
of Jammu Kashmir G. C.
47)
The Government of Manu Bhaker
Murmu is serving as CAG
Delhi launched the
• 10m air pistol mixed team of India.
‘Shramik Mitra‘ scheme
competition: Manu Bhaker have already been passed
for construction workers. 54)
Prime Minister Narendra
Under the scheme, 800 Silver into law, the government Modi has inaugurated the
‘Shramik Mitras’ will reach will have to formally bring 341-km long Purvanchal
• Women’s 25m pistol silver three new bills to repeal
out to construction workers,
individual competition: them and get them passed
and spread awareness on
Rahi Sarnobat in both Houses.
the government schemes.
Delhi Govt has also • Men’s 10m Air Pistol The three contentious bills
increased the Dearness individual competition: are:
allowance for Unskilled, Saurabh Chaudhary
• The Farmer’s Produce
Semi-skilled workers, to Bronze Trade and Commerce
increase their salary by
• Men’s 10m Air Pistol (Promotion and
around 1%. Shramik Mitras
individual competition: Facilitation) Bill, 2020 Express in Sultanpur
will inform construction
workers registered by the Abhishek Verma
• The Farmers district, Uttar Pradesh. The
Construction Board at the (Empowerment and expressway links state capital
50) Former union minister and
ward level about assistance Protection) Agreement of Lucknow with Ghazipur
Congress leader Salman
schemes of the government. Price Assurance and Farm and has been constructed
Khurshid, who recently
Services Bill, 2020 at an estimated cost of
48)
An idol of Goddess launched his book on the
Rs 22,500 crore. The key
Annapurna that was stolen Ayodhya verdict named • The Farmers’ Produce feature of the Expressway is
from Varanasi about 100 “Sunrise over Ayodhya – Trade and Commerce the 3.2 km long airstrip to
years ago and later found in Nationhood in our Times”. (Promotion and enable landing and take-off
Canada recently is all Khurshid said, “People used Facilitation) Bill of Indian Air Force fighter
to think that it will take
52)
The central government planes in case of emergency.
100 years for the verdict to
come. of India promulgated two 55) The first-of-its-kind,
ordinances for extending dedicated fisheries business
51)
Prime Minister Narendra the tenure of Directors of the incubator has been
Modi has announced that Enforcement Directorate inaugurated in Gurugram
his government will repeal
of Haryana to nurture
the three contentious
fisheries start-ups under
agricultural laws and
real market-led conditions.
set to be back at its rightful requested protesting
The incubator is known as
place and will be gracing the farmers to go back to their
LINAC- NCDC Fisheries
Kashi Vishwanath Temple fields and homes. The
Business Incubation Centre
in Varanasi. It is 17 cm in announcement came on the
(ED) and Central Bureau (LlFlC). It was inaugurated
height, 9 cm in breadth and Gurupurab/Prakash Utsav
of Investigation (CBI) for by the Union Minister
4 cm in thickness. festival, when Sikhism
up to five years. At present, of Fisheries, Animal
founder Guru Nanak’s
Bhopal +91-7676564400 Indore +91-9589613810, +91-731-4987379 Prayagraj +91-8114000926, +91-8114000927 Kanpur +91-8576066660, +91-8707605589 E: support@toprankers.com
Gurugram +91-8448444207 Delhi +91-9810610466 Ranchi +91-9334969993 Lucknow +91-6390576666 Jabalpur +91-7004386936 W: www.toprankers.com
45 The CLAT Post • November 2021
Husbandry and Dairying, the state butterfly. Kaiser- Forest Department. To
Shri Parshottam Rupala. create awareness about the
importance of grass species,
56)
Union Minister, Piyush
promote their conservation,
Goyal virtually launched
and facilitate research in the
India’s first Digital Food
field.
Museum in Thanjavur,
Tamil Nadu. It is a 1,860- 62)
Pochampally village in
sqft museum codeveloped Yadadri Bhuvanagiri district
by the Food Corporation of Telangana, known for (US) was its biggest source,
of India (FCI) and its famous handwoven Ikat accounting for over 20%
Visvesvaraya Industrial and saris, was selected as one of of these funds. India is
Technological Museums, i-Hind is scientifically the best tourism villages by followed by China, Mexico,
Bengaluru (Karnataka) known as Teinopalpus the United Nations World the Philippines, and Egypt.
with an estimated outlay of imperialis. In literal terms, Tourism Organisation In India, remittances are
Rs 1.1 crore. The museum it means Emperor of India. (UNWTO). The award will projected to grow 3% in
is the first one-of-its-kind The butterfly is having a be given at the 24th session 2022 to USD 89.6 billion.
effort to depict India’s food wingspan of 90-120 mm. of the UNWTO general
65) The RBI has projected the
story from the beginning to It is found across six States assembly on December 2, in
CPI inflation at 5.3 per cent
India becoming the largest along Eastern Himalayas Madrid.
for 2021-22. According
food gain exporter in the at elevations from 6,000- 63)
The Union Minister to MoSPI data, inflation
country. 10,000 feet in a well-wooded of Heavy Industries in the food basket rose to
57) India has been re-elected to terrain. Mahendra Nath Pandey 0.85 per cent in October,
the Executive Board 60)
The government of has inaugurated the first air compared to 0.68 per cent
Arunachal Pradesh pollution control tower of in the preceding month.
approved the ‘Pakke Tiger the state of Uttar Pradesh in India’s retail inflation rate,
Reserve 2047 Declaration Noida. The Air measured by the Consumer
on Climate Change Resilient Price Index (CPI), rose
and Responsive Arunachal slightly in October to 4.48%
Pradesh’, which aims to year-on-year from 35% in
promote “climate-resilient September due to an uptick
of UNESCO for the term
development” in the state. in food prices. The inflation
2021-25. India received 164
The announcement is the was 61% in October last
votes to get re-elected to
first of its kind by any state year.
a four-year term of office. Pollution Control Tower
Apart from India, Japan, government in the country. (APCT) prototype has been 66)
The Reserve Bank of
Philippines, Vietnam, 61)
India’s first ‘grass developed by state-run India has announced to
Cook Islands and China conservatory’ or ‘germplasm Bharat Heavy Electricals Ltd introduce the Internal
have also been elected in conservation centre’ spread (Bhel). The indigenously Ombudsman mechanism
Group IV Asian and Pacific over an area of 2 acres was developed APCT is set up for the following two types
States category. UNESCO inaugurated at Ranikhet between the DND flyway of Non-Banking Financial
Executive Board consists of in Almora district of and the slip road to Noida Companies (NBFCs).
58 memberstates each with Uttarakhand. Expressway. The tower will
a four-year term of office. help to mitigate the rising
58)
Meghalaya state observed air pollution problem in the
the 44th edition of city.
‘Wangala’, the festival of 100 64)
According to the latest
Drums Festival begins. It is report by the World
a post-harvest festival of the Bank titled ‘World
Garos tribe which is being This conservatory Bank’s Remittance Prices These two types of NBFCs
held every year to honour is funded under the Worldwide Database’, India are Deposit-taking NBFCs
‘Saljong’, the Sun God of Central Government’s became the world’s largest (NBFCs-D) with 10 or more
Garos, which also marks the CAMPA (Compensatory recipient of remittances by branches and Non-Deposit
end of the harvest season. Afforestation Fund receiving USD 87 billion in taking NBFCs (NBFCsND)
Management and Planning 2021. The United States with asset size of Rs.5,000
59) State Cabinet of Arunachal
Authority) scheme and is crore and above having
Pradesh, headed by Chief
developed by the research public customer interface.
Minister Pema Khandu,
approved “Kaiser-iHind” as wing of the Uttarakhand 67)
The Navies of India and
Bhopal +91-7676564400 Indore +91-9589613810, +91-731-4987379 Prayagraj +91-8114000926, +91-8114000927 Kanpur +91-8576066660, +91-8707605589 E: support@toprankers.com
Gurugram +91-8448444207 Delhi +91-9810610466 Ranchi +91-9334969993 Lucknow +91-6390576666 Jabalpur +91-7004386936 W: www.toprankers.com
46 The CLAT Post • November 2021
France will carry out to enhance the cooperation Purab, as it marks an
the 6th edition of the between the participating important festival for the
biennial training exercise navies towards augmenting
“EX SHAKTI 2021” from the overall maritime
November 15 to 26, 2021 in security in the region.
Frejus, France. The Indian
69)
Union Minister of Youth
Affairs & Sports Shri
Anurag Thakur has
presented the first-ever 45. Denmark has topped Sikh community. Guru
SAI Institutional Awards to the rankings with a score of Nanak is the first of the
246 athletes and coaches in 2. An anti-bribery standard- ten Sikh gurus who is
New Delhi. A total of 162 setting organisation, known considered as the one who
athletes and 84 coaches as TRACE, measures brought enlightenment to
have been conferred awards business bribery risk in the world. He was born in
Army will be represented by
in the Outstanding Award 194 countries, territories, 1469 in a village named
the Gorkha Rifles Infantry
and Best Award category and autonomous and semi- Talwandi, which is currently
Battalion and France Army
for their performance in autonomous regions. located in Nankana Sahib,
will be represented bytroops
national and international 72)
National Press Day is Pakistan.
of the 21st Marine Infantry
competitions, with cash observed on 16th November
Regiment of the 6th 75)
The Habibganj railway
awards amounting to a total every year to celebrate
Light Armoured Brigade. station in Bhopal, Madhya
of Rs 85.02 Lakhs during the free and responsible press
The exercise will focus Pradesh has been renamed
ceremony at the Jawaharlal in India. On this day the
on Counter-Terrorism
Nehru Stadium. Press Council of India
operations and enhance
military cooperation and 70) According to the data from started functioning as a
interoperability between air quality and pollution city moral watchdog to ensure
the two Armies. Apart tracking service from IQAir, that the press maintains
from this, India and France a Switzerlandbased climate high standards and is
will also carry out biennial group, Delhi, Kolkata and not constrained by any
airforce training exercise Mumbai are among the top influence or threats. It also after 18th-century Gond
‘Exercise GARUDA’ , and ten most polluted commemorates the day Queen of Bhopal, Rani
biennial maritime training when the Press Council of Kamlapati. Prime Minister
exercise ‘Exercise VARUNA’. India started functioning. Shri Narendra Modi will
inaugurate the revamped
68) The 3rd edition of the 73)
The United Nations
Rani Kamlapati railway
Trilateral Maritime Exercise observes “International
station on November 15,
named SITMEX–21 is being Day for Tolerance” on 16th
during his visit to Bhopal.
held from 15 to 16 Nov November every year.
21 in the Andaman Sea. cities of the world. Delhi 76) India successfully
The Navies of the India, topped the list with AQI at launched the 41st Scientific
Singapore and Thailand will 556, Kolkata and Mumbai Expedition to Antarctica
participate in the event. recorded an AQI of 177 and on November 15, 2021. The
169, respectively, at 4th and first batch of its contingents
6th position. The cities with comprising of 23 scientists
the worst AQI indices also and support staff has
include Lahore, in Pakistan, reached the Indian Antarctic
and Chengdu, in China. The United Nations is station Maitri. Four more
committed to strengthening batches will be landing in
71) India has slipped to 82nd tolerance by fostering Antarctica by mid-January
position (drop by 5 slots mutual understanding 2022. The Indian Antarctic
from 2020) with a risk score among cultures and peoples. program began in 1981 and
Indian Naval Ship (INS) of 44 in the global list of
74)
Guru Nanak Jayanti is has completed 40 scientific
Karmuk is participating in 2021 TRACE Bribery Risk
observed every year as the expeditions
the 3rd edition from India. Matrix (TRACE Matrix)that
It is an indigenously built measures business bribery birth anniversary of the 77)
The Centre has approved
Missile Corvette. The event risks, which was released Sikh founder, Guru Nanak a new Rajya Sainik Board
is being hosted by Royal by TRACE International. In Dev Ji. This year marks the (RSB) for Ladakh. The
Thai Navy (RTN) in the 2020, India ranked 77 with a 552nd birth anniversary of Board will be an effective
Andaman Sea with an aim score of Guru Nanak, also known link between the Centre and
as Prakash Utsav or Guru the Ladakh Administration.
Bhopal +91-7676564400 Indore +91-9589613810, +91-731-4987379 Prayagraj +91-8114000926, +91-8114000927 Kanpur +91-8576066660, +91-8707605589 E: support@toprankers.com
Gurugram +91-8448444207 Delhi +91-9810610466 Ranchi +91-9334969993 Lucknow +91-6390576666 Jabalpur +91-7004386936 W: www.toprankers.com
47 The CLAT Post • November 2021
aircraft on February 27, on Independence Day.
2019. Kirti Chakra President The order of precedence of
Kovind accorded the these awards is Param Vir
second-highest peacetime Chakra, Ashoka Chakra,
gallantry award Kirti Mahavir Chakra, Kirti
Chakra (posthumously) Chakra, Vir Chakra and
Rajya Sainik Board will cities were surveyed. The to Sapper Prakash Jadhav Shaurya Chakra.
play an advisory role on cities are ranked based on for neutralising terrorists
81)
Union Education Minister
the matters relating to the three parameters, which in an operation in Jammu
Dharmendra Pradhan
exservicemen, war widows, are service level progress and Kashmir. His wife and
inaugurated the state-
widows and noncombatants, (SLP), certifications and mother received the
including serving soldiers citizen’s voice. Once again,
and their dependents. Zila Indore has been adjudged
Sainik welfare offices of Leh as the cleanest city of India
and Kargil will function for the fifth consecutive
under the newly constituted year. Indore is followed by
Rajya Sainik Board. Surat in Gujarat at second
78)
The Border Roads place and Vijaywada in
award. Shaurya Chakra
Organisation has received Andhra Pradesh in the third of-the-art Centre for
• Major Vibhuti Shankar
the Guinness World position Nanotechnology and Centre
Dhoundiyal was accorded
Record for constructing for Indian Knowledge
80) President of India, Ram Nath the Shaurya Chakra
& blacktopping the world System at IIT Guwahati.
Kovind presented Gallantry (posthumously) for his role
highest motorable road He also released a book on
Awards and Distinguished in an operation where five
passing through the 19,024 NEP 2020 implementation.
Service Decorations terrorists were eliminated
feet 0.73 inches Assam Education Minister
in Defence Investiture and 200 kg explosive
Ranoj Pegu was present on
Ceremony at Rashtrapati material was recovered.
the occasion. IIT Guwahati
Bhavan. Gallantry Awards His wife Lt Nitika Kaul and
has achieved excellent
have been instituted by the mother received the award.
rankings in the various
Government of India to • Naib Subedar Sombir
national and international
honour the acts of bravery was accorded the Shaurya
ranking systems.
and sacrifice of the officers/ Chakra posthumously for
(5798.251m) high Umlingla personnel of the Armed killing an A++ category 82) Union Home Minister, Amit
Pass in the Union Territory Forces as well as other terrorist during an operation Shah laid the foundation
of Ladakh. The 52-kilometre lawfully constituted forces in Jammu and Kashmir.
long Chisumle to Demchok and civilians. The order of His wife Suman Devi and
tarmac road was developed precedence of these awards mother Rajendra Devi
under Project HIMANK is Vir Chakra, Kirti Chakra received the honour. • The
(93RCC/753 BRTF) of BRO. and Shaurya Chakra. Vir President also conferred the
Lieutenant General Rajeev Chakra: Group Captain Shaurya Chakra to Major
Chaudhry, Director General Abhinandan Varthaman Maheshkumar Bhure who is
Border Roads, received the was accorded the Vir an alumnus of Sainik School for ‘Rani Gaidinliu Tribal
Guinness World Records Chakra by President Ram Satara. According to the Freedom Fighters Museum’
certificate. Nath Kovind for his role citation, Major Bhure led in Manipur, through video
in pushing back Pakistan’s an operation in which six conferencing. The museum
79)
President of India, Ram will be set up at Luangkao
fighter jets in February top terrorist commanders
Nath Kovind presented the village in the Tamenglong
2019. were killed. About the
Swachh Survekshan Awards district of Manipur, which
Gallantry Awards: The
2021 at the Swachh Amrit is the birthplace of freedom
prestigious awards are
Mahotsav organised by the fighter Rani Gaidinliu. The
usually conferred to the
Ministry of Housing and proposed museum is being
awarde es/Next-of-Kins
Urban Affairs in New Delhi. set up at an estimated cost of
(NoKs) by the President
2021 is the 6th edition of Rs 15 crore by the Ministry
at the Defence Investiture
Swachh Survekshan awards of Tribal Affairs.
Abhinandan Varthaman Ceremony held every year
in which as many as 4,320
was Wing Commander at the Rashtrapati Bhawan 83) The Union Minister of State
then. In the ensuing aerial in New Delhi. The gallantry (Independent Charge) for
dogfight, he shot down awards are announced twice the Ministry of Science and
a Pakistani F-16 fighter a year first on the occasion Technology, Dr Jitendra
of Republic Day and then
Bhopal +91-7676564400 Indore +91-9589613810, +91-731-4987379 Prayagraj +91-8114000926, +91-8114000927 Kanpur +91-8576066660, +91-8707605589 E: support@toprankers.com
Gurugram +91-8448444207 Delhi +91-9810610466 Ranchi +91-9334969993 Lucknow +91-6390576666 Jabalpur +91-7004386936 W: www.toprankers.com
48 The CLAT Post • November 2021
Singh inaugurated Project declaration to end the commissioned into the
SWADESH. The project current political crisis Indian Navy at the Naval
SWADESH is a first of its by Hamdok and Abdel Dockyard, Mumbai. This
kind largescale multimodal Fattah Al-Burhan, general is the first of the four
neuroimaging database commander of the Sudanese ‘Visakhapatnam’
designed specifically for Armed Force.
the Indian population. its own satellite in low-
The unique brain initiative earth orbit using an Anti-
has been developed by Satellite (ASAT) Missile
the DBTNational Brain named ‘Nudol’, which led
Research Centre (DBT- to the formation of a cloud
NBRC), Gurgaon, Haryana. of space debris that could
Stat potentially destroy other
class destroyers. It has
84)
Andhra Pradesh has been orbiting satellites & the
Before becoming prime been designed by the
named the best marine International Space Station
minister, Hamdok worked Indian Navy’s in-house
state in the country (ISS).
for the U.N. Economic organization Directorate
by the Department of 86) El Salvador President Nayib Commission for Africa, the of Naval Design and
Fisheries. The Department Bukele has announced that African Development Bank constructed by Mazagon
of Fisheries under the the country is planning and as a special adviser at Dock Shipbuilders
Ministry of Fisheries, to build the world’s first the Trade and Development Limited, Mumbai. INS
Animal Husbandry and “Bitcoin City”. The new city Bank in Ethiopia. Visakhapatnam measures
Dairying awarded best is planned to be developed 163m in length, 17m in
performing States for 2021- 89)
Wall Street brokerage,
in the eastern region of La breadth with a displacement
22 on 21 November 2021 on Goldman Sachs in its
Union and will be initially of 7,400 tonnes. It was
the occasion of the ‘World recent Macro Outlook
funded by bitcoinbacked commissioned in the
Fisheries Day, to recognise 2022 note revised upward
bonds. presence of Raksha Mantri
their accomplishments its projection for the gross
87)
Australia officially became domestic product (GDP) to Shri Rajnath Singh.
in the field and their
contribution to the growth a part of the new Nuclear 9.1 per cent, from the earlier 92) The 5-day long, 15th edition
of the sector. Top States: Powered Submarine estimate of 8 per cent for of the Maldives, India, and
Marine states the calendar year 2022. For
2021-22 (FY22), it pegged
economic growth at 8.5 per
cent.
90) Moody’s Investors Service in
its latest report has projected
that the economic growth in
defence alliance with the India will rebound strongly.
It has pegged GDP growth Sri Lanka biennial Trilateral
United Kingdom and the
for the nation at Exercise ‘Dosti’ was
: Andhra Pradesh; Inland United States after signing
conducted in the Maldives
states: Telangana; Hilly and a deal with the countries
from 20-24 November
North East states: Tripura. in Canberra, Australia
2021 as part of regional
Top Districts: Best Marine Under the AUKUS deal,
security information for a
District: Balasore in Odisha; Australia will be provided
peaceful and stable Indian
Best Inland District: with 8 nuclear-powered
Ocean Region. The Exercise
Balaghat in Madhya submarines capable of
is conducted biennially
Pradesh; Best Hilly and stealthy and long-range
among the Coast Guards
NE District: Bongaigaon in missions. It is the first 9.3% and 7.9% in FY22 and
of the 3 countries. This year
Assam agreement on technology FY23, respectively. India
marks the 30th year since
signed by the three countries recently hit record Covid-19
85)
Russian Navy successfully the initiation of the exercise
after the formation of the vaccination rates. Moody’s
test-fired the ‘Zircon’ defence alliance AUKUS notes that the vaccination 93)
Indian Navy has
Hypersonic Cruise Missile (Australia-UK-US). drive in India has gathered commissioned indigenously
from Frigate – Admiral pace after the second wave. builtScorpene-class
Gorshkov warship, which 88)
Sudan’s removed Prime
submarine Vela at Mumbai’s
rightly hit the test target Minister Abdalla Hamdok 91)
INS Visakhapatnam, a
Naval Dockyard. INS Vela
placed in the Russian Arctic was reappointed after P15B stealth guided-
is the fourth in Project 75
waters. Russia destroyed signing a political missile destroyer has been
Bhopal +91-7676564400 Indore +91-9589613810, +91-731-4987379 Prayagraj +91-8114000926, +91-8114000927 Kanpur +91-8576066660, +91-8707605589 E: support@toprankers.com
Gurugram +91-8448444207 Delhi +91-9810610466 Ranchi +91-9334969993 Lucknow +91-6390576666 Jabalpur +91-7004386936 W: www.toprankers.com
49 The CLAT Post • November 2021
series, following Kalvari, CORPAT. Indonesian 98) The US space agency NASA India’s White Revolution, Dr
Naval Ship KRI Sultan has launched a first-ofits- Verghese Kurien. He is also
Thaha Syaifuddin, (376) kind mission named DART nicknamed as “Milkman of
is participating from to change the path of an India”. To celebrate National
Indonesia. asteroid by intentionally Milk Day, the College of
crashing a spacecraft into Dairy Science & Technology
96)
Pratham NGO has been
it. DART stands for Double (CODST) and Guru
awarded the Indira Gandhi
Asteroid Redirection Test. Angad Dev Veterinary &
Khanderi, and Karanj. It is Peace Prize 2021 for its work
Animal Sciences University
expected to further boost on expanding the scope
(GADVASU) is organizing
Indian capability to defend for education in India. Its
“Milk Adulteration Testing
and secure its strategic pioneering work for more
Camp” on 25 and 26
sea lanes. It was built by than a quarter-century
November 2021.
Mazagon Dock Shipbuilders
Ltd in collaboration with
M/s Naval Group of France.
94) Army chief General, M M
Naravane has observed the
military exercise ‘Dakshin

The $325 million DART


in ensuring that every mission was launched into
child has access to quality orbit on November 24, 2021,
education. Its innovative from Vandenberg Space
use of digital technology Force Base in California
to deliver education. Its atop a SpaceX Falcon 9
regular evaluation of the rocket
Shakti’ being held here
with the Army and the quality of education. Its 99)
The United Nations
Air Force taking part in it. timely response in making designated International
The exercise began in the children learn amid Day for the Elimination of
deserts of Jaisalmer. T72, Covid-19 restrictions Violence Against Women
T-90 as well as Vijayanta 97)
NITI Aayog launched the
tanks of the Army and the inaugural SDG Urban ANHAD KAUR
IAF’s Dhruv and Rudha Index & Dashboard 2021-
helicopters, and the Jaguar 22, to further strengthen
fighter aircraft participated Sustainable Development
in the joint exercise. Goals (SDGs) localization
is celebrated worldwide
95) The 37th edition of India- and institute robust SDG
on November 25. The
Indonesia Coordinated monitoring at the city level.
day is celebrated to raise
Patrol (CORPAT) is The index is an SDG
awareness of the fact that
being held in the Indian women around the world
Ocean region from 23-24 are subject to various forms
November 2021. CORPAT PULKITGOYAL
of violence and the true
is held twice in a year to nature of the issue is often
ensure safety and security. It hidden. This year’s theme
was conducted for the first for the International Day for
the Elimination of Violence
against Women is “Orange
progress monitoring tool at the World: End Violence
the ULB level to highlight against Women Now!”.
the strengths and gaps of 100) Every year November
ULB-level data, monitoring, 26 is celebrated as National SWAPNIL DAS
time in 2002. Indigenously and reporting systems. Milk Day in India. The day
built Indian Naval Ship Shimla has topped among is being observed since 2014
(INS) Khanjar and Dornier the 56 urban areas while to commemorate the birth
Maritime Patrol Aircraft Dhanbad in Jharkhand is at anniversary of the Father of
is participating in the the bottom
Bhopal +91-7676564400 Indore +91-9589613810, +91-731-4987379 Prayagraj +91-8114000926, +91-8114000927 Kanpur +91-8576066660, +91-8707605589 E: support@toprankers.com
Gurugram +91-8448444207 Delhi +91-9810610466 Ranchi +91-9334969993 Lucknow +91-6390576666 Jabalpur +91-7004386936 W: www.toprankers.com
50 The CLAT Post • November 2021

Legal Current Affairs


1. The Supreme Court (SC) national level”. The data manage climate-related and the Scheduled Tribes
has appointed an expert will be used to update risks and opportunities, and (Prevention of Atrocities
committee, which is to the disclosure requirements Act), 1989 (SC/ST Act).
be overseen by a retired will be based on standards
(Source: https://
apex court judge Justice from New Zealand’s
indianexpress.com/article/
Raveendran Committee, in independent accounting
india/can-quash-sc-st-act-
the Pegasus case. Under the body. The law will force
cases-if-civil-offence-top-
case, the Union Government financial firms to assess not
court-7590718/)
is alleged to have used only their own investments,
Israeli spyware “Pegasus” but also to evaluate the
the National Population companies they are lending
5. The Madras High Court has
Register (Citizenship Act, money to, in terms of their
declared as unconstitutional
1955) and the electoral environmental impact. The
a reservation law passed by
register (Registration of disclosures will become
the Tamil Nadu legislative
Electors Rules, 1960), mandatory for financial
Assembly. The law envisaged
and Aadhaar (Aadhaar years beginning in 2023.
providing 10.5% internal
Act, 2016), ration card
(Source: https:// reservation to Vanniakula
(National Food Security
indianexpress.com/article/ Kshatriya community,
Act, 2013), passport
world/climate-change/ within the 20% earmarked
(Passport Act) and driving
new-zealand-passes- for Most Backward Classes
for surveillance on private licence databases (Motor
climate-change-disclosure- (MBCs) in education and
citizens. The court rejected Vehicles (Amendment)
laws-for-financial-firms- public employment. The
the government’s plea to Act, 2019). With a view
in-world-first-7583510/)
set up its own probe. The to simplify the provisions
court asserted that the of various sections of the
government appointment existing RBD Act, 1969 and
4. A Constitutional bench of
of probe would violate the to make it people friendly,
the Supreme Court of India,
settled judicial principle the amendment has been
consisting of Chief Justice
against bias, i.e., that ‘justice proposed.
of India N V Ramana and
must not only be done, but
(Source: https://www. Justices Surya Kant and
also be seen to be done’. On
thehindu.com/news/ Hima Kohli, observed that reservation was provided
account of the government’s
national/centre-wants- the apex court and high under the State within the
inaction to file a detailed
to-keep-birth-death- courts have the power to reservation for the Most
response to the allegations
database/article37203036. quash criminal cases filed Backward Classes and
made by the petitioners,
ece) under various ‘special Denotified Communities
the Court has constituted
Act, 2021. The second
a panel of experts under
Tamil Nadu Backward
former SC judge Justice R V
3. New Zealand has become Commission in 1983 held
Raveendran.
the first country to pass laws that the population of
(Source: https://www. requiring banks, insurers Vanniakula Kshatriyas was
thehindu.com/news/ and investment managers found to be 13.01% of the
national/supreme-court- to report the impacts of State’s total population.
ju dg ment-on-p egasus/ Therefore, provision
article37184269.ece) statutes’ including the of 10.5% reservation
SC/ST Act. The Supreme to a community with a
Court has inherent powers population of 13.01%
2. The Centre has proposed under Article 142 of the could not be called
amendments to the Constitution or that of disproportionate.
Registration of Births and the High Court under
(Source: https://www.
Deaths Act (RBD), 1969. It climate change on their Section 482 of the Code
thehindu.com/news/
will enable it to “maintain business. The new laws will of Criminal Procedure to
national/tamil-nadu/
the database of registered require financial firms to quash proceedings under
madras-hc-declares-105-
birth and deaths at the explain how they would the Scheduled Castes
v a n n i y a r- r e s e r v at i o n -
Bhopal +91-7676564400 Indore +91-9589613810, +91-731-4987379 Prayagraj +91-8114000926, +91-8114000927 Kanpur +91-8576066660, +91-8707605589 E: support@toprankers.com
Gurugram +91-8448444207 Delhi +91-9810610466 Ranchi +91-9334969993 Lucknow +91-6390576666 Jabalpur +91-7004386936 W: www.toprankers.com
51 The CLAT Post • November 2021
l aw - u n c o n s t i tu t i o n a l / Jobs will be provided article37359465.ece) mainly from Punjab and
article37278566.ece) in various companies, Haryana, on the borders
societies, trusts, and limited of Delhi for more than a
liability partnership firms 9. The National Legal Services year. The 3 farm laws were:
6. The Khasi Hills Autonomous situated in the state. The Day (NLSD) is celebrated Farmers Produce Trade and
District Council (KHADC) law will be applicable for a on 9th November every Commerce (Promotion and
in Meghalaya announced period of 10 years. Violation year to spread awareness Facilitation) Act, 2020: It
that it would introduce of any provision of this Act for ensuring reasonable is aimed at allowing trade
the ‘Khasi Inheritance of will be a punishable offence. fair and justice procedure in agricultural produce
Property Bill, 2021. The for all citizens. NLSD outside the existing APMC
(Source: https://
was first started by the (Agricultural Produce
economictimes.indiatimes.
Supreme Court of India in Market Committee)
com/news/india/haryana-
1995 to provide help and mandis. Farmers
govts-law-providing-75-
(Empowerment and
reservation-to-locals-in-
Protection) Agreement on
private-jobs-to-come-into-
Price Assurance and Farm
force-from-january-2022/
Services Act, 2020: It seeks
articleshow/87555809.cms)
to provide a framework
bill is aimed at “equitable for contract farming.
distribution” of parental Essential Commodities
8. A Supreme Court’s Bench of
property among siblings (Amendment) Act, 2020:
Justices D.Y. Chandrachud
in the Khasi community. If It is aimed at removing
and B.V. Nagarathna made
implemented, the proposed commodities such as
the observation in a recent
Bill would modify an age- support to poor and weaker cereals, pulses, oilseeds,
judgment based on an
old customary practice sections of the society. Free edible oils, onion and potato
appeal filed by Karnataka
of inheritance of the legal services are provided from the list of essential
against a High Court
matrilineal Khasi tribe. in matters before Civil, commodities.
decision to quash a case of
Criminal and Revenue
(Source: https://www. abetment of suicide against (Source: https://www.
Courts, Tribunals or any
thehindu.com/news/ g k t o d a y. i n / c u r r e n t -
other authority exercising
n at i on a l / o t h e r- s t at e s / affairs/3-farm-laws-
judicial or quasi-judicial
seek-public-opinion-on- repealed-after-1-year-of-
functions. It is observed
inheritance-bill-activists/ protest/)
to make the citizens of the
article37346833.ece)
country aware of the various
provisions under the
11. A Joint Parliamentary
Legal Services Authorities
7. The Haryana Government Committee (JPC) has
Act and the rights of the
has said that the a government officer, that finalised and adopted the
litigants. On this day, each
Employment of Local the judiciary should not draft report on The Personal
jurisdiction organizes legal
Candidates Act, 2020 will treat the mental health of Data Protection (PDP)
aid camps, Lok adalats, and
be implemented in the state a person with a one-size- Bill, 2019 by a majority.
legal aid programmes.
from 15th January 2022. It fits-all approach, upholding The Bill will be soon tabled
requires firms with 10 or the need for judges to be (Source: https://www. in the upcoming Winter
sensitive to the gravity jagranjosh.com/current- Session of Parliament. The
of mental health issues. af f ai r s / n at i on a l - l e g a l - JPC has got five extensions
It further noted that an ser vices-day-histor y- to submit a report on the
individual copes with a significance-indian-legal- Bill in two years. The Joint
threat — both: physical and s y s te m - l e g a l - s e r v i c e s - Parliamentary Committee
emotional, expressing (or authorities-act-1987- (JPC) on “Personal Data
refraining to express) love, national-legal-ser vices- Protection Bill, 2019”
loss, sorrow and happiness, authority-1636438044-1) met recently under the
varies greatly in view of the leadership of MP PP
multi-faceted nature of the Chaudhary.
more employees to reserve human mind and emotions. 10. The Prime Minister
(Source: https://www.
75% of all jobs offering announced the repeal of the
(Source: https://www. g k t o d a y. i n / c u r r e n t -
a salary of less than Rs. three contentious farm laws.
thehindu.com/news/ affairs/personal-data-bill-
30,000 a month for eligible The farm laws had witnessed
national/be-sensitive-to- adopted-by-jpc/)
candidates of State domicile. protests from farmers,
mental-health-issues-sc/
Bhopal +91-7676564400 Indore +91-9589613810, +91-731-4987379 Prayagraj +91-8114000926, +91-8114000927 Kanpur +91-8576066660, +91-8707605589 E: support@toprankers.com
Gurugram +91-8448444207 Delhi +91-9810610466 Ranchi +91-9334969993 Lucknow +91-6390576666 Jabalpur +91-7004386936 W: www.toprankers.com
52 The CLAT Post • November 2021

ARTICLES YOU CANNOT AFFORD TO MISS


CENTRE AMENDS RULES FOR EXTENSION
OF TENURE OF ED, CBI DIRECTORS
They can serve up to two years beyond two-year fixed tenure in ‘public interest’
A day after promulgating Directorate of Enforcement of the ED in 2020. The Finance Appointments Committee
two ordinances that would appointed under the Central Ministry said a November 19, of the Cabinet (ACC) gave a
allow the Centre to extend Vigilance Commission Act, 2018 order through which Mr. one-year extension to Home
the tenures of the Directors of 2003 (45 of 2003) in the Mishra, a 1984-batch Indian Secretary Ajay Kumar Bhalla,
the CBI and the Enforcement Central Government for such Revenue Service official, was beyond his two-year fixed
Directorate from two years to period or periods as it may appointed has been modified tenure that was to end on
up to five years, the Personnel deem proper on a case-to- with approval from the August 22. On May 28, one-
Ministry issued an order to case basis for reasons to be President and it would now be year extension in service was
amend the Fundamental Rules, recorded in writing, subject read for three years. He is to granted to Intelligence Bureau
1922 adding the two posts to to the condition that the total retire on November 17. Director Arvinda Kumar and
the list whose services can be term of such Secretaries or R&AW Secretary Samant
extended by up to two years Directors, as the case may be, The NGO Common Cause, Kumar Goel. Their two-year
beyond the two-year fixed who are given such extension represented by advocate fixed term would have ended
tenure in “public interest”.

The previous list comprised


Defence Secretary, Foreign
Secretary, Home Secretary,
Director, Intelligence Bureau
and Secretary, Research
and Analysis Wing. Though
Director, CBI, was also
mentioned in the previous
order, the Monday notification
adds the Delhi Special Police
Establishment Act, 1946 (25
of 1946) under which the
investigation agency’s head is
appointed.

The notification amended fifth


proviso of Clause (d) of Rule
56 of the Fundamental Rules,
1922. It said, “Provided also
that the Central Government
may, if it considers necessary
in public interest so to do,
give extension in service Prashant Bhushan, had
in service under this rule, on June 30.
to the Defence Secretary, challenged the retrospective
does not exceed two years or
Home Secretary, Director of extension in the Supreme (Source: https://www.
the period provided in the
Intelligence Bureau, Secretary Court. The SC on September thehindu.com/news/national/
respective Act or rules made
of Research and Analysis 8 said Mr. Mishra should not centre-amends-rules-for-
there under, under which their
Wing and Director of Central be given any further extension. extension-of-tenure-of-ed-
appointments are made.”
Bureau of Investigation An SC ruling in 2017 had fixed cbi-directors/article37508309.
appointed under the Delhi The Centre had retrospectively two-year tenure for Director, ece)
Special Police Establishment extended the tenure of Sanjay ED.
Act, 1946 (25 of 1946) and Kumar Mishra as the Director
Director of Enforcement in the On August 12, the
Bhopal +91-7676564400 Indore +91-9589613810, +91-731-4987379 Prayagraj +91-8114000926, +91-8114000927 Kanpur +91-8576066660, +91-8707605589 E: support@toprankers.com
Gurugram +91-8448444207 Delhi +91-9810610466 Ranchi +91-9334969993 Lucknow +91-6390576666 Jabalpur +91-7004386936 W: www.toprankers.com
53 The CLAT Post • November 2021

EXPLAINED | HOW TO REPEAL A LAW


Article 245 of the Indian Constitution which gives Parliament the power to make laws also gives
the legislative body the power to repeal them
Prime Minister Narendra Modi Law Secretary P.K. Malhotra repeal 58 obsolete laws and through the same procedure
in his address to the nation told PTI. “There is no other make minor amendments as any other Bills. It will have
on November 19 announced way,” said former Lok Sabha to the Income Tax Act, 1961 to cleared by both Houses of
the repeal of three contentious Secretary General P.D.T. and The Indian Institutes of Parliament and the President
farm laws and assured farmer Acharya. Responding to a Management Act, 2017. would give his assent to make
groups protesting against query by PTI, Mr. Acharya it a law.
these laws that the legislative said the government can repeal The Act was sixth such
process for the repeal would the three laws through a single repealing act, aimed at Also, this is the second time
be completed in the upcoming repealing bill. repealing laws, tabled by the the NDA government is taking
Winter Session of the Narendra Modi government. a U-turn. In 2015, ahead of
Parliament. Article 245 of the Constitution The Narendra Modi the Bihar elections, it agreed
which gives Parliament the government already repealed to bring back crucial clauses
The government can repeal power to make laws also gives 1,428 Acts during its first term. of the land acquisition law

the laws in two ways -- it can the legislative body the power of 2013 after its proposed
either bring a Bill to repeal the to repeal them through the Generally, laws are repealed to amendments to the law were
three laws or promulgate an Repealing and Amending either remove inconsistencies met with opposition in Rajya
ordinance that will have to be Act. The Act was first passed or after they have served Sabha.
subsequently replaced with a in 1950 when 72 Acts were their purpose. When new
Bill within six months. repealed. laws are enacted, old laws on (Source: https://www.
the subject are repealed by thehindu.com/news/national/
“For repeal, the power of The last time the Repealing inserting a repeal clause in the explained-how-to-repeal-a-
Parliament is the same as and Amending provision was new law. law/article37574716.ece)
enacting a law under the invoked was in 2019 when the
Constitution,” former Union Union government sought to The Repealing and Amending
(Amendment) Bill will pass
Bhopal +91-7676564400 Indore +91-9589613810, +91-731-4987379 Prayagraj +91-8114000926, +91-8114000927 Kanpur +91-8576066660, +91-8707605589 E: support@toprankers.com
Gurugram +91-8448444207 Delhi +91-9810610466 Ranchi +91-9334969993 Lucknow +91-6390576666 Jabalpur +91-7004386936 W: www.toprankers.com
54 The CLAT Post • November 2021

EXPLAINED | WILL MPLADS BE CHANGED


FOR POST-PANDEMIC NEEDS?
With the scheme back in force, will Centre heed MPs’ call to alter norms to buy smartphones,
laptops for poor students?
The story so far: Under the Narasimha Rao Government In case of a natural calamity, Commissioner, Navin Chawla,
Member of Parliament Local in 1993 with the grant of ₹50 the MPs from non-affected got funds under the scheme.
Area Development Scheme, lakh a year to each MP. This areas in both Houses of Finally, on May 6, 2010, the
every MP is entitled to ₹5 sum was increased to ₹1 Parliament can recommend scheme’s constitutional validity
crore a year, adding up to crore during 1994-95. The works estimated at a maximum was upheld. The Supreme
₹3,950 crore for the 790 third revision to ₹2 crore of ₹25 lakh a year in disaster- Court said in its judgment that
members. The fund is to happened in 1997-98. The hit places. mere allegations that the funds
be utilised for “creation of United Progressive Alliance were prone to misuse could not
durable community assets Government in 2011-12 raised What are the controversies? be the ground for scrapping
and for provision of basic the annual entitlement to ₹5 the scheme. However, it
The scheme was first
facilities, including community crore. There have been regular suggested improvements to the
challenged in 1999 by Jammu
infrastructure, based on locally demands from MPs across scheme.
and Kashmir National
felt needs”. This money does party lines to increase the Panthers Party chief Bhim
not directly go to the account amount further. When was the scheme
Singh and an NGO, Common suspended?
of the MPs. They can only

recommend works. Thereafter, How does it work?


Cause. They alleged that in the On April 6, 2020, the scheme
it is the responsibility of the Each Lok Sabha member has absence of any guidelines, the was suspended for two years.
district authorities to sanction, to designate a district as the funds were misused by MPs. The Government argued that
execute and complete the nodal district. The District In 2005, a sting operation it needed every last penny
works within the stipulated Magistrate is responsible showed some MPs allegedly to tackle the COVID-19
period. More money is for handling the funds and demanding money from pandemic. The scheme’s
released only on receipt of monitoring the projects contractors to award work for budget of ₹7,900 crore for
the completion certificate. sanctioned under the scheme. projects under the scheme. The two years was to be subsumed
The Ministry of Statistics and A Lok Sabha member can exposé led to the expulsion of under the Consolidated
Programme Implementation recommend works in his members from both Houses. Fund of India. This move
monitors the scheme. constituency alone, while a In 2006, the scheme made was severely criticised by the
When was the scheme started? Rajya Sabha member can use the headlines because of the Opposition MPs, who said
the funds for works anywhere allegations that a Trust run by the State Governments had
It was launched during the in a State. the family of the then Election already been cash-strapped,
Bhopal +91-7676564400 Indore +91-9589613810, +91-731-4987379 Prayagraj +91-8114000926, +91-8114000927 Kanpur +91-8576066660, +91-8707605589 E: support@toprankers.com
Gurugram +91-8448444207 Delhi +91-9810610466 Ranchi +91-9334969993 Lucknow +91-6390576666 Jabalpur +91-7004386936 W: www.toprankers.com
55 The CLAT Post • November 2021

and they were in a dire Cabinet brought back the 2021-22, it is only a partial for the funds to be spent on
need of the funds under the scheme, almost six months restoration, since instead of ₹5 smartphones and laptops
scheme. Senior Congress before its expected restoration. crore for each MP, the sum will for poor students to ensure
leader Shashi Tharoor argued According to a statement only be ₹2 crore. that they did not miss out on
that the scheme was the only from the Government, the online education in future as
means for an MP to direct country is on the road to What are the changes they did during the pandemic.
development resources to his economic recovery, and expected? This issue was also raised at
constituency. “Now the money the scheme continues to be a meeting of the Rajya Sabha
The COVID-19 pandemic has
will be allocated by the Centre beneficial to the creation of Standing Committee on
forced a significant change
and will follow the priorities durable community assets, the MPLADS on Friday.
in the policy decisions. The
and preferences of New Delhi, fulfilment of the aspirations Government may refine (Source: https://www.
rather than reflect 543 sets of the locally felt needs of the scheme to suit the post- thehindu.com/news/national/
of local needs,” he wrote on the community, the skill pandemic world. Currently, explained-will-mplads-be-
Twitter. development and creation of the funds are only to be changed-for-post-pandemic-
jobs across the country, thus spent on “durable assets”, but
When and why was it restored? helping to achieve the objective needs/article37481526.ece)
many MPs have demanded
of Atamnirbhar Bharat. For that the guidelines be altered
Last Wednesday, the Union

THE CONTROVERSIES AROUND THE FINAL


DRAFT OF CITIZENSHIP IN ASSAM
Why has the final NRC list not been notified yet? What are the contentions of the BJP led Gov-
ernment of Assam?
The story so far: In response to alleged that lakhs of “illegal lakh people who are waiting yet to start. The ministry had
a Right to Information (RTI) migrants” have been included to prove their citizenship for clarified that “non-inclusion
application by The Hindu, the in the register while genuine more than two years,” Mr. of a person’s name in NRC
State Coordinator, National citizens have been excluded. Wadud said. does not by itself amount to
Register of Citizens (NRC), In December 2020, the State him/her being declared as a
Assam said that 1032 doubtful Coordinator, NRC submitted What is the NRC? foreigner” as they would be
cases in the final draft of NRC, an affidavit to the Gauhati given adequate opportunity to
The NRC was first created in
Assam have been referred High Court containing the present their case before the
1951 in Assam to determine
to the concerned district details of “illegal foreigners” FTs.
those born there, therefore
commissioners for necessary whose names had crept into Indian, and who might be a
action. The Supreme Court the updated list of citizens What is the background to
migrant from erstwhile East update NRC?
monitored final draft of the released in 2019. The RTI Pakistan, now Bangladesh.
NRC was published on August response obtained by The It was a culmination of the
The register was updated on
31, 2019. The final citizens’ Hindu specify the number Assam Accord signed in
August 31, 2019. Out of 3.29
register is yet to be notified of such illegal entries in 1985 after the six-year long
crore applicants, 19.06 lakh
by the Registrar General of the database- 1,032. Assam Assam agitation, for detection,
were excluded from the final
India (RGI) that works under based lawyer Aman Wadud disenfranchisement and
draft NRC. The final NRC
the Union Home Ministry. said that the 2019 register is deportation of foreigners.
is yet to be notified by the
The State coordinator is a final and not supplementary. The Assam Public Works
Registrar General of India
Secretary rank officer of Assam “The Supreme Court on (RGI) under Home Ministry, (APW) moved the apex court
government who assists the five occasions has called the as those excluded will get in 2009, following which the
RGI as citizenship is a central register as final. Moreover, an opportunity to present process was revived. In 2013,
subject. the RGI has repeatedly asked their case before Foreigners following the court’s directive
the State Coordinator to
Why is it significant? Tribunals (FT), quasi-judicial a notification to update the
issue rejection slips to those bodies unique to the State. The register was published in the
excluded from the NRC, but
The BJP Government in Assam FTs can begin hearing only official gazette. A draft list was
it has refused to listen to
has rejected the NRC in its when the excluded applicants first published in June 2018
the RGI and questioned the receive “rejection slips” from and then June 2019, in total
current form and demanded
entire SC monitored process. the authorities. This process is excluding 41 lakh people.
a reverification. It has
This is gross injustice to 19
Bhopal +91-7676564400 Indore +91-9589613810, +91-731-4987379 Prayagraj +91-8114000926, +91-8114000927 Kanpur +91-8576066660, +91-8707605589 E: support@toprankers.com
Gurugram +91-8448444207 Delhi +91-9810610466 Ranchi +91-9334969993 Lucknow +91-6390576666 Jabalpur +91-7004386936 W: www.toprankers.com
56 The CLAT Post • November 2021

36 lakh people reapplied for What did the RTI reply say? been referred to the concerned declared foreigners, reference
inclusion at various NRC Deputy Commissioner pending at Foreigners
Seva kendras . It was updated In response to the RTI to take necessary action” Tribunals (FT), descendants
as per the provisions of The application filed by The as per Clause 4(3) & 4(6) of D voters, descendants of
Citizenship Act, 1955 and The Hindu, the Office of State of the Schedule 4(A) 4 of declared foreigners, cases of
declared foreigners pending
in FTs. Doubtful voters are
those whose cases are pending
in FTs or who have been
declared foreigners by the
tribunals and have challenged
the order. Rule 3 and 4 says
that the Local Registrar of
Citizen Registration may at
any time before the final-
publication “cause or direct to
cause verification of names”
and district registrar should
maintain that list separately at
village and ward level.

(Source: https://www.
thehindu.com/news/national/
the-controversies-around-the-
final-draft-of-citizenship-in-
Citizenship (Registration of Coordinator of NRC, Assam Citizenship Rules, 2003. The assam/article37556708.ece)
Citizens and Issue of National said that “1032 number of abbreviations cited above
Identity Cards) Rules, 2003. cases of DV/DF/PFT/DVD/ stand for doubtful voters,
DFD/PFTD categories have

EXPLAINED: THE ROAD TO DATA


PROTECTION LAW
On Monday, the Joint Parliamentary Committee (JPP) on the Personal Data Protection Bill of
2019 is said to have adopted the final draft. The Bill is slated to be tabled in the Winter Session.
Why does India need a data government in 2019 tabled the to have sought additional is said to have recommended
protection law? Personal Data Protection Bill compliance for companies that that no social media company
for the first time. deal exclusively with children’s be allowed to operate in India
Amid the proliferation of data, by asking them to register unless the parent company
computers and the Internet, What is said to be in the final with the Data Protection handling the technology sets
consumers have been draft? Authority — a regulatory body up an office in India.
generating a lot of data, that will have powers to decide
which has allowed companies One of the major changes that Other aspects such as
on implementing the law’s
to show them personalised the final draft of the PDP Bill setting up of an indigenous
various provisions.
advertisements based on their is believed to have pushed for architecture, which can
browsing patterns and other is to include non-personal A third key aspect that the be an alternative to the
online behaviour. Companies data within its ambit, which committee is said to have internationally accepted
began to store a lot of these changes the nature of the Bill pushed for is to consider all SWIFT payment system,
datasets without taking the from personal data protection social media companies as are also said to have been
consent of the users, and did to just data protection. publishers, and to hold them suggested.
not take responsibility when accountable for the content
the data leaked. To hold such on their platform if they are A key suggestion said to be
companies accountable, the not acting as intermediaries. It made by the JCP, which also
The final draft is also said
received the most dissent from
Bhopal +91-7676564400 Indore +91-9589613810, +91-731-4987379 Prayagraj +91-8114000926, +91-8114000927 Kanpur +91-8576066660, +91-8707605589 E: support@toprankers.com
Gurugram +91-8448444207 Delhi +91-9810610466 Ranchi +91-9334969993 Lucknow +91-6390576666 Jabalpur +91-7004386936 W: www.toprankers.com
57 The CLAT Post • November 2021

members, is wide-ranging Bill was “a blank cheque to the Twitter India, Amazon Web wider consultations on the
powers for the government state”. Services as well as Amazon various aspects of the Bill. The
such as exempting any agency India, among others, have committee, however, went
from application of the law. The Bill, which is said to made submissions to the panel. ahead with these deliberations.
contain 98 clauses, was
referred to the JPC headed Companies, tech policy groups
by BJP MP Meenakshi Lekhi and even JCP members had
When will it be tabled? in December 2019. Lekhi What were their submissions? also called for reconsideration
was replaced as chairperson of the one-size-fits-all
Since the final draft has been In their meeting with the
with another BJP MP, P P approach based on binary
adopted by all members of JPC, Google’s representatives
Chaudhary, earlier this year. age threshold for children,
the JCP, it is likely to be tabled had said India should avoid
The 30-member panel got given the vast geographic and
during the Winter Session. making data localisation a
extensions in March and cultural diversity of children
However, some of the JCP requirement, which had upset
September 2020 as well as a across the country and their
members have dissented the members of the committee.
final extension in February varying maturity levels and
to certain aspects, so some Paytm, on the other hand,
2021. needs.
changes are possible before the had said data generated in

Bill is tabled in Parliament. Officials from the Ministries India should be parked in
of IT, Law and Home Affairs, the country. Cab aggregators
the Unique Identification such as Ola and Uber, whose Companies and policy
Authority of India, National representatives appeared groups had also expressed
How long has it taken to
Investigation Agency, before the JPC earlier this apprehensions about the
complete the draft?
Narcotics Control Bureau and month, have supported data possible inclusion of certain
First proposed by the the Reserve Bank of India, localisation norms. clauses related to non-personal
government in 2018, the Bill among others, have deposed data and had told the JCP
In November last year, before that it carried a very high risk
has been pending for close to before the panel.
the JPC had started a clause- of re-identification and may
three years now. It has seen
From the private sector, by-clause consideration of lead to legal complications for
several changes to the original
executives from Visa, the Bill, several tech policy stakeholders.
draft drawn by retired Supreme
MasterCard India, Google groups had written to then
Court Justice B N Srikrishna,
India, PayTM, Facebook India, chairperson Lekhi, seeking Policy groups had repeatedly
who has said that the revised
Bhopal +91-7676564400 Indore +91-9589613810, +91-731-4987379 Prayagraj +91-8114000926, +91-8114000927 Kanpur +91-8576066660, +91-8707605589 E: support@toprankers.com
Gurugram +91-8448444207 Delhi +91-9810610466 Ranchi +91-9334969993 Lucknow +91-6390576666 Jabalpur +91-7004386936 W: www.toprankers.com
58 The CLAT Post • November 2021

objected to the blanket element of oversight,” said adjudication, which would citizens, businesses and the
exemptions to the central and Kazim Rizvi, founder of public end up “overburdening” the government themselves,” Rizvi
state governments along with policy group The Dialogue. architecture. said.
allied agencies.
Rizvi said that as per “The functional and structural (Source: https://indianexpress.
“We are hoping that these the 2019 draft, the Data independence of India’s first com/article/explained/
provisions have been relooked, Protection Authority had data regulator is a key aspect road-to-data-protection-
especially in the absence of a been entrusted with a wide considering the crucial role it law-7638088/)
comprehensive surveillance variety of functions ranging plays as the mediator between
framework, to introduce some from standard-setting to all vested stakeholders that is

Bhopal +91-7676564400 Indore +91-9589613810, +91-731-4987379 Prayagraj +91-8114000926, +91-8114000927 Kanpur +91-8576066660, +91-8707605589 E: support@toprankers.com
Gurugram +91-8448444207 Delhi +91-9810610466 Ranchi +91-9334969993 Lucknow +91-6390576666 Jabalpur +91-7004386936 W: www.toprankers.com
59 The CLAT Post • November 2021

DEATH PENALTY - COURT DUTY BOUND TO


CONSIDER POSSIBILITY OF REFORMATION
EVEN IF ACCUSED REMAINS SILENT :
SUPREME COURT
The Supreme Court on Friday the harshest punishment of Factual Background infirm person, were brutally
allowed the review petitions death sentence, even if the murdered in a pre-planned
filed by two death-row convicts accused is remaining silent. The Court was hearing the manner, to conclude that
to convert the death sentence Also, the State is under a review petitions filed by Mofil the case belonged to “rarest-
imposed on them to life duty to procure evidence Khan and Mobarak Khan who ofrarest” categorty deserving
imprisonment for a term of 30 to establish that there is no were sentenced to death for capital punishment.
years. possibility of reformation and the murder of eight persons in
rehabilitation of the accused. 2007. The death sentence was
A bench comprising Justices L upheld by the Jharkhand High

Nageswara Rao, BR Gavai and Court. In 2014, the Supreme After that, the petitioner filed
BV Nagarathna noted in the Court also upheld the death review petitions before the
order that the Court is duty In the instant case, the sentence by dismissing their Supreme Court against the
bound to elicit all relevant bench noted that the death appeals. The Courts took 2014 order which dismissed
information regarding the sentence was awarded without note of the fact that the eight their appeals.
possibility of the reformation reference to the possibility of persons, including innocent
reformation of the convicts. The Supreme Court then
of the convicts before imposing children and a physically
heard the review petitions in
Bhopal +91-7676564400 Indore +91-9589613810, +91-731-4987379 Prayagraj +91-8114000926, +91-8114000927 Kanpur +91-8576066660, +91-8707605589 E: support@toprankers.com
Gurugram +91-8448444207 Delhi +91-9810610466 Ranchi +91-9334969993 Lucknow +91-6390576666 Jabalpur +91-7004386936 W: www.toprankers.com
60 The CLAT Post • November 2021

the open court based on the reformation and rehabilitation of the Petitioners, the absence family of their siblings in a
judgment in Mohd. Arif v. of the accused”, the judgment of any criminal antecedents, pre-planned manner without
Registrar, Supreme Court of authored by Justice Nageswara affidavits filed by their family provocation due to a property
India. Rao stated, after referring to and community members dispute, the Court was of the
precedents. with whom they continue opinion that the Petitioners
Though Senior Advocate CU to share emotional ties and deserve a sentence of a period
Singh, appearing on behalf the certificate issued by the of 30 years.
of the petitioners, argued Jail Superintendent on their
that there was no sufficient “It is well-settled law that the Case Title : Mofil Khan and
conduct during their long
sentence to convict them in the possibility of reformation and another versus The State of
incarceration of 14 years.
first place, the Court said that rehabilitation of the convict is Jharkhand Coram : Justice
Considering all of the above,
under a review jurisdiction, it an important factor which has L Nageswara Rao, Justice
it cannot be said that there is
cannot re-appreciate evidence. to be taken into account as a BR Gavai and Justice BV
no possibility of reformation
mitigating circumstance before Nagarathna Citation : LL 2021
of the Petitioners, foreclosing
However, the Court noted that sentencing him to death. There SC 681
the alternative option of a
the aspect regarding possibility is a bounden duty cast on the
lesser sentence and making the
of reformation was ignored Courts to elicit information Appearances : Senior Advocate
imposition of death sentence
while awarding death sentence. of all the relevant factors and CU Singh for petitioners;
imperative. Therefore, we Advocate Prerna Singh for the
consider those regarding the
“One of the mitigating convert the sentence imposed State.
possibility of reformation, even
circumstances is the on the Petitioners from death
if the accused remains silent”,
probability of the accused to life” (Source: https://www.livelaw.
the judgment added.
being reformed and in/top-stories/death-penalty-
rehabilitated. The State is The Court further said : possibility-of-reformation-
under a duty to procure However, keeping in mind the supreme-court-review-
evidence to establish that “We have examined the commutes-186392)
gruesome murder of the entire
there is no possibility of socio-economic background

Bhopal +91-7676564400 Indore +91-9589613810, +91-731-4987379 Prayagraj +91-8114000926, +91-8114000927 Kanpur +91-8576066660, +91-8707605589 E: support@toprankers.com
Gurugram +91-8448444207 Delhi +91-9810610466 Ranchi +91-9334969993 Lucknow +91-6390576666 Jabalpur +91-7004386936 W: www.toprankers.com
61 The CLAT Post • November 2021

RESTRICTING ‘TOUCH’ OR ‘PHYSICAL


CONTACT’ ONLY TO ‘SKIN TO SKIN’
CONTACT ABSURD : SUPREME COURT
REVERSES BOMBAY HC’S POCSO JUDGMENT
The Supreme Court on situation. held that the word touch has of the words “touch” or
Thursday set aside the been referred to with reference “physical contact” to “skin to
controversial judgment of The Construction of rule to the special parts of the body skin contact” would not only
the Bombay HC which held should give effect to rule whereas the word ‘physical be a narrow and pedantic
that ‘skin-to-skin’ contact is rather than destroying it. The contact’ has been used for any interpretation of the provision
necessary for the offence of intention of legislature cannot other act, therefore the act of contained in Section 7 of the
sexual assault under Protection be given effect to unless wider touching the sexual part of POCSO Act, but it would lead
of Children from Sexual interpretation is given do. The the body if done with sexual to an absurd interpretation
Offences(POCSO) Act. purpose of the law cannot be intent would amount to sexual of the said provision. “skin to
to allow the offender to escape assault within the meaning of skin contact” for constituting
A bench comprising Justice the meshes of the law. S.7 of the POCSO Act”, Justice an offence of “sexual assault”
Uday Umesh Lalit, Justice S Trivedi said. could not have been intended
Ravindra Bhat and Justice Bela Since the Act does not define
or contemplated by the
M Trivedi pronounced the ‘touch’ or ‘physical contact’, When legislature has expressed Legislature. The very object
judgment in the appeals filed the dictionary meanings
by the Attorney General of
India, National Commission
for Women and the State
of Maharashtra against the
judgment of the High Court.

Important observations by the


Supreme Court

Justice Bela Trivedi, who read


out the operative portion
of the judgment, stated that
restricting ‘touch’ or ‘physical
contact’ under Section 7 of
POCSO is absurd and will
destroy the intent of the Act,
which is enacted to protect
children from sexual offences.

Restricting the meaning


of expression ‘touch’ and of enacting the POCSO Act
were referred. Act of touch, clear intention, the courts
‘physical contact’ under is to protect the children
if done with sexual intent, cannot create ambiguity in the
Section 7 of POCSO to “skin from sexual abuse, and if
will be an offence. Most provision. The Courts cannot
to skin contact” would not such a narrow interpretation
important ingredient is be overzealous in creating
only be narrow and pedantic is accepted, it would lead to
the sexual intent and not ambiguity.
interpretation but will also lead a very detrimental situation,
the skin-to-skin contact of
to absurd interpretation of the frustrating the very object of
the child. Sexual intent is a Main ingredient of offence is
provision. the Act, inasmuch as in that
question of fact which is to be “sexual intent” and not “skin-
case touching the sexual or
If such an interpretation determined from the attendant to-skin” contact.
non sexual parts of the body of
is adopted, a person who circumstances.
Justice Bela Trivedi’s lead a child with gloves, condoms,
uses gloves or any other like
“For the interpretation of S.7 judgment stated : sheets or with cloth, though
material while physical groping
of the POCSO Act, we have done with sexual intent would
will not get conviction for the “Restricting the interpretation
referred to the dictionary and not amount to an offence of
offence. That will be an absurd
Bhopal +91-7676564400 Indore +91-9589613810, +91-731-4987379 Prayagraj +91-8114000926, +91-8114000927 Kanpur +91-8576066660, +91-8707605589 E: support@toprankers.com
Gurugram +91-8448444207 Delhi +91-9810610466 Ranchi +91-9334969993 Lucknow +91-6390576666 Jabalpur +91-7004386936 W: www.toprankers.com
62 The CLAT Post • November 2021

sexual assault under Section covered by S.7 or is not touch an appeal, and referred to an sexual assault
7 of the POCSO Act. The at all. That provision is meant earlier criminal appeal filed by
most important ingredient to cover both direct and the Attorney General against
for constituting the offence of indirect touch. a judgment of the Rajasthan On August 6, the bench had
sexual assault under Section 7 High Court directing public appointed Senior Advocate
of the Act is the “sexual intent” The use of a spoon for an hanging of the accused(a case
instance, without touching Siddharth Dave as an amicus
and not the “skin to skin” of the year 1985). curiae in the matter. Along
contact with the child” it with the hand, in no way
diminishes the sense of touch Background with the above matter,
High Court trivialized and experienced by the lips and the the Supreme Court is also
legitimized unacceptable mouth. Similarly, when a stick As per the impugned considering the appeal filed
behaviour : Justice Bhat’s or other object is pressed onto judgment, the High by the State of Maharashtra
concurring opinion a person even when clothed, Court(Nagpur Bench) had against another controversial
their sense of touch is keen acquitted an accused observing judgment of the Bombay High
Justice S Ravindra Bhat, enough to feel that”. that groping of the breasts of a Court which held that the
who authored a separate minor girl over her clothes will act of holding a minor girl’s
but concurring judgment, “...the reasoning in the High not amount to the offence of hands and opening the zip of
observed that the High Court’s judgment quite ‘sexual assault’ under Section pants will not come under the
Court’s view legitimized insensitively trivializes 8 of POCSO. Holding that definition of “sexual assault”
an unacceptable behaviour - indeed legitimizes -an there should be ‘skin to skin’ under the Protection of
towards a child. entire range of unacceptable contact to attract the offence Children from Sexual Offences
behaviour which undermines under Section 8 POCSO, the Act 2012.
Justice Bhat’s judgment further a child’s dignity and autonomy, High Court held that the act in
state : through unwanted intrusions. question will only amount to a Both the judgments were
The High Court, therefore lesser offence of ‘molestation’ passed by Justice Pushpa
“The need to interpret a Ganediwala of Bobmay High
clearly erred in acting on such under Section 354 IPC.
statute in the context of the Court (Nagpur Bench).
interpretation, and basing its
circumstances that resulted The State of Maharashtra’s
conviction of and awarding The ‘skin-to-skin’ judgment
in its birth...Thereby I have counsel Rahul Chitnis told
sentence to the respondents; as was passed while modifying
moved down to the mischief the bench that the State
it did they were guilty of sexual the order of the Sessions Court
rule, the circumstances which was supporting the AG’s
assault” which held a 39-year-old man
led to this legislation, the arguments. The National
previous avtars which this law After the judgment was Commission for Women has guilty of sexual assault for
underwent. pronounced, Justice Lalit, also filed a separate appeal groping a 12- year- old- girl
the presiding judge thanked against the judgment. and removing her salwar.
The circumstances in which
the amicus curiae Senior In paragraph no. 26 of the
touch or physical contact Amicus Curiae in the matter,
Advocate Sidharth Dave, and impugned order, the Single
occurs would be determinative Senior Advocate Siddhartha
Senior Advocate Siddharth Judge had held that “there is
of whether it is motivated by Dave had argued that skin-
Luthra, who gave legal aid to no direct physical contact i.e.
sexual intent. There could be to-skin touching was not
the accused on behalf of the skin to skin with sexual intent
a good explanation for such required as the word ‘naked’
Supreme Court Legal Services without penetration”. “
physical contact which include was missing from S.7 of the
Committee.
the nature of the relationship POCSO. He had submitted
between the child and the “The act of pressing of breast
“I think this is the first time that “when two interpretations of the child aged 12 years, in
offender, the length of the the Attorney General has are possible, the interpretation
contact, its purposefulness the absence of any specific
challenged a judgment on the which is favorable to children detail as to whether the top
and also if there is a legitimate criminal side. Also this is the must be taken.”
non-sexual purpose for the was removed or whether he
first time brother and sister inserted his hand inside top
contact. It is also relevant to opposed each other”, Justice Senior Advocate Siddharth
see the conduct of the offender Luthra, representing the and pressed her breast, would
Lalit said. not fall in the definition of
before and after the contact. In accused on behalf of the Legal
this regard it is always useful Senior Advocate Siddharth Aid Society, had argued that ‘sexual assault’”, the judgment
to keep in mind that sexual Luthra is the brother of Senior since in the second part of held.
intent is not defined but fact Advocate Geeta Luthra, who S.7 the phrase ‘contact’ was (Source: https://www.livelaw.
dependent. appeared for the NCW. preceded by ‘physical’, the in/top-stories/pocso-skin-
import would be that skin-to- to-skin-judgment-supreme-
The fallacy in the High Court’s Justice Bhat then pointed out skin contact would be essential
reasoning is that is assumes that this is not the first instance court-bombay-high-court-
to constitute an offense of attorney-general-185784)
that indirect touch is not of Attorney General filing
Bhopal +91-7676564400 Indore +91-9589613810, +91-731-4987379 Prayagraj +91-8114000926, +91-8114000927 Kanpur +91-8576066660, +91-8707605589 E: support@toprankers.com
Gurugram +91-8448444207 Delhi +91-9810610466 Ranchi +91-9334969993 Lucknow +91-6390576666 Jabalpur +91-7004386936 W: www.toprankers.com
63 The CLAT Post • November 2021

INJUNCTION ORDERS CANNOT BE PASSED


AGAINST THIRD PARTIES WITHOUT
HEARING THEM: SUPREME COURT
The Supreme Court has held ex-parte ad-interim injunction. on the suit property would The Court took notice of the
that injunction orders with Though initially, the Trial be at the parties’ risk leaving fact that the Trial Court had
respect to a suit property Court had granted an ex- no room to claim equity. rejected to grant injunction on
cannot be passed in detriment parte injunction restraining Aggrieved, the third parties the very ground that the suit
to the interest of third parties the defendants in the suit having interest in the suit property was owned by firms,
who are directly affected by it, from alienating or creating property approached the trusts and companies that were
without impleading them or third party rights in the suit Supreme Court. not made parties to the suit.
giving them an opportunity of property, the application for Therefore, the Court thought
being heard. ex-parte ad-interim injunction indings of the Supreme Court it fit to quash and set aside the
unsustainable injunction order
A bench comprising of the High Court. It held -
Justices M.R. Shah and B.V.
Nagarathna set aside the “Therefore, the impugned
common judgment and order common judgment and order
passed by the Karnataka High passed by the High Court
Court, which had granted granting injunction with
injunction against alienation to respect to 1/7th share in the
the extent of 1/7th share in the total plaint schedule properties
suit property, without giving which has been passed without
the third parties (appellants), giving an opportunity of being
who have right, title or interest heard to the appellants and
in the property by way of without impleading them as
development agreements and/ party-defendants in the suit
or otherwise, an opportunity by the learned trial Court, is
of being heard. At the outset, the Court unsustainable and deserves to
was rejected on the ground
observed that, the respondents, be quashed and set aside.”
that certain portion of the
Factual Background who are original plaintiffs
suit property was owned by
have acknowledged that the The Trial Court was enjoined
Some of the respondents had firms, trusts and companies,
third parties (appellants) to first decide the impleadment
instituted a suit before the Trial that have not been made a
are necessary and property applications and thereafter,
Court seeking a declaration party to the suit. Some of the
parties by filing applications consider the interim injunction
that they are entitled to respondents filed appeals
to implead them as parties application afresh.
their mother’s 1/7th share before the High Court, which
to the suit. In view of the
in the total plaint schedule granted interim injunction (Source: https://www.livelaw.
same, the Court noted that
properties, with consequential against alienation till the in/top-stories/supreme-court-
before granting injunction,
relief of partition and separate disposal of the case along with injunction-orders-interest-of-
the appellants ought to have
possession. Subsequently, an the direction that any activity third-parties-oppurtunity-to-
been impleaded and given an
application was filed seeking pertaining to construction, be-heard-185940)
opportunity to be heard.
improvement, modification

JUVENILE JUSTICE ACT : AGE RECORDED


BY JJ BOARD OR CWC DEEMED TO BE TRUE
AGE OF ACCUSED - SUPREME COURT
The Supreme Court has held Welfare Committee of the A Bench comprising Justice challenging Allahabad High
that for the purpose of the person so brought before it will DY Chandrachud and Justice Court’s order sustaining the
Juvenile Justice Act 2015, the be deemed to be the true age of BV Nagarathna has made the judgment of the District &
age recorded by the Juvenile the person. observation while delivering Sessions Court as well as of
Justice Board or the Child its judgement in a plea the Juvenile Justice Board
Bhopal +91-7676564400 Indore +91-9589613810, +91-731-4987379 Prayagraj +91-8114000926, +91-8114000927 Kanpur +91-8576066660, +91-8707605589 E: support@toprankers.com
Gurugram +91-8448444207 Delhi +91-9810610466 Ranchi +91-9334969993 Lucknow +91-6390576666 Jabalpur +91-7004386936 W: www.toprankers.com
64 The CLAT Post • November 2021

declaring the accused a Therefore the Bench has (iii) Age has to be determined juvenile delinquent in case for
juvenile delinquent. held that the presumption of by an ossification test or offences under sections 147,
the accused being a juvenile any other medical age 148, 149, 323, 307, 302 and 34
In the present matter, victim may not be drawn when the determination test conducted of the Indian Penal Code.
of the crime had approached Committee or the Board has on the orders of the committee
the Supreme Court challenging reasonable grounds for doubt or the board. The District and Sessions
the declaration of the accused regarding the person brought Court and High Court
as a juvenile, arguing that the before it is a child or not. The Bench noted that in the sustained the Board’s order
matriculation certificate can’t instant case, there is no other holding that section 94 of the
be a conclusive document for “Thus, in the initial stage a document indicating the JJ Act, 2015 had been complied
determining the age of the presumption that the child date of birth of the person with in the instant case
juvenile irrespective of other brought before the Committee contrary to what has been inasmuch as the matriculation
material discrepancies in the or the JJ Board is a juvenile indicated in the matriculation or equivalent certificate from
oral testimony of the witnesses has to be drawn by the certificate and therefore such a the concerned Examination
or other documents being said authorities. The said discrepancy in the date of birth Board had indicated his date of
produced. presumption has to be drawn does not arise. birth to be 25.09.2004.
on observation of the child,
The Bench has held that the However, the said presumption While noting that no contra Therefore, it was held by the
deeming provision Section may not be drawn when the evidence to the documents impugned order that Sub-
94 (3) of the JJ Act, 2015 is Committee or the Board has produced by the second section 2 of Section 94 of the
significant inasmuch as ‘the reasonable grounds for doubt respondent have been JJ Act, 2015 applies in the
controversy or the doubt regarding the person brought produced by the appellant, the present case as there were no
regarding the age of the child reasonable ground to doubt
brought before the Committee the said document, and in the
or the JJ Board is sought to be absence of there being any
set at rest at the level of the JJ evidence to negate the same,
Board or the Committee itself ’. the criminal revision was
dismissed.
“The age recorded by the
Committee or the Board to Principles relating to
be the age of the person so determination of juvenility
brought before it shall for the
purpose of the JJ Act, 2015 The judgment authored
be deemed to be the true age by Justice Nagarathna also
of the person. The deeming summarized the principles
provision in sub-section (3) of relating to the determination
section 94 of the JJ Act, 2015 of juvenility as follows :
is also significant inasmuch as Bench refused to differ from (i) A claim of juvenility may
before it is a child or not.” the
the controversy or the doubt the order of the High court. be raised at any stage of a
Bench said.
regarding the age of the child criminal proceeding, even after
brought before the Committee The Bench has also specified In the present case, the a final disposal of the case. A
or the JJ Board is sought to be the evidence on which the Juvenile Justice Board had delay in raising the claim of
set at rest at the level of the JJ process of age determination observed that a letter dated juvenility cannot be a ground
Board or the Committee itself ” can be done by the Board in 22nd July 2020 issued by the for rejection of such claim. It
cases of such doubts: Office of the Administrative can also be raised for the first
According to the Bench, while Officer, Intermediate
Section 94 of the Act raises time before this Court.
(i) Date of birth certificate Education Council, Meerut,
a presumption regarding from the school or the UP, revealed that the date of (ii) An application claiming
juvenility of the age of the matriculation certificate birth of accused had rightly juvenility could be made either
child brought before the JJ from the concerned board, been recorded as 25.09.2004 in before the Court or the JJ
board or the Committee, in if available or in the absence the High School mark-sheet, Board.
case the Board or Committee thereof and he was 15 years and 8
has reasonable grounds about months of age as on the date of (iia) When the issue of
them being a child or not, it (ii) The birth certificate the incident. juvenility arises before a Court,
can undertake the process given by a corporation or by it would be under sub-section
of determination of age by a municipal authority or a Through its order dated 11th (2) and (3) of section 9 of the
seeking evidence. panchayat and in the absence November 2020, the JJ Board JJ Act, 2015 but when a person
of the above declared the accused as a is brought before a Committee
Bhopal +91-7676564400 Indore +91-9589613810, +91-731-4987379 Prayagraj +91-8114000926, +91-8114000927 Kanpur +91-8576066660, +91-8707605589 E: support@toprankers.com
Gurugram +91-8448444207 Delhi +91-9810610466 Ranchi +91-9334969993 Lucknow +91-6390576666 Jabalpur +91-7004386936 W: www.toprankers.com
65 The CLAT Post • November 2021

or JJ Board, section 94 of the JJ when an inquiry is made by be deemed to be the true age determination of age is on
Act, 2015 applies. a court before which the case of the person brought before the basis of evidence such as
regarding the commission of it. Thus, the standard of proof school records, it is necessary
(iib) If an application is filed the offence is pending (vide in an inquiry is different from that the same would have to
before the Court claiming section 9 of the JJ Act, 2015). that required in a proceeding be considered as per Section
juvenility, the provision of sub- where the determination and 35 of the Indian Evidence Act,
section (2) of section 94 of the (iii) That when a claim for declaration of the age of a inasmuch as any public or
JJ Act, 2015 would have to be juvenility is raised, the burden person has to be made on the official document maintained
applied or read along with sub- is on the person raising the basis of evidence scrutinised in the discharge of official duty
section (2) of section 9 so as to claim to satisfy the Court to and accepted only if worthy of would have greater credibility
seek evidence for the purpose discharge the initial burden. such acceptance. than private documents.
of recording a finding stating However, the documents
the age of the person as nearly mentioned in Rule 12(3) (vi) That it is neither feasible (x) Any document which
as may be. (a)(i), (ii), and (iii) of the JJ nor desirable to lay down an is in consonance with
Rules 2007 made under the JJ abstract formula to determine public documents, such as
(iic) When an application Act, 2000 or sub-section (2) the age of a person. It has to matriculation certificate, could
claiming juvenility is made of section 94 of JJ Act, 2015, be on the basis of the material be accepted by the Court or
under section 94 of the JJ shall be sufficient for prima on record and on appreciation the JJ Board provided such
Act, 2015 before the JJ Board facie satisfaction of the Court. of evidence adduced by the public document is credible
when the matter regarding the On the basis of the aforesaid parties in each case. and authentic as per the
alleged commission of offence documents a presumption of provisions of the Indian
is pending before a Court, then juvenility may be raised. (vii) This Court has observed Evidence Act viz., section 35
the procedure contemplated that a hyper-technical and other provisions.
under section 94 of the JJ Act, (iv) The said presumption is approach should not be
2015 would apply. Under the however not conclusive proof adopted when evidence is (xi) Ossification Test cannot
said provision if the JJ Board of the age of juvenility and adduced on behalf of the be the sole criterion for
has reasonable grounds for the same may be rebutted by accused in support of the plea age determination and a
doubt regarding whether the contra evidence let in by the that he was a juvenile. mechanical view regarding
person brought before it is a opposite side. the age of a person cannot
child or not, the Board shall (viii) If two views are possible be adopted solely on the
undertake the process of age (v) That the procedure of an on the same evidence, the basis of medical opinion by
determination by seeking inquiry by a Court is not the court should lean in favour radiological examination. Such
evidence and the age recorded same thing as declaring the of holding the accused to be evidence is not conclusive
by the JJ Board to be the age of age of the person as a juvenile a juvenile in borderline cases. evidence but only a very useful
the person so brought before sought before the JJ Board This is in order to ensure guiding factor to be considered
it shall, for the purpose of when the case is pending for that the benefit of the JJ Act, in the absence of documents
the JJ Act, 2015, be deemed trial before the concerned 2015 is made applicable to the mentioned in Section 94(2) of
to be true age of that person. criminal court. In case of an juvenile in conflict with law. the JJ Act, 2015.
Hence the degree of proof inquiry, the Court records a At the same time, the Court
required in such a proceeding prima facie conclusion but should ensure that the JJ Act, (Source: https://www.livelaw.
before the JJ Board, when an when there is a determination 2015 is not misused by persons in/top-stories/juvenile-justice-
application is filed seeking a of age as per sub-section (2) to escape punishment after act-age-recorded-by-jj-board-
claim of juvenility when the of section 94 of 2015 Act, a having committed serious or-cwc-deemed-to-be-true-
trial is before the concerned declaration is made on the offences. age-of-accused-supreme-
criminal court, is higher than basis of evidence. Also the age court-185987)
recorded by the JJ Board shall (ix) That when the

SALE DEED EXECUTED WITHOUT PAYMENT


OF PRICE IS VOID; HAS NO LEGAL EFFECT:
SUPREME COURT
The Supreme Court observed If a sale deed in respect of for the payment of price at Rastogi and Abhay S. Oka said.
that the payment of price is an an immovable property is a future date, it is not a sale
essential part of a sale. executed without payment of at all in the eyes of law, the The court also observed that a
price and if it does not provide bench comprising Justices Ajay document which is void need

Bhopal +91-7676564400 Indore +91-9589613810, +91-731-4987379 Prayagraj +91-8114000926, +91-8114000927 Kanpur +91-8576066660, +91-8707605589 E: support@toprankers.com
Gurugram +91-8448444207 Delhi +91-9810610466 Ranchi +91-9334969993 Lucknow +91-6390576666 Jabalpur +91-7004386936 W: www.toprankers.com
66 The CLAT Post • November 2021

not be challenged by claiming a decree for possession. The Transfer of Property Act, 1882, The court noted that no
a declaration as the said plea Trial Court dismissed the the bench observed: evidence was adduced by
can be set up and proved even suits filed by Kewal Krishan. Sudarshan Kumar about
in collateral proceedings. In appeal, the District Court Hence, a sale of an immovable the payment of the price
partly decreed the suits. The property has to be for a price. mentioned in the sale deeds
In this case, one Kewal Krishan High Court held that the suits The price may be payable in as well as the earning capacity
executed a power of attorney for declaration of invalidity of future. It may be partly paid at the relevant time of his wife
in favour of Sudarshan Kumar the sale deeds were barred by and the remaining part can and minor sons. Hence, the
on 28th March 1980. Acting on limitation as the said prayers be made payable in future. sale deeds will have to be held
the basis of the said power of were belatedly incorporated on The payment of price is an as void being executed without
attorney, two sale deeds were 23rd November 1985. essential part of a sale covered consideration, the court added.
executed by Sudarshan Kumar by section 54 of the TP Act. On the issue of limitation, the
on 10th April 1981. The first bench said:
sale deed was executed by him
by which he purported to sell “It was not necessary for the
a part of the suit properties appellant to specifically claim a
to his minor sons. The sale declaration as regards the sale
consideration was shown as deeds by way of amendment
Rs.5,500/-. The other sale deed to the plaint. The reason
was executed by Sudarshan being that there were specific
Kumar in favour of his wife pleadings in the plaints as
in respect of remaining part originally filed that the sale
of the suit properties. The deeds were void. A document
consideration shown in the which is void need not be
sale deed was of Rs.6,875/-. challenged by claiming a
declaration as the said plea can
Kewal Krishan filed two be set up and proved even in
separate suits. One was against In appeal, it was contended If a sale deed in respect of collateral proceedings. Hence,
Sudarshan Kumar and his that there was no evidence an immovable property is the issue of bar of limitation
two sons and the other one adduced to show that the executed without payment of of the prayers for declaration
was against Sudarshan Kumar purchasers under the sale price and if it does not provide incorporated by way of an
and his wife. Both the suits, deeds dated 10th April 1981 for the payment of price at amendment does not arise at
as originally filed, were for had paid consideration to a future date, it is not a sale all.”
injunction restraining the Sudarshan Kumar, and that at all in the eyes of law. It is
defendants from interfering the minor sons of Sudarshan of no legal effect. Therefore, (Source: https://www.livelaw.
with his possession and from Kumar and his wife had no such a sale will be void. It will in/top-stories/supreme-court-
alienating his share in the suit source of earning not effect the transfer of the sale-deed-without-payment-
properties. In the alternative, immovable property. price-void-186062)
a prayer was made for passing Referring to Section 54 of the

Bhopal +91-7676564400 Indore +91-9589613810, +91-731-4987379 Prayagraj +91-8114000926, +91-8114000927 Kanpur +91-8576066660, +91-8707605589 E: support@toprankers.com
Gurugram +91-8448444207 Delhi +91-9810610466 Ranchi +91-9334969993 Lucknow +91-6390576666 Jabalpur +91-7004386936 W: www.toprankers.com
67 The CLAT Post • November 2021

2 3

4 5

6 7

10

11

12 13

14 15 16

17

18

19

20

Across Down
4. The decision of a court of appeal ruling that the 1. A conscious, intentional wrongdoing either of a civil
judgment of a lower court was incorrect and is wrong or a criminal act
therefore reversed. 2. To get something from someone by trick and
7. Any proceeding before a judge or other magistrate falsehood.
9. Written document transferring property from one 3. Lacking a legal merit, not serious.
person to another. 5. Alteration of term of court order.
11. A piece of paper used in casting votes 6. An unofficial court wherein people are punished
12. If somebody in authority put an end to a system or unfairly
practice, they formally put an end to it 8. Government scheme providing advice or
13. Stopping an activity for a limited period by formal assistance from a solicitor or barrister free or at a
agreement reduced rate.
14. To send back. 10. The criminal offence of marrying one person while
15. A natural calamity which no one can prevent such still legally married to another
as an earthquake etc. 16. A person who is hiding in order to avoid being
18. It is an act of entering into somebody's house by caught by the police.
force and stealing things 17. A written document which leaves the estate of the
19. If a person in authority vetoes something it stops it person who signed the will to named person
from taking place.
20. Having the same mother but different fathers.

15. Fugitive 1O. Ballot 5. Variation


19. Uterine 14. Act of God 9. Bigamy 4. Reversal
18. Veto 13. Remand 8. Legal Aid 3. Frivolous
17. Burglary 12. Moratorium 7. Hearing 2. Defraud
16. Will 11. Abolish 6. Kangaroo court 1. Malice

Bhopal +91-7676564400 Indore +91-9589613810, +91-731-4987379 Prayagraj +91-8114000926, +91-8114000927 Kanpur +91-8576066660, +91-8707605589 E: support@toprankers.com
Gurugram +91-8448444207 Delhi +91-9810610466 Ranchi +91-9334969993 Lucknow +91-6390576666 Jabalpur +91-7004386936 W: www.toprankers.com
68 The CLAT Post • November 2021

INITIATIVE

LLM

BHOPAL INDORE
Head Office: 127, Next To Sbi, Zone II, M.P. Nagar, Bhopal (Mp) Corporate Office: 405-406, Tulsi Tower Geeta Bhawan Square, Indore (MP)

GURUGRAM PRAYAGRAJ
#NM 2&3, 1st Floor , Old DLF Colony, Sector 14, Gurugram, Haryana 122001 #1 Lohia Marg, Anant Kripa Bala Ji Apartment, Near Next To GHS Civil Lines Prayagraj (UP)

KANPUR RAIPUR
#24/147, 1st Floor, Kalpana Plaza, Birhana Rd, Patkapur, Near KPM Kanpur (UP) #29/650, 1st Floor, Beside SBI, New Shanti Nagar, Raipur (CG)

RANCHI LUCKNOW
#302, Vasundra Mart, Argora Opposite to Mahi Restaurant #21, Ashok Marg , Opp Doordarshan Kendra, Hazratganj Lucknow 226001 (UP)

Bhopal +91-7676564400 Indore +91-9589613810, +91-731-4987379 Prayagraj +91-8114000926, +91-8114000927 Kanpur +91-8576066660, +91-8707605589 E: support@toprankers.com
Gurugram +91-8448444207 Delhi +91-9810610466 Ranchi +91-9334969993 Lucknow +91-6390576666 Jabalpur +91-7004386936 W: www.toprankers.com

You might also like